IASbaba's Flagship Course: Integrated Learning Programme (ILP) - 2024  Read Details

Posts

[DAY 36] 60 DAY RAPID REVISION (RaRe) SERIES for UPSC Prelims 2024 – POLITY, CURRENT AFFAIRS & CSAT TEST SERIES!

Archives Hello Friends The 60 Days Rapid Revision (RaRe) Series is IASbaba’s Flagship Initiative recommended by Toppers and loved by the aspirants’ community every year. It is the most comprehensive program which will help you complete the syllabus, revise and practice tests on a daily basis. The Programme on a daily basis includes Daily Prelims MCQs from Static (Monday – Saturday) Daily Static Quiz will cover all the topics of static subjects – Polity, History, Geography, Economics, Environment and Science and technology. 20 questions will be posted daily and these questions are framed from the topics mentioned in the schedule. It will ensure timely and streamlined revision of your static subjects. Daily Current Affairs MCQs (Monday – Saturday) Daily 5 Current Affairs questions, based on sources like ‘The Hindu’, ‘Indian Express’ and ‘PIB’, would be published from Monday to Saturday according to the schedule. Daily CSAT Quiz (Monday – Friday) CSAT has been an Achilles heel for many aspirants. Daily 5 CSAT Questions will be published. Note – Daily Test of 20 static questions, 10 current affairs, and 5 CSAT questions. (35 Prelims Questions) in QUIZ FORMAT will be updated on a daily basis. To Know More about 60 Days Rapid Revision (RaRe) Series – CLICK HERE   60 Day Rapid Revision (RaRe) Series Schedule – CLICK HERE  Important Note Comment your Scores in the Comment Section. This will keep you accountable, responsible and sincere in days to come. It will help us come out with the Cut-Off on a Daily Basis. Let us know if you enjoyed today’s test 🙂  You can post your comments in the given format  (1) Your Score (2) Matrix Meter (3) New Learning from the Test Time limit: 0 Test-summary 0 of 35 questions completed Questions: 1 2 3 4 5 6 7 8 9 10 11 12 13 14 15 16 17 18 19 20 21 22 23 24 25 26 27 28 29 30 31 32 33 34 35 Information The following Test is based on the syllabus of 60 Days Plan-2023 for UPSC IAS Prelims 2022. To view Solutions, follow these instructions: Click on – ‘Start Test’ button Solve Questions Click on ‘Test Summary’ button Click on ‘Finish Test’ button Now click on ‘View Questions’ button – here you will see solutions and links. You have already completed the test before. Hence you can not start it again. Test is loading... You must sign in or sign up to start the test. You have to finish following test, to start this test: Results 0 of 35 questions answered correctly Your time: Time has elapsed You have scored 0 points out of 0 points, (0) Average score     Your score     Categories Not categorized 0% Your result has been entered into leaderboard Loading Name: E-Mail: Captcha: maximum of 70 points Pos. Name Entered on Points Result Table is loading No data available 1 2 3 4 5 6 7 8 9 10 11 12 13 14 15 16 17 18 19 20 21 22 23 24 25 26 27 28 29 30 31 32 33 34 35 Answered Review Question 1 of 35 1. Question Consider the following statements about the Goods and Services Tax (GST) Council of India: It is a constitutional body chaired by the Union Finance Minister. Every decision of this Council shall be taken by a majority of not less than three-fourths of the weighted votes of the members present and voting. In the Voting procedure, the Central Government will have 2/3rd of the votes, while all the state governments have 1/3rd voting power. How many of the above statements are correct? a) Only one b) Only two c) All three d) None Correct Solution (b) Statement 1 Statement 2 Statement 3 Correct Correct Incorrect GST Council of India was established with 101st amendment act. It is chaired by Finance Minister. The decisions in the council are take in accordance with the principle of voting of not less than three-fourths of the weighted votes of the members present and voting. The votes of all the state governments combined shall have a weightage of two-thirds of the total votes cast in that meeting and that of central government is one-thirds. Note: GST COUNCIL: Chairperson: Union Finance Minister OTHER MEMBERS: The Union Minister of State in charge of Revenue or Finance. The Minister in charge of Finance or Taxation or any other. Minister nominated by each State Government. The GST Council shall make recommendations to the Union and the States on: Taxes, cesses and surcharges which may be subsumed in the goods and services tax. The goods and services that may be subjected to, or exempted from the goods and services tax. The threshold limit of turnover below which goods and services may be exempted from goods and services tax. The rates including floor rates with bands of goods and services tax. Any special rates to raise additional resources during any natural calamity or disaster; Special provision to the States of Arunachal Pradesh, Assam, Jammu and Kashmir, Manipur, Meghalaya, Mizoram, Nagaland, Sikkim, Tripura, Himachal Pradesh and Uttarakhand. Incorrect Solution (b) Statement 1 Statement 2 Statement 3 Correct Correct Incorrect GST Council of India was established with 101st amendment act. It is chaired by Finance Minister. The decisions in the council are take in accordance with the principle of voting of not less than three-fourths of the weighted votes of the members present and voting. The votes of all the state governments combined shall have a weightage of two-thirds of the total votes cast in that meeting and that of central government is one-thirds. Note: GST COUNCIL: Chairperson: Union Finance Minister OTHER MEMBERS: The Union Minister of State in charge of Revenue or Finance. The Minister in charge of Finance or Taxation or any other. Minister nominated by each State Government. The GST Council shall make recommendations to the Union and the States on: Taxes, cesses and surcharges which may be subsumed in the goods and services tax. The goods and services that may be subjected to, or exempted from the goods and services tax. The threshold limit of turnover below which goods and services may be exempted from goods and services tax. The rates including floor rates with bands of goods and services tax. Any special rates to raise additional resources during any natural calamity or disaster; Special provision to the States of Arunachal Pradesh, Assam, Jammu and Kashmir, Manipur, Meghalaya, Mizoram, Nagaland, Sikkim, Tripura, Himachal Pradesh and Uttarakhand. Question 2 of 35 2. Question Consider the following remedial measures in the context of Centre-State Relations in India: There must be a permanent Inter-State Council to solve the federal issues. The mandatory consulting with the Chief Minister in the appointment of state Governor. The institution of All-India Services should be further strengthened and some more much services should be created. Which one of the following commissions has prescribed the above-given measures to smoothen the relationship between the Centre and State? a) Rajamannar Committee b) Sarkaria Commission c) Punchhi Commission d) Second Administrative Reforms Commission Correct Solution (b) SARKARIA COMMISSION: In 1983, the Central Government appointed a 3-member committee on Centre- state relations headed by R.S Sarkaria. A permanent inter-state Council called the Inter- Governmental Council should be set up under Article 263. National Development Council to be revamped as National Economic and Development Council. Article 356 should be used very sparingly, in extreme cases as a last resort when the entire available alternative fails. The institution of All-India Services should be further strengthened and some more much services should be created. The residuary powers of taxation should continue to remain with the parliament, while the other residuary powers should be placed in the concurrent lists. The centre should consult the states before marking a law on a subject of the concurrent list. Consulting with the Chief minister in the appointment of state governor. The zonal councils should be constituted afresh and reactivated to promote the spirit of federalism. PUNCHHI COMMISSION: They are planning to implement of the mega projects like the inter-linking of rivers. The role is to promoting effective devolution of powers and autonomy to Panchayati Raj Institution and Local Bodies under the sixth schedule of the constitution. Adopting approaches and policies based on positive discrimination in favour of backward states. The need and relevance of separate taxes on the production and on the sales of goods and services subsequent to the introduction of value added taxes regime. RAJMANNAR COMMITTEE: An inter-state Council should be set up immediately and Finance Commission should be made a permanent body. Planning Commission should be disbanded and its place should be taken by a statutory body. Article 356, 357 and 365 (dealing with the president rule) should be omitted. Certain subjects of the union list and concurrent should be transferred to the state list. The residuary powers should be allocated to the states. Incorrect Solution (b) SARKARIA COMMISSION: In 1983, the Central Government appointed a 3-member committee on Centre- state relations headed by R.S Sarkaria. A permanent inter-state Council called the Inter- Governmental Council should be set up under Article 263. National Development Council to be revamped as National Economic and Development Council. Article 356 should be used very sparingly, in extreme cases as a last resort when the entire available alternative fails. The institution of All-India Services should be further strengthened and some more much services should be created. The residuary powers of taxation should continue to remain with the parliament, while the other residuary powers should be placed in the concurrent lists. The centre should consult the states before marking a law on a subject of the concurrent list. Consulting with the Chief minister in the appointment of state governor. The zonal councils should be constituted afresh and reactivated to promote the spirit of federalism. PUNCHHI COMMISSION: They are planning to implement of the mega projects like the inter-linking of rivers. The role is to promoting effective devolution of powers and autonomy to Panchayati Raj Institution and Local Bodies under the sixth schedule of the constitution. Adopting approaches and policies based on positive discrimination in favour of backward states. The need and relevance of separate taxes on the production and on the sales of goods and services subsequent to the introduction of value added taxes regime. RAJMANNAR COMMITTEE: An inter-state Council should be set up immediately and Finance Commission should be made a permanent body. Planning Commission should be disbanded and its place should be taken by a statutory body. Article 356, 357 and 365 (dealing with the president rule) should be omitted. Certain subjects of the union list and concurrent should be transferred to the state list. The residuary powers should be allocated to the states. Question 3 of 35 3. Question Which one of the following statements is not correct? a) The President can delegate some executive functions of the centre to the state with the state’s consent. b) The Governor can delegate some executive functions of the state to the centre with the centre’s consent. c) The Parliament can delegate some executive functions of the centre to the state without the state’s consent. d) The State legislature can delegate its executive powers to the Centre without its consent. Correct Solution (d) State legislature cannot delegate its executive powers to Centre without the consent of Centre. MUTUAL DELEGATION BETWEEN CENTRAL AND STATE GOVERNMENTS: The Constitution provides for inter-government delegation of executive functions in order to mitigate rigidity and avoid a situation of deadlock. Accordingly, the President may, with the consent of the state government, entrust to that government any of the executive functions of the Centre. Conversely, the governor of a state may, with the consent of the Central government, entrust to that government any of the executive functions of the state. This mutual delegation of administrative functions may be conditional or unconditional. The Constitution also makes a provision for the entrustment of the executive functions of the Centre to a state without the consent of that state. But, in this case, the delegation is by the Parliament and not by the president. Thus, a law made by the Parliament on a subject of the Union List can confer powers and impose duties on a state, or authorise the conferring of powers and imposition of duties by the Centre upon a state (irrespective of the consent of the state concerned). Notably, the same thing cannot be done by the state legislature. From the above, it is clear that the mutual delegation of functions between the Centre and the state can take place either under an agreement or by legislation. While the Centre can use both the methods, a state can use only the first method. MUTUAL DELEGATION OF POWER BY ARTICLE 258 President can delegate the central government’s executive power to the state government with the consent of state government. Example: Centrally sponsored scheme Parliament can delegate central government’s executive power to the state government even without the consent of state government. This is because Rajya sabha represents states in parliament. Governor can delegate state government’s executive power to the central government with the consent of central government. State legislature cannot delegate such power as Centre’s representation is not in states. Incorrect Solution (d) State legislature cannot delegate its executive powers to Centre without the consent of Centre. MUTUAL DELEGATION BETWEEN CENTRAL AND STATE GOVERNMENTS: The Constitution provides for inter-government delegation of executive functions in order to mitigate rigidity and avoid a situation of deadlock. Accordingly, the President may, with the consent of the state government, entrust to that government any of the executive functions of the Centre. Conversely, the governor of a state may, with the consent of the Central government, entrust to that government any of the executive functions of the state. This mutual delegation of administrative functions may be conditional or unconditional. The Constitution also makes a provision for the entrustment of the executive functions of the Centre to a state without the consent of that state. But, in this case, the delegation is by the Parliament and not by the president. Thus, a law made by the Parliament on a subject of the Union List can confer powers and impose duties on a state, or authorise the conferring of powers and imposition of duties by the Centre upon a state (irrespective of the consent of the state concerned). Notably, the same thing cannot be done by the state legislature. From the above, it is clear that the mutual delegation of functions between the Centre and the state can take place either under an agreement or by legislation. While the Centre can use both the methods, a state can use only the first method. MUTUAL DELEGATION OF POWER BY ARTICLE 258 President can delegate the central government’s executive power to the state government with the consent of state government. Example: Centrally sponsored scheme Parliament can delegate central government’s executive power to the state government even without the consent of state government. This is because Rajya sabha represents states in parliament. Governor can delegate state government’s executive power to the central government with the consent of central government. State legislature cannot delegate such power as Centre’s representation is not in states. Question 4 of 35 4. Question Arrange the Following committee/commission  in a Chronological order which were setup to  examine the Centre-State relations in India? Sarkaria Commission Administrative Reforms Commission under Morarji Desai Rajamannar Committee Punchhi Commission Select the correct answer using the code given below: a) 2,4,1,3 b) 1,3,2,4 c) 3,4,2,1 d) 2,3,1,4 Correct Solution (d) Option 1 Option 2 Option 3 Option  4 1983 1966 1969 2007 In 1983, the central government appointed a commission on centre-state relations under the chairmanship of R.S. Sarkaria to examine and review the centre and states in all the spheres and recommend appropriate changes and measures. The central government appointed Administrative Reforms Commission (ARC) in 1966 under Moraji Desai to examine the various issues in Centre-state relations. In 1969, the state government appointed a new committee under the chairman of P V Rajmannar to examine the entire question of centre-state relations and to suggest amendments so as to secure utmost autonomy to the states. It is the recent commission on 2007 under the Madan Mohan Punchhi to look into the issues of centre-state relations in view the sea-changes that have taken place in the polity and economy of India. Incorrect Solution (d) Option 1 Option 2 Option 3 Option  4 1983 1966 1969 2007 In 1983, the central government appointed a commission on centre-state relations under the chairmanship of R.S. Sarkaria to examine and review the centre and states in all the spheres and recommend appropriate changes and measures. The central government appointed Administrative Reforms Commission (ARC) in 1966 under Moraji Desai to examine the various issues in Centre-state relations. In 1969, the state government appointed a new committee under the chairman of P V Rajmannar to examine the entire question of centre-state relations and to suggest amendments so as to secure utmost autonomy to the states. It is the recent commission on 2007 under the Madan Mohan Punchhi to look into the issues of centre-state relations in view the sea-changes that have taken place in the polity and economy of India. Question 5 of 35 5. Question With reference to the distribution of legislative powers between Centre and states in India, consider the following statements: The division of legislative powers in the constitution follows the scheme of Government of India act, 1935. The states have exclusive powers to make laws for matters listed in the state list of the Seventh Schedule in all the situations If a law made by the state on concurrent list subjects receives President’s assent, it cannot be overturned by the Parliament subsequently. How many of the above statements are correct? a) Only one b) Only two c) All three d) None Correct Solution (a) Statement 1 Statement 2 Statement 3 Correct Incorrect Incorrect Division of legislative power under seventh schedule in the constitution follows similar pattern of 3 lists division of government of India act 1935. States have the power to legislate on state lists under normal circumstances but parliament and even the president can legislate on state lists under extraordinary circumstances. Centre law prevails over state law in the concurrent list according to article 254 entailing doctrine of repugnancy but when the state legislature gets the assent of president in certain circumstances the state law prevails over the concurrent list in that circumstances. But Parliament can overturn it. Incorrect Solution (a) Statement 1 Statement 2 Statement 3 Correct Incorrect Incorrect Division of legislative power under seventh schedule in the constitution follows similar pattern of 3 lists division of government of India act 1935. States have the power to legislate on state lists under normal circumstances but parliament and even the president can legislate on state lists under extraordinary circumstances. Centre law prevails over state law in the concurrent list according to article 254 entailing doctrine of repugnancy but when the state legislature gets the assent of president in certain circumstances the state law prevails over the concurrent list in that circumstances. But Parliament can overturn it. Question 6 of 35 6. Question With respect to the Zonal Councils, which one of the following statements is correct? a) These are Constitutional bodies established by the 42nd Constitutional Amendment Act. b) It is headed by the Prime Minister of India. c) They are only a deliberative and advisory bodies. d) There are 4 zones in the country and there is a Zonal Council for each zone. Correct Solution (c) Zonal Councils: Zonal Councils are statutory body established by the State Reorganisation act of 1956. This act divides the country into 5 zones (East, West, North, South and Central). It is headed by Home Minister of the Central Government. It is aimed at promoting cooperation and coordination between states and Union Territories. It is only a deliberative and advisory body. COMPOSITION OF THE ZONAL COUNCIL: Chairman- The Union Home Minister Vice Chairman- The Chief Ministers of the States included in each zone by rotation, each holding office for a period of one year at a time. Members- Chief Minister and two other Ministers as nominated by the Governor from each of the States and two members from Union Territories included in the zone. FUNCTIONS: Zonal Council may discuss and make recommendations with regard to: Any matter of common interest, among the states, in the field of economic and social planning; Any matter concerning border disputes, linguistic minorities or inter-state transport; Any matter connected with or arising out of, the re-organization of the states. Incorrect Solution (c) Zonal Councils: Zonal Councils are statutory body established by the State Reorganisation act of 1956. This act divides the country into 5 zones (East, West, North, South and Central). It is headed by Home Minister of the Central Government. It is aimed at promoting cooperation and coordination between states and Union Territories. It is only a deliberative and advisory body. COMPOSITION OF THE ZONAL COUNCIL: Chairman- The Union Home Minister Vice Chairman- The Chief Ministers of the States included in each zone by rotation, each holding office for a period of one year at a time. Members- Chief Minister and two other Ministers as nominated by the Governor from each of the States and two members from Union Territories included in the zone. FUNCTIONS: Zonal Council may discuss and make recommendations with regard to: Any matter of common interest, among the states, in the field of economic and social planning; Any matter concerning border disputes, linguistic minorities or inter-state transport; Any matter connected with or arising out of, the re-organization of the states. Question 7 of 35 7. Question With reference to the North-Eastern Council (NEC), which of the following statements is/are correct? It comprises both Governors and Chief Ministers of all North Eastern States as its members. The Union Minister of Home Affairs is the Vice Chairman of NEC. Select the correct answer using the code given below: a) 1 only b) 2 only c) Both 1 and 2 d) Neither 1 nor 2 Correct Solution (a) Statement 1 Statement 2 Correct Incorrect The NEC Act 1971 was amended in 2002. The ‘North-Eastern Areas’ now means the area comprising the eight states of Assam, Manipur, Meghalaya, Nagaland, Tripura, Arunachal Pradesh, Sikkim, and Mizoram. Thus, NEC comprises eight states. The Council comprises of Governors and Chief Ministers of constituent States and three members to be nominated by the President as per Clause (iii) of Section 3 of the North Eastern Council (Amendment) Act, 1971. Minister of State (Independent Charge) Ministry of Development of North Eastern Region (DoNER) is the Vice Chairman of NEC.   Note: North Eastern Council (NEC): NEC was constituted as a statutory body under the NEC Act 1971. It was constituted for securing the balanced development of North Eastern Areas comprising of seven states -Arunachal Pradesh, Assam, Nagaland, Manipur, Meghalaya, Mizoram, Tripura, The NEC Act 1971 was amended in 2002. The ‘North-Eastern Areas’ now means the area comprising the eight states of Assam, Manipur, Meghalaya, Nagaland, Tripura, Arunachal Pradesh, Sikkim, and Mizoram. Thus, NEC comprises eight states. The Council comprises of Governors and Chief Ministers of constituent States and three members to be nominated by the President as per Clause (iii) of Section 3 of the North Eastern Council (Amendment) Act, 1971. The North Eastern Council is the nodal agency for the economic and social development of the North Eastern Region. Composition: Chairperson- Union Home Minister (ex-officio) Vice Chairperson- Minister of State (Independent Charge) Ministry of Development of North Eastern Region (DoNER) Members- Governor and Chief Minister of eight states. Incorrect Solution (a) Statement 1 Statement 2 Correct Incorrect The NEC Act 1971 was amended in 2002. The ‘North-Eastern Areas’ now means the area comprising the eight states of Assam, Manipur, Meghalaya, Nagaland, Tripura, Arunachal Pradesh, Sikkim, and Mizoram. Thus, NEC comprises eight states. The Council comprises of Governors and Chief Ministers of constituent States and three members to be nominated by the President as per Clause (iii) of Section 3 of the North Eastern Council (Amendment) Act, 1971. Minister of State (Independent Charge) Ministry of Development of North Eastern Region (DoNER) is the Vice Chairman of NEC.   Note: North Eastern Council (NEC): NEC was constituted as a statutory body under the NEC Act 1971. It was constituted for securing the balanced development of North Eastern Areas comprising of seven states -Arunachal Pradesh, Assam, Nagaland, Manipur, Meghalaya, Mizoram, Tripura, The NEC Act 1971 was amended in 2002. The ‘North-Eastern Areas’ now means the area comprising the eight states of Assam, Manipur, Meghalaya, Nagaland, Tripura, Arunachal Pradesh, Sikkim, and Mizoram. Thus, NEC comprises eight states. The Council comprises of Governors and Chief Ministers of constituent States and three members to be nominated by the President as per Clause (iii) of Section 3 of the North Eastern Council (Amendment) Act, 1971. The North Eastern Council is the nodal agency for the economic and social development of the North Eastern Region. Composition: Chairperson- Union Home Minister (ex-officio) Vice Chairperson- Minister of State (Independent Charge) Ministry of Development of North Eastern Region (DoNER) Members- Governor and Chief Minister of eight states. Question 8 of 35 8. Question With reference to the power of parliament to legislate on state list under Article 249 of the Indian Constitution, consider the following statements: Parliament can make laws on state lists if Rajya Sabha passes a resolution supported by two-thirds of the members present and voting, declaring that such a law is for the national interest. The parliament must legislate within one year of the passing of that resolution by Rajya Sabha. Once the parliament passes a law under Article 249, the state loses its law-making power on that matter. How many of the above statements are correct? a) Only one b) Only two c) All three d) None Correct Solution (b) Statement 1 Statement 2 Statement 3 Correct Correct Incorrect Article 249 empowers Parliament to legislate even on matters in the state list, provided not less than two-thirds of the members present and voting in the Rajya Sabha, the council of states, pass a resolution declaring that such legislation is necessary or expedient in the national interest. The resolution remains in force for one year. It can be renewed any number of times but not exceeding one year at a time. The laws cease to have effect on the expiration of six months after the resolution has ceased to be in force. So, parliament must legislate within one year of passing of that resolution by Rajya Sabha. This provision of parliament to legislate in state list does not restrict the power of a state legislature to make laws on the same matter. But, in case of inconsistency between a state law and a parliamentary law, the latter is to prevail. Note: SPECIAL POWERS OF RAJYA SABHA: Due to its federal character, the Rajya Sabha has been given two exclusive or special powers that are not enjoyed by the Lok Sabha: It can authorise the Parliament to make a law on a subject enumerated in the State List (Article 249). It can authorise the Parliament to create new All-India Services common to both the Centre and states (Article 312). An analysis of the above points makes it clear that the position of the Rajya Sabha in our constitutional system is not as weak as that of the House of Lords in the British constitutional system nor as strong as that of the Senate in the American constitutional system. Except in financial matters and control over the council of ministers, the powers and status of the Rajya Sabha in all other spheres are broadly equal and coordinate with that of the Lok Sabha. Incorrect Solution (b) Statement 1 Statement 2 Statement 3 Correct Correct Incorrect Article 249 empowers Parliament to legislate even on matters in the state list, provided not less than two-thirds of the members present and voting in the Rajya Sabha, the council of states, pass a resolution declaring that such legislation is necessary or expedient in the national interest. The resolution remains in force for one year. It can be renewed any number of times but not exceeding one year at a time. The laws cease to have effect on the expiration of six months after the resolution has ceased to be in force. So, parliament must legislate within one year of passing of that resolution by Rajya Sabha. This provision of parliament to legislate in state list does not restrict the power of a state legislature to make laws on the same matter. But, in case of inconsistency between a state law and a parliamentary law, the latter is to prevail. Note: SPECIAL POWERS OF RAJYA SABHA: Due to its federal character, the Rajya Sabha has been given two exclusive or special powers that are not enjoyed by the Lok Sabha: It can authorise the Parliament to make a law on a subject enumerated in the State List (Article 249). It can authorise the Parliament to create new All-India Services common to both the Centre and states (Article 312). An analysis of the above points makes it clear that the position of the Rajya Sabha in our constitutional system is not as weak as that of the House of Lords in the British constitutional system nor as strong as that of the Senate in the American constitutional system. Except in financial matters and control over the council of ministers, the powers and status of the Rajya Sabha in all other spheres are broadly equal and coordinate with that of the Lok Sabha. Question 9 of 35 9. Question Which of the following councils were created under the provisions of article 263 of the Indian Constitution? Central Council of Health North-East Council Central Council of Local Government and Urban Development Central Council of Indian Medicine Select the correct answer using the code given below: a) 1 and 3 only b) 2 and 3 only c) 1, 2 and 3 only d) 2 and 4 only Correct Solution (a) Under the above provisions of Article 263, the president has established the following councils to make recommendations for the better coordination of policy and action in the related subjects: Central Council of Health. Central Council of Local Government and Urban Development. Four Regional Councils for Sales Tax for the Northern, Eastern, Western and Southern Zones. The Central Council of Indian Medicine and the Central Council of Homoeopathy were set up under the Acts of Parliament. The North-Eastern Council was created by a separate Act of Parliament—the North-Eastern Council Act of 1971. INTER-STATE COUNCILS: Article 263 contemplates the establishment of an Inter-State Council to effect coordination between the states and between Centre and states. Thus, the President can establish such a council if at any time it appears to him that the public interest would be served by its establishment. He can define the nature of duties to be performed by such a council and its organisation and procedure. Even though the president is empowered to define the duties of an interstate council, Article 263 specifies the duties that can be assigned to it in the following manner: enquiring into and advising upon disputes which may arise between states; investigating and discussing subjects in which the states or the Centre and the states have a common interest; and making recommendations upon any such subject, and particularly for the better co-ordination of policy and action on it. The council’s function to enquire and advice upon inter-state disputes is complementary to the Supreme Court’s jurisdiction under Article 131 to decide a legal controversy between the governments. The Council can deal with any controversy whether legal or non-legal, but its function is advisory unlike that of the court which gives a binding decision. Incorrect Solution (a) Under the above provisions of Article 263, the president has established the following councils to make recommendations for the better coordination of policy and action in the related subjects: Central Council of Health. Central Council of Local Government and Urban Development. Four Regional Councils for Sales Tax for the Northern, Eastern, Western and Southern Zones. The Central Council of Indian Medicine and the Central Council of Homoeopathy were set up under the Acts of Parliament. The North-Eastern Council was created by a separate Act of Parliament—the North-Eastern Council Act of 1971. INTER-STATE COUNCILS: Article 263 contemplates the establishment of an Inter-State Council to effect coordination between the states and between Centre and states. Thus, the President can establish such a council if at any time it appears to him that the public interest would be served by its establishment. He can define the nature of duties to be performed by such a council and its organisation and procedure. Even though the president is empowered to define the duties of an interstate council, Article 263 specifies the duties that can be assigned to it in the following manner: enquiring into and advising upon disputes which may arise between states; investigating and discussing subjects in which the states or the Centre and the states have a common interest; and making recommendations upon any such subject, and particularly for the better co-ordination of policy and action on it. The council’s function to enquire and advice upon inter-state disputes is complementary to the Supreme Court’s jurisdiction under Article 131 to decide a legal controversy between the governments. The Council can deal with any controversy whether legal or non-legal, but its function is advisory unlike that of the court which gives a binding decision. Question 10 of 35 10. Question Suppose, two of the Indian states request the Parliament to enact a law on a matter in the state list and the Parliament accepts their request. In this context, which one of the following statements will hold true as per the Constitution? The law applies only to these two states and cannot be adopted by any third state. If the two states want to repeal the law passed by the parliament they cannot do so on their own. This provision of the constitution has not been used so far in the Indian Constitutional history since 1950. How many of the above statements are correct? a) Only one b) Only two c) All three d) None Correct Solution (a) Statement 1 Statement 2 Statement 3 Incorrect Correct Incorrect When the legislatures of two or more states pass resolutions requesting the Parliament to enact laws on a matter in the State List, then the Parliament can make laws for regulating that matter. A law so enacted applies only to those states which have passed the resolutions. It cannot be applied to any other states that do not request the parliament. However, any other state may adopt it afterwards by passing a resolution to that effect in its legislature. Such a law can be amended or repealed only by the Parliament and not by the legislatures of the concerned states. The state legislature ceases to have the power to make a law with respect to that matter. This provision of the constitution has been used. Some examples of laws passed under the above provision are Prize Competition Act, 1955; Wild Life (Protection) Act, 1972; Water (Prevention and Control of Pollution) Act, 1974; Urban Land (Ceiling and Regulation) Act, 1976; and Transplantation of Human Organs Act, 1994. Incorrect Solution (a) Statement 1 Statement 2 Statement 3 Incorrect Correct Incorrect When the legislatures of two or more states pass resolutions requesting the Parliament to enact laws on a matter in the State List, then the Parliament can make laws for regulating that matter. A law so enacted applies only to those states which have passed the resolutions. It cannot be applied to any other states that do not request the parliament. However, any other state may adopt it afterwards by passing a resolution to that effect in its legislature. Such a law can be amended or repealed only by the Parliament and not by the legislatures of the concerned states. The state legislature ceases to have the power to make a law with respect to that matter. This provision of the constitution has been used. Some examples of laws passed under the above provision are Prize Competition Act, 1955; Wild Life (Protection) Act, 1972; Water (Prevention and Control of Pollution) Act, 1974; Urban Land (Ceiling and Regulation) Act, 1976; and Transplantation of Human Organs Act, 1994. Question 11 of 35 11. Question In the context of Centre-State relations in India, if a particular Subject is not listed in any of the lists of the Seventh Schedule, then what would be the best option to make law on such subject? a) Only Parliament can enact a law in this regard as it is the exclusive jurisdiction under the Union List. b) Only States are empowered to pass legislation in this regard. c) Both Centre and states are empowered to pass legislation in this regard as it is the common subject. d) The Parliament shall pass a law in this regard because it is a residuary subject. Correct Solution (d) A subject not specifically mentioned in any of the three lists comes under the Residuary Powers of the Union under entry 97 of the Union List. Only the government of the Union trough Parliament has the powers to render law on Residuary Subjects. These subjects are e-commerce, Computer software and so on. After the constitution was established, these subjects came into use which doesn’t come in any list. Incorrect Solution (d) A subject not specifically mentioned in any of the three lists comes under the Residuary Powers of the Union under entry 97 of the Union List. Only the government of the Union trough Parliament has the powers to render law on Residuary Subjects. These subjects are e-commerce, Computer software and so on. After the constitution was established, these subjects came into use which doesn’t come in any list. Question 12 of 35 12. Question Under Article 371-J, the President is empowered to provide that the Governor of Karnataka would have special responsibility for- The establishment of a separate development board for the Hyderabad-Karnataka region. Making a provision that a report on the working of the board would be placed every year before the State Legislative Assembly. The equitable allocation of funds for developmental expenditure over the region. How many of the above statements are correct? a) Only one b) Only two c) All three d) None Correct Solution (c) Statement 1 Statement 2 Statement 3 Correct Correct Correct The establishment of a separate development board for the Hyderabad-Karnataka region. Making a provision that a report on the working of the board would be placed every year before the State Legislative Assembly. The equitable allocation of funds for developmental expenditure over the region.   Note: Under Article 371-J, the President is empowered to provide that the Governor of Karnataka would have special responsibility for- The establishment of a separate development board for Hyderabad-Karnataka region Making a provision that a report on the working of the board would be placed every year before the State Legislative Assembly The equitable allocation of funds for developmental expenditure over the region The reservation of seats in educational and vocational training institutions in the region for students who belong to the region 5. The reservation in state government posts in the region for persons who belong to the region. Incorrect Solution (c) Statement 1 Statement 2 Statement 3 Correct Correct Correct The establishment of a separate development board for the Hyderabad-Karnataka region. Making a provision that a report on the working of the board would be placed every year before the State Legislative Assembly. The equitable allocation of funds for developmental expenditure over the region.   Note: Under Article 371-J, the President is empowered to provide that the Governor of Karnataka would have special responsibility for- The establishment of a separate development board for Hyderabad-Karnataka region Making a provision that a report on the working of the board would be placed every year before the State Legislative Assembly The equitable allocation of funds for developmental expenditure over the region The reservation of seats in educational and vocational training institutions in the region for students who belong to the region 5. The reservation in state government posts in the region for persons who belong to the region. Question 13 of 35 13. Question The Jammu and Kashmir Reorganisation Act, 2019 reorganizes the state of Jammu and Kashmir into: The Union Territory of Jammu and Kashmir without a legislature. The Union Territory of Ladakh with a legislature. Which of the statements given above is/are correct? a) 1 only b) 2 only c) Both 1 and 2 d) Neither 1 nor 2 Correct Solution (d) Statement 1 Statement 2 Incorrect Incorrect The Jammu and Kashmir Reorganisation Act, 2019 reorganizes the state of Jammu and Kashmir into- The Union Territory of Jammu and Kashmir with a legislature The Jammu and Kashmir Reorganisation Act, 2019 reorganizes the state of Jammu and Kashmir into- The Union Territory of Ladakh without a legislature. Note: The Jammu and Kashmir Reorganisation Act, 2019 reorganizes the state of Jammu and Kashmir into- The Union Territory of Jammu and Kashmir with a legislature, and The Union Territory of Ladakh without a legislature. The Union Territory of Ladakh will comprise Kargil and Leh districts. The Union Territory of Jammu and Kashmir will comprise the remaining territories of the existing state of Jammu and Kashmir. Incorrect Solution (d) Statement 1 Statement 2 Incorrect Incorrect The Jammu and Kashmir Reorganisation Act, 2019 reorganizes the state of Jammu and Kashmir into- The Union Territory of Jammu and Kashmir with a legislature The Jammu and Kashmir Reorganisation Act, 2019 reorganizes the state of Jammu and Kashmir into- The Union Territory of Ladakh without a legislature. Note: The Jammu and Kashmir Reorganisation Act, 2019 reorganizes the state of Jammu and Kashmir into- The Union Territory of Jammu and Kashmir with a legislature, and The Union Territory of Ladakh without a legislature. The Union Territory of Ladakh will comprise Kargil and Leh districts. The Union Territory of Jammu and Kashmir will comprise the remaining territories of the existing state of Jammu and Kashmir. Question 14 of 35 14. Question Consider the following statements regarding special provisions for Nagaland: The Governor of Nagaland shall have special responsibility for law and order in the state so long as internal disturbances caused by the hostile Nagas continue. A regional council consisting of 35 members should be established for the Tuensang district of the state. Which of the statements given above is/are correct? a) 1 only b) 2 only c) Both 1 and 2 d) Neither 1 nor 2 Correct Solution (c) Statement 1 Statement 2 Correct Correct The Governor of Nagaland shall have special responsibility for law and order in the state so long as internal disturbances caused by the hostile Nagas continue. In the discharge of this responsibility, the Governor, after consulting the Council of Ministers, exercises his individual judgement and his decision is final. This special responsibility of the Governor shall cease when the President so directs. A regional council consisting of 35 members should be established for the Tuensang district of the state. PROVISIONS FOR NAGALAND: Article 371-A makes the following special provisions for Nagaland: The Acts of Parliament relating to the following matters would not apply to Nagaland unless the State Legislative Assembly so decides: religious or social practices of the Nagas; Naga customary law and procedure; administration of civil and criminal justice involving decision according to Naga customary law; and ownership and transfer of land and its resources. The Governor of Nagaland shall have special responsibility for law and order in the state so long as internal disturbances caused by the hostile Nagas continue. In the discharge of this responsibility, the Governor, after consulting the Council of Ministers, exercises his individual judgement and his decision is final. This special responsibility of the Governor shall cease when the President so directs. The Governor has to ensure that the money provided by the Central Government for any specific purpose is included in the demand for a grant relating to that purpose and not in any other demand moved in the State Legislative Assembly. A regional council consisting of 35 members should be established for the Tuensang district of the state. The Governor should make rules for the composition of the council, manner of choosing its members, their qualifications, term, salaries and allowances; the procedure and conduct of business of the council; the appointment of officers and staff of the council and their service conditions; and any other matter relating to the constitution and proper functioning of the council. Incorrect Solution (c) Statement 1 Statement 2 Correct Correct The Governor of Nagaland shall have special responsibility for law and order in the state so long as internal disturbances caused by the hostile Nagas continue. In the discharge of this responsibility, the Governor, after consulting the Council of Ministers, exercises his individual judgement and his decision is final. This special responsibility of the Governor shall cease when the President so directs. A regional council consisting of 35 members should be established for the Tuensang district of the state. PROVISIONS FOR NAGALAND: Article 371-A makes the following special provisions for Nagaland: The Acts of Parliament relating to the following matters would not apply to Nagaland unless the State Legislative Assembly so decides: religious or social practices of the Nagas; Naga customary law and procedure; administration of civil and criminal justice involving decision according to Naga customary law; and ownership and transfer of land and its resources. The Governor of Nagaland shall have special responsibility for law and order in the state so long as internal disturbances caused by the hostile Nagas continue. In the discharge of this responsibility, the Governor, after consulting the Council of Ministers, exercises his individual judgement and his decision is final. This special responsibility of the Governor shall cease when the President so directs. The Governor has to ensure that the money provided by the Central Government for any specific purpose is included in the demand for a grant relating to that purpose and not in any other demand moved in the State Legislative Assembly. A regional council consisting of 35 members should be established for the Tuensang district of the state. The Governor should make rules for the composition of the council, manner of choosing its members, their qualifications, term, salaries and allowances; the procedure and conduct of business of the council; the appointment of officers and staff of the council and their service conditions; and any other matter relating to the constitution and proper functioning of the council. Question 15 of 35 15. Question Consider the following pairs with respect to the financial relations between Centre and State: Article Distribution of Tax Revenue 1.     Article 268 Taxes levied by the Centre but collected and appropriated by theStates 2.     Article 269 Taxes levied and collected by the Centre but distributed between the Centre and the States. 3.     Article 270 Taxes levied and collected by the Centre but assigned to the States. How many of the above pairs is/are correctly matched? a) None of the pairs b) One pair only c) Two pairs only d) Three pairs only Correct Solution (b) Statement 1 Statement 2 Statement 3 Correct Incorrect Incorrect Article 268- Taxes levied by the Centre but collected and appropriated by the States Article 269- Taxes levied and collected by the Centre but assigned to the States. Article 270- Taxes levied and collected by the Centre but distributed between the Centre and the States. Note: The 80th Amendment Act of 2000 and the 88th Amendment Act of 2003 changed the way tax revenues were distributed between the federal government and the states. Article 268: The Centre imposes taxes, while the states are in charge of collecting them. It contains a variety of duties and tax revenues, including: Stamp duty is charged on bills of exchange, promissory notes, insurance policies, checks, stock transfers, and other documents The collected duties levied by any state (inside the state) are given to the state rather than to the Consolidated Fund of India The centre imposes a service tax, but the states collect and appropriate it (Article 268-A) (now outlawed amid GST) Article 269: Taxes levied and collected by the federal government but distributed to state. This category includes the following taxes: Various tariffs were levied on the sale or purchase of commodities (other than newspapers) in the course of interstate commerce or trade Various tariffs on products sent in the course of interstate trade or commerce All of these taxes’ net proceeds do not go into the Consolidated Fund of India (CFI). According to the principles established by the Parliament, they are assigned to the involved states Article 269- A: Imposition and collection of Goods and Services Tax in line with interstate trade or commerce. The Centre imposes and collects the Goods and Services Tax (GST) on supplies made in the course of interstate trade or commerce However, this tax is split between the Centre and the States in the manner proposed by Parliament based on the GST Council’s recommendations Furthermore, the Parliament has the authority to develop standards for establishing the site of supply and when commodities or services, or both, are supplied in the course of interstate trade or commerce Article 270: Taxes imposed and collected by the Centre but distributed amongst the Centre and the States proportionately. This category comprises all taxes and duties referred to in the Union List except the following: Articles 268, 269, and 269-A deal with duties and taxes (mentioned above). Article 271 imposes a surcharge on taxes and duties (mentioned below). Any tax imposed for a specified purpose. The President, on the recommendation of the Finance Commission, prescribes the method for distributing the net earnings of all these taxes and duties (FCs). Article 271-Surcharges on certain taxes and duties for purposes of the centre. Articles 269 and 270 of the Constitution provide that the Parliament may impose surcharges on taxes and duties at any time (mentioned above). The Centre receives all of the profits from such surcharges. In other words, the states aren’t paying any of the levies. This fee is not applicable to the Goods and Services Tax (GST). To put it another way, the GST will not be subject to this surcharge. State Government Taxes: Taxes of this nature are entirely the responsibility of the governments. They are 18 in number and are included on the State List. Incorrect Solution (b) Statement 1 Statement 2 Statement 3 Correct Incorrect Incorrect Article 268- Taxes levied by the Centre but collected and appropriated by the States Article 269- Taxes levied and collected by the Centre but assigned to the States. Article 270- Taxes levied and collected by the Centre but distributed between the Centre and the States. Note: The 80th Amendment Act of 2000 and the 88th Amendment Act of 2003 changed the way tax revenues were distributed between the federal government and the states. Article 268: The Centre imposes taxes, while the states are in charge of collecting them. It contains a variety of duties and tax revenues, including: Stamp duty is charged on bills of exchange, promissory notes, insurance policies, checks, stock transfers, and other documents The collected duties levied by any state (inside the state) are given to the state rather than to the Consolidated Fund of India The centre imposes a service tax, but the states collect and appropriate it (Article 268-A) (now outlawed amid GST) Article 269: Taxes levied and collected by the federal government but distributed to state. This category includes the following taxes: Various tariffs were levied on the sale or purchase of commodities (other than newspapers) in the course of interstate commerce or trade Various tariffs on products sent in the course of interstate trade or commerce All of these taxes’ net proceeds do not go into the Consolidated Fund of India (CFI). According to the principles established by the Parliament, they are assigned to the involved states Article 269- A: Imposition and collection of Goods and Services Tax in line with interstate trade or commerce. The Centre imposes and collects the Goods and Services Tax (GST) on supplies made in the course of interstate trade or commerce However, this tax is split between the Centre and the States in the manner proposed by Parliament based on the GST Council’s recommendations Furthermore, the Parliament has the authority to develop standards for establishing the site of supply and when commodities or services, or both, are supplied in the course of interstate trade or commerce Article 270: Taxes imposed and collected by the Centre but distributed amongst the Centre and the States proportionately. This category comprises all taxes and duties referred to in the Union List except the following: Articles 268, 269, and 269-A deal with duties and taxes (mentioned above). Article 271 imposes a surcharge on taxes and duties (mentioned below). Any tax imposed for a specified purpose. The President, on the recommendation of the Finance Commission, prescribes the method for distributing the net earnings of all these taxes and duties (FCs). Article 271-Surcharges on certain taxes and duties for purposes of the centre. Articles 269 and 270 of the Constitution provide that the Parliament may impose surcharges on taxes and duties at any time (mentioned above). The Centre receives all of the profits from such surcharges. In other words, the states aren’t paying any of the levies. This fee is not applicable to the Goods and Services Tax (GST). To put it another way, the GST will not be subject to this surcharge. State Government Taxes: Taxes of this nature are entirely the responsibility of the governments. They are 18 in number and are included on the State List. Question 16 of 35 16. Question Consider the following statements: Article 371-E of the Constitution of India contains special provisions with respect to Sikkim. The 38th Constitutional Amendment Act made Sikkim a full-fledged state of the Indian Union. Which of the statements given above is/are correct? a) 1 only b) 2 only c) Both 1 and 2 d) Neither 1 nor 2 Correct Solution (d) Statement 1 Statement 2 Incorrect Incorrect Article 371-E empowers the Parliament to provide for the establishment of a Central University in the state of Andhra Pradesh. Article 371-F of the Constitution of India contains special provisions with respect to Sikkim. The 36th Constitutional Amendment Act made Sikkim a full-fledged state of the Indian Union. Articles 371 to 371-J in Part XXI of the Constitution of India contain special provisions for twelve states: Maharashtra Andhra Pradesh Telangana Sikkim Mizoram Arunachal Pradesh Gujarat Nagaland Assam Manipur Goa Karnataka Incorrect Solution (d) Statement 1 Statement 2 Incorrect Incorrect Article 371-E empowers the Parliament to provide for the establishment of a Central University in the state of Andhra Pradesh. Article 371-F of the Constitution of India contains special provisions with respect to Sikkim. The 36th Constitutional Amendment Act made Sikkim a full-fledged state of the Indian Union. Articles 371 to 371-J in Part XXI of the Constitution of India contain special provisions for twelve states: Maharashtra Andhra Pradesh Telangana Sikkim Mizoram Arunachal Pradesh Gujarat Nagaland Assam Manipur Goa Karnataka Question 17 of 35 17. Question Consider the following statements: The Governor of Arunachal Pradesh shall have special responsibility for law and order in the state. The Arunachal Pradesh Legislative Assembly is to consist of not less than 50 members. Which of the statements given above is/are correct? a) 1 only b) 2 only c) Both 1 and 2 d) Neither 1 nor 2 Correct Solution (a) Statement 1 Statement 2 Correct Incorrect The Governor of Arunachal Pradesh shall have special responsibility for law and order in the state. In the discharge of this responsibility, the Governor, after consulting the Council of Ministers, exercises his individual judgement and his decision is final. This special responsibility of the Governor shall cease when the President directs. The Legislative Assembly of Arunachal Pradesh should consist of not less than 30 members. Incorrect Solution (a) Statement 1 Statement 2 Correct Incorrect The Governor of Arunachal Pradesh shall have special responsibility for law and order in the state. In the discharge of this responsibility, the Governor, after consulting the Council of Ministers, exercises his individual judgement and his decision is final. This special responsibility of the Governor shall cease when the President directs. The Legislative Assembly of Arunachal Pradesh should consist of not less than 30 members. Question 18 of 35 18. Question Under Article 371, the President is authorized to provide that the Governor of Maharashtra would have special responsibility for: The establishment of separate development boards for Vidarbha, Marathwada and the rest of Maharashtra. Making a provision that a report on the working of these boards would be placed every 5 years before the State Legislative Assembly. Which of the statements given above is/are correct? a) 1 only b) 2 only c) Both 1 and 2 d) Neither 1 nor 2 Correct Solution (a) Statement 1 Statement 2 Correct Incorrect The establishment of separate development boards for (i) Vidarbha, Marathwada and the rest of Maharashtra, (ii) Saurashtra, Kutch and the rest of Gujarat. Making a provision that a report on the working of these boards would be placed every year before the State Legislative Assembly. Note: Under Article 371, the President is authorized to provide that the Governor of Maharashtra and that of Gujarat would have special responsibility for- The establishment of separate development boards for (i) Vidarbha, Marathwada and the rest of Maharashtra, (ii) Saurashtra, Kutch and the rest of Gujarat; Making a provision that a report on the working of these boards would be placed every year before the State Legislative Assembly; The equitable allocation of funds for developmental expenditure over the above-mentioned areas, and An equitable arrangement providing adequate facilities for technical education and vocational training and adequate employment opportunities in the state services in respect of the above-mentioned areas. Incorrect Solution (a) Statement 1 Statement 2 Correct Incorrect The establishment of separate development boards for (i) Vidarbha, Marathwada and the rest of Maharashtra, (ii) Saurashtra, Kutch and the rest of Gujarat. Making a provision that a report on the working of these boards would be placed every year before the State Legislative Assembly. Note: Under Article 371, the President is authorized to provide that the Governor of Maharashtra and that of Gujarat would have special responsibility for- The establishment of separate development boards for (i) Vidarbha, Marathwada and the rest of Maharashtra, (ii) Saurashtra, Kutch and the rest of Gujarat; Making a provision that a report on the working of these boards would be placed every year before the State Legislative Assembly; The equitable allocation of funds for developmental expenditure over the above-mentioned areas, and An equitable arrangement providing adequate facilities for technical education and vocational training and adequate employment opportunities in the state services in respect of the above-mentioned areas. Question 19 of 35 19. Question With respect to the National Integration Council (NIC), consider the following statements: The council of national integration primarily sets its objective not to stand with any internal conflict against religious freedom. It helps to implement such policies and principles to maintain the harmony for which the country will stand together. It emphasizes the privileges of common citizenship. How many of the statements given above is/are correct? a) Only one b) Only two c) All three d) None Correct Solution (c) Statement 1 Statement 2 Statement 3 Correct Correct Correct The council of national integration primarily sets its objective not to stand with any internal conflict against religious freedom, any injustice in the political, economic, and social things. NIC helps to implement such policies and principles to maintain the harmony for which the country will stand together. National integration also helps to formulate appropriate community activities and programs to bring peer-to-peer sentiment. It also emphasizes the privileges of common citizenship. Incorrect Solution (c) Statement 1 Statement 2 Statement 3 Correct Correct Correct The council of national integration primarily sets its objective not to stand with any internal conflict against religious freedom, any injustice in the political, economic, and social things. NIC helps to implement such policies and principles to maintain the harmony for which the country will stand together. National integration also helps to formulate appropriate community activities and programs to bring peer-to-peer sentiment. It also emphasizes the privileges of common citizenship. Question 20 of 35 20. Question Consider the following: Scrutiny by departmental committees General discussion Passing of appropriation bill Passing of finance bill Voting on demands for grants Which one of the following is the correct sequence of stages in enactment of budget in the Parliament? a) 1-2-3-5-4 b) 2-1-5-3-4 c) 2-5-1-4-3 d) 1-2-5-4-3 Correct Solution (b) The budget goes through the following six stages in the Parliament: Presentation of budget. General discussion. Scrutiny by departmental committees. Voting on demands for grants. Passing of appropriation bill. Passing of finance bill. Budget: The finance minister presents the budget in the Lok Sabha. He makes his budget in the Lok Sabha. Simultaneously, the copy of the budget is laid on the table of the Rajya Sabha. Printed copies of the budget are distributed among the members of the parliament to go through the details of the budgetary provisions. The finance bill is presented to the parliament immediately after the presentation of the budget. Finance Bill relates to the proposals regarding the imposition of new taxes, modification on the existing taxes or the abolition of the old taxes. The proposals on revenue and expenditure are discussed in the Parliament. Members of the Parliament actively take part in the discussion. After the general discussion on the budget is over, the Houses are adjourned for about three to four weeks. During this gap period, the 24 departmental standing committees of Parliament examine and discuss in detail the demands for grants of the concerned ministers and prepare reports on them. These reports are submitted to both the Houses of Parliament for consideration. Demands for grants are presented to the Parliament along with the budget. These demands for grants show that the estimates of the expenditure for various departments and they need to be voted by the Parliament. No demand for grants shall be made except on the recommendation of the President. After the demands for grants are voted by the parliament, the Appropriation Bill is introduced, considered and passed by the appropriation of the Parliament. It provides the legal authority for withdrawal of funds of what is known as the Consolidated Fund of India. After the passing of the appropriation bill, finance bill is discussed and passed. At this stage, the members of the parliament can suggest and make some amendments which the finance minister can approve or reject. Appropriation bill and Finance bill are sent to Rajya Sabha. The Rajya Sabha is required to send back these bills to the Lok Sabha within fourteen days with or without amendments. However, Lok Sabha may or may not accept the bill. Finance Bill is sent to the President for his assent. The bill becomes the statue after President’s sign. The president does not have the power to reject the bill. Incorrect Solution (b) The budget goes through the following six stages in the Parliament: Presentation of budget. General discussion. Scrutiny by departmental committees. Voting on demands for grants. Passing of appropriation bill. Passing of finance bill. Budget: The finance minister presents the budget in the Lok Sabha. He makes his budget in the Lok Sabha. Simultaneously, the copy of the budget is laid on the table of the Rajya Sabha. Printed copies of the budget are distributed among the members of the parliament to go through the details of the budgetary provisions. The finance bill is presented to the parliament immediately after the presentation of the budget. Finance Bill relates to the proposals regarding the imposition of new taxes, modification on the existing taxes or the abolition of the old taxes. The proposals on revenue and expenditure are discussed in the Parliament. Members of the Parliament actively take part in the discussion. After the general discussion on the budget is over, the Houses are adjourned for about three to four weeks. During this gap period, the 24 departmental standing committees of Parliament examine and discuss in detail the demands for grants of the concerned ministers and prepare reports on them. These reports are submitted to both the Houses of Parliament for consideration. Demands for grants are presented to the Parliament along with the budget. These demands for grants show that the estimates of the expenditure for various departments and they need to be voted by the Parliament. No demand for grants shall be made except on the recommendation of the President. After the demands for grants are voted by the parliament, the Appropriation Bill is introduced, considered and passed by the appropriation of the Parliament. It provides the legal authority for withdrawal of funds of what is known as the Consolidated Fund of India. After the passing of the appropriation bill, finance bill is discussed and passed. At this stage, the members of the parliament can suggest and make some amendments which the finance minister can approve or reject. Appropriation bill and Finance bill are sent to Rajya Sabha. The Rajya Sabha is required to send back these bills to the Lok Sabha within fourteen days with or without amendments. However, Lok Sabha may or may not accept the bill. Finance Bill is sent to the President for his assent. The bill becomes the statue after President’s sign. The president does not have the power to reject the bill. Question 21 of 35 21. Question Which one of the following pairs of islands is separated from each other by the ‘Eight Degree Channel’? a) Sumatra and Java b) Andaman and Nicobar c) Nicobar and Sumatra d) Maldives and Minicoy Correct Solution (d) The Eight Degree Channel separates the islands of Minicoy and Maldives. Hence option d is correct. The Nine Degree Channel separates the island of Minicoy from the main Lakshadweep archipelago. The Ten Degree Channel separates the Andaman Islands and the Nicobar Islands from each other in the Bay of Bengal. Note: Maldives is a low-lying island country in the north-central Indian Ocean. Its capital is Male and its official language is an Indo-European language called Dhivehi. Its state religion is Islam and its economy mainly revolves around tourism. Incorrect Solution (d) The Eight Degree Channel separates the islands of Minicoy and Maldives. Hence option d is correct. The Nine Degree Channel separates the island of Minicoy from the main Lakshadweep archipelago. The Ten Degree Channel separates the Andaman Islands and the Nicobar Islands from each other in the Bay of Bengal. Note: Maldives is a low-lying island country in the north-central Indian Ocean. Its capital is Male and its official language is an Indo-European language called Dhivehi. Its state religion is Islam and its economy mainly revolves around tourism. Question 22 of 35 22. Question Consider the following statements regarding the ‘World Heritage Committee’:  It defines the use of the World Heritage Fund and allocates financial assistance upon requests from state parties. It has the final say on whether a property is inscribed on the World Heritage List. The Committee consists of 21 members who hold office for a term of one year. How many of the above statements are correct? a) Only one b) Only two c) All three d) None Correct Solution (b) World Heritage Committee is a committee of the United Nations Educational, Scientific, and Cultural Organization. It is responsible for the implementation of the World Heritage Convention, defines the use of the World Heritage Fund, and allocates financial assistance upon requests from state parties. Hence statement 1 is correct. It has the final say on whether a property is inscribed on the World Heritage List. Hence statement 2 is correct. It examines reports on the state of conservation of inscribed properties and asks States Parties to take action when properties are not being properly managed. It also decides on the inscription or deletion of properties on the List of World Heritage in Danger. The Committee consists of 21 members from the state parties to the Convention elected by their General Assembly and they hold office for a term of six years. However, most state parties choose voluntarily to be members of the committee for only four years to allow other state parties to be on the committee. Hence statement 3 is incorrect. Note: The United Nations Educational, Scientific, and Cultural Organization (UNESCO) is a specialized agency of the United Nations. Its mission is to contribute to peace and security by promoting international collaboration through education, science, and culture. It provides a platform for international cooperation and establishes a holistic cultural governance system based on human rights and shared values.   Incorrect Solution (b) World Heritage Committee is a committee of the United Nations Educational, Scientific, and Cultural Organization. It is responsible for the implementation of the World Heritage Convention, defines the use of the World Heritage Fund, and allocates financial assistance upon requests from state parties. Hence statement 1 is correct. It has the final say on whether a property is inscribed on the World Heritage List. Hence statement 2 is correct. It examines reports on the state of conservation of inscribed properties and asks States Parties to take action when properties are not being properly managed. It also decides on the inscription or deletion of properties on the List of World Heritage in Danger. The Committee consists of 21 members from the state parties to the Convention elected by their General Assembly and they hold office for a term of six years. However, most state parties choose voluntarily to be members of the committee for only four years to allow other state parties to be on the committee. Hence statement 3 is incorrect. Note: The United Nations Educational, Scientific, and Cultural Organization (UNESCO) is a specialized agency of the United Nations. Its mission is to contribute to peace and security by promoting international collaboration through education, science, and culture. It provides a platform for international cooperation and establishes a holistic cultural governance system based on human rights and shared values.   Question 23 of 35 23. Question Consider the following statements about the ‘Pallas’s Sea Eagle’: It is found only in India, Nepal, Bangladesh and Myanmar. It is listed as critically endangered on the IUCN Red List. The Pallas’s Sea Eagle also known as band-tailed fish eagle. How many of the above statements are correct? a) Only one b) Only two c) All three d) None Correct Solution (a) The Pallas’s Sea Eagle also known as band-tailed fish eagle, is a large, brownish sea eagle. Hence statement 3 is correct. It is found in the east Palearctic in Kazakhstan, Russia, Tajikistan, Turkmenistan, Uzbekistan, Mongolia, China, India, Nepal, Bangladesh and Myanmar. Hence statement 1 is incorrect. It can be seen near lakes, marshes, and large rivers, from lowlands to 5,000 metres of elevation. It feeds primarily on fish, but many other preys are part of its diet. It breeds usually near water in a large nest placed in a tall tree. It is listed as endangered on the IUCN Red List. Hence statement 2 is incorrect. Chilka Lake is a brackish water lake and a shallow lagoon with estuarine character spread across the districts of Puri, Khurda, and Ganjam in the state of Odisha. Chilka Lake is located at the mouth of the Daya River, flowing into the Bay of Bengal. It is connected to the Bay of Bengal by a wide channel that mostly runs parallel to the Bay separated by a narrow spit. Incorrect Solution (a) The Pallas’s Sea Eagle also known as band-tailed fish eagle, is a large, brownish sea eagle. Hence statement 3 is correct. It is found in the east Palearctic in Kazakhstan, Russia, Tajikistan, Turkmenistan, Uzbekistan, Mongolia, China, India, Nepal, Bangladesh and Myanmar. Hence statement 1 is incorrect. It can be seen near lakes, marshes, and large rivers, from lowlands to 5,000 metres of elevation. It feeds primarily on fish, but many other preys are part of its diet. It breeds usually near water in a large nest placed in a tall tree. It is listed as endangered on the IUCN Red List. Hence statement 2 is incorrect. Chilka Lake is a brackish water lake and a shallow lagoon with estuarine character spread across the districts of Puri, Khurda, and Ganjam in the state of Odisha. Chilka Lake is located at the mouth of the Daya River, flowing into the Bay of Bengal. It is connected to the Bay of Bengal by a wide channel that mostly runs parallel to the Bay separated by a narrow spit. Question 24 of 35 24. Question Consider the following statements about the National Real Estate Development Council (NAREDCO): It is the leading industry association for the real estate sector in the country. It works under the Ministry of Housing and Urban Affairs of the Government of India. Its organizational structure includes National, State, and City Councils. How many of the above statements are correct? a) Only one b) Only two c) All three d) None Correct Solution (c) The National Real Estate Development Council (NAREDCO) is the leading industry association for the real estate sector in the country. Hence statement 1 is correct. Its primary objective is to provide a legitimate platform for the government, the real estate industry, and the general public to address their concerns and find effective solutions to the challenges faced by the real estate sector. Its mission is to improve the real estate industry’s building, construction, and marketing standards. It works under the Ministry of Housing and Urban Affairs of the Government of India. Hence statement 2 is correct. It plays a crucial role in policy formulation by representing the viewpoints of its members to various ministries regularly. All major national developers and public sector organizations in the fields of housing and real estate development, finance, and marketing are members of NAREDCO. Its organizational structure includes National, State, and City Councils. Hence statement 3 is correct. The councils ensure that the policy recommendations accurately reflect the real conditions on the ground and cover the entire geography. The Union Minister for Housing and Urban Affairs, Govt. of India, serves as the Chief Patron of NAREDCO. Incorrect Solution (c) The National Real Estate Development Council (NAREDCO) is the leading industry association for the real estate sector in the country. Hence statement 1 is correct. Its primary objective is to provide a legitimate platform for the government, the real estate industry, and the general public to address their concerns and find effective solutions to the challenges faced by the real estate sector. Its mission is to improve the real estate industry’s building, construction, and marketing standards. It works under the Ministry of Housing and Urban Affairs of the Government of India. Hence statement 2 is correct. It plays a crucial role in policy formulation by representing the viewpoints of its members to various ministries regularly. All major national developers and public sector organizations in the fields of housing and real estate development, finance, and marketing are members of NAREDCO. Its organizational structure includes National, State, and City Councils. Hence statement 3 is correct. The councils ensure that the policy recommendations accurately reflect the real conditions on the ground and cover the entire geography. The Union Minister for Housing and Urban Affairs, Govt. of India, serves as the Chief Patron of NAREDCO. Question 25 of 35 25. Question Consider the following statements about Thanjavur Doll: It originated during the 12th Century under King Saraboji’s reign. They are handmade with paper mache, wax, plaster of Paris, and cement. They have been recognized as a Geographical Indication by the Government of India. How many of the above statements are correct? a) Only one b) Only two c) All three d) None Correct Solution (b) The Thanjavur Doll is a type of traditional Indian toy made in the city of Thanjavur, formerly Tanjore, in Tamil Nadu. Thanjavur Doll originated during the 18th Century under King Saraboji’s reign. Hence statement 1 is incorrect. The centre of gravity and the total weight of the doll is concentrated at its bottom-most point, generating a dance-like continuous movement with slow oscillations. They are handmade with paper mache, wax, plaster of Paris, coloring agents, clay, cardboard, plastics, and cement. Hence statement 2 is correct. They have been recognized as a Geographical Indication by the Government of India. Hence statement 3 is correct. Incorrect Solution (b) The Thanjavur Doll is a type of traditional Indian toy made in the city of Thanjavur, formerly Tanjore, in Tamil Nadu. Thanjavur Doll originated during the 18th Century under King Saraboji’s reign. Hence statement 1 is incorrect. The centre of gravity and the total weight of the doll is concentrated at its bottom-most point, generating a dance-like continuous movement with slow oscillations. They are handmade with paper mache, wax, plaster of Paris, coloring agents, clay, cardboard, plastics, and cement. Hence statement 2 is correct. They have been recognized as a Geographical Indication by the Government of India. Hence statement 3 is correct. Question 26 of 35 26. Question Consider the following statements about INS Chennai: It is a guided missile destroyer developed jointly by India and Russia. It is the last ship of the Kolkata-class stealth-guided missile destroyers under the Project 15A. It is equipped to fight under nuclear, biological, and chemical (NBC) warfare conditions. How many of the above statements are correct? a) Only one b) Only two c) All three d) None Correct Solution (b) INS Chennai is a guided missile destroyer indigenously designed and constructed by the Mazagon Dock Limited (MDL) in Mumbai. Hence statement 1 is incorrect. It was commissioned into the Indian Navy on November 21, 2016. It is the last ship of the Kolkata-class stealth-guided missile destroyers under the Project 15A. Hence statement 2 is correct. Project 15A is a class of stealth guided-missile destroyers constructed for the Indian Navy. It comprises three ships – Kolkata, Kochi,and Chennai. It is equipped to fight under nuclear, biological, and chemical (NBC) warfare conditions. Hence statement 3 is correct. It is fitted with a modern surveillance radar, which provides target data to the gunnery weapon systems of the ship. It is armed with vertical launch and long-range surface-to-air and surface-to-surface missile systems like supersonic BrahMos, and ‘Barak-8’ long-range surface-to-air missiles. Incorrect Solution (b) INS Chennai is a guided missile destroyer indigenously designed and constructed by the Mazagon Dock Limited (MDL) in Mumbai. Hence statement 1 is incorrect. It was commissioned into the Indian Navy on November 21, 2016. It is the last ship of the Kolkata-class stealth-guided missile destroyers under the Project 15A. Hence statement 2 is correct. Project 15A is a class of stealth guided-missile destroyers constructed for the Indian Navy. It comprises three ships – Kolkata, Kochi,and Chennai. It is equipped to fight under nuclear, biological, and chemical (NBC) warfare conditions. Hence statement 3 is correct. It is fitted with a modern surveillance radar, which provides target data to the gunnery weapon systems of the ship. It is armed with vertical launch and long-range surface-to-air and surface-to-surface missile systems like supersonic BrahMos, and ‘Barak-8’ long-range surface-to-air missiles. Question 27 of 35 27. Question Consider the following statements regarding the ‘Kadamba inscription’: Its epigraph is engraved in Kannada and Nagari characters of the 10th century AD. It is in theliterary style of the Talangre inscription of Jayasimha I of the same period. Choose the correct code: a) 1 only b) 2 only c) Both 1 and 2 d) Neither 1 nor 2 Correct Solution (c) The Kadamba inscription has been found in the Mahadeva temple at Cacoda in southern Goa. Its epigraph is engraved in Kannada and Nagari characters of the 10th century AD. Hence statement 1 is correct. The record is composed as a vocal statement on the death of his son from the mouth of a grieving father. It records that when Talara Nevayya was administering the mandala, his son Gundayya having taken a vow to fulfil his father’s desire of capturing a gopura of the port of Goa, fought and died after fulfilling his father’s wish. It is in the literary style of the Talangre inscription of Jayasimha I of the same period. Hence statement 2 is correct. Incorrect Solution (c) The Kadamba inscription has been found in the Mahadeva temple at Cacoda in southern Goa. Its epigraph is engraved in Kannada and Nagari characters of the 10th century AD. Hence statement 1 is correct. The record is composed as a vocal statement on the death of his son from the mouth of a grieving father. It records that when Talara Nevayya was administering the mandala, his son Gundayya having taken a vow to fulfil his father’s desire of capturing a gopura of the port of Goa, fought and died after fulfilling his father’s wish. It is in the literary style of the Talangre inscription of Jayasimha I of the same period. Hence statement 2 is correct. Question 28 of 35 28. Question Consider the following statements about the ‘Indian Army Day’: It is celebrated to commemorate the victory of the Indian Army over the Pakistan Army. The theme of Indian Army Day 2024 is “In Service of the Nation”. Choose the correct code: a) 1 only b) 2 only c) Both 1 and 2 d) Neither 1 nor 2 Correct Solution (b) Indian Army Day is celebrated every year on January 15. The Indian Army Day is celebrated to commemorate the victory of the Indian Army over the British Army. Hence statement 1 is incorrect. On January 15, 1949, Field Marshal K.M. Cariappa became the first Commander-in-Chief of the Indian Army, and took over authority from the British Commander-in-Chief General Francis Butcher, and became the first Indian to command the Army in its long, rich history. From 1949 until 2022, the Army Day parade was organised at the Cariappa Parade Ground in the Delhi Cantonment. In 2023, the Southern Command was responsible for the parade in Bengaluru. It was the first time the Army Day parade was held outside the country’s capital. This year, the parade will be held under the command of the Army’s ‘Central Command, which is headquartered in Lucknow. The theme of Indian Army Day 2024 is “In Service of the Nation”. This year’s theme also resembles the motto of the Indian Army, “Service Before Self.” Hence statement 2 is correct. Incorrect Solution (b) Indian Army Day is celebrated every year on January 15. The Indian Army Day is celebrated to commemorate the victory of the Indian Army over the British Army. Hence statement 1 is incorrect. On January 15, 1949, Field Marshal K.M. Cariappa became the first Commander-in-Chief of the Indian Army, and took over authority from the British Commander-in-Chief General Francis Butcher, and became the first Indian to command the Army in its long, rich history. From 1949 until 2022, the Army Day parade was organised at the Cariappa Parade Ground in the Delhi Cantonment. In 2023, the Southern Command was responsible for the parade in Bengaluru. It was the first time the Army Day parade was held outside the country’s capital. This year, the parade will be held under the command of the Army’s ‘Central Command, which is headquartered in Lucknow. The theme of Indian Army Day 2024 is “In Service of the Nation”. This year’s theme also resembles the motto of the Indian Army, “Service Before Self.” Hence statement 2 is correct. Question 29 of 35 29. Question Consider the following statements about ‘Kalaram Mandir’: It is the site of a landmark agitation led by Babasaheb Ambedkar demanding temple entry rights for Dalits. It is located on the banks of Krishna in the Panchavati area of Nashik in Maharashtra. Choose the correct code: a) 1 only b) 2 only c) Both 1 and 2 d) Neither 1 nor 2 Correct Solution (a) Kalaram Mandir is the site of a landmark agitation led by Babasaheb Ambedkar demanding temple entry rights for Dalits. Hence statement 1 is correct. In 1930, B R Ambedkar and the Marathi teacher and social activist Pandurang Sadashiv Sane, known as Sane Guruji, led an agitation to demand access for Dalits to Hindu temples. It derives its name from a black statue of the Lord — Kala Ram translates literally to “Black Ram”. It was built in 1792 with the efforts of Sardar Rangarao Odhekar. It is located on the banks of Godavari in the Panchavati area of Nashik in Maharashtra. Hence statement 2 is incorrect. Its sanctum sanctorum has statues of Ram, Sita, and Lakshman, and a black idol of Hanuman at the main entrance. The main temple has 14 steps, which represent the 14 years of Ram’s exile. It has 84 pillars, which represents the cycle of 84 lakh species that one has to complete to be born as a human. Incorrect Solution (a) Kalaram Mandir is the site of a landmark agitation led by Babasaheb Ambedkar demanding temple entry rights for Dalits. Hence statement 1 is correct. In 1930, B R Ambedkar and the Marathi teacher and social activist Pandurang Sadashiv Sane, known as Sane Guruji, led an agitation to demand access for Dalits to Hindu temples. It derives its name from a black statue of the Lord — Kala Ram translates literally to “Black Ram”. It was built in 1792 with the efforts of Sardar Rangarao Odhekar. It is located on the banks of Godavari in the Panchavati area of Nashik in Maharashtra. Hence statement 2 is incorrect. Its sanctum sanctorum has statues of Ram, Sita, and Lakshman, and a black idol of Hanuman at the main entrance. The main temple has 14 steps, which represent the 14 years of Ram’s exile. It has 84 pillars, which represents the cycle of 84 lakh species that one has to complete to be born as a human. Question 30 of 35 30. Question Consider the following statements about’ Punganur Cow’: It is native to the Indian state of Telangana. It has a high resilience to drought and can adapt to low-quality feed. Its milk is rich in nutrients such as Omega fatty acids, calcium, potassium, and magnesium. Its milk has cultural significance as it is used for Ksheeraabhishekam in Tirupati Thirumala Temple. How many of the above statements are correct? a) Only one b) Only two c) Only three d) All four Correct Solution (c) Punganur Cow is native to Punganur village in the Chittoor district of Andhra Pradesh. Hence statement 1 is incorrect. It has a high resilience to drought and can adapt to low-quality feed. They are considered eco-friendly, requiring less water, feed, and space compared to hybrid breeds. Hence statement 2 is correct. It is white, grey, or light brown to dark brown or red. Sometimes, animals with white colour mixed with red, brown, or black patches are also seen. Its milk is rich in nutrients such as Omega fatty acids, calcium, potassium, and magnesium. Hence statement 3 is correct. It is also prized for its milk, which boasts a higher fat content, making it ideal for producing ghee. A Punganur cow can give around 1 to 3 litres a day, and the milk fat content is 8 percent as compared to 3 to 4 percent in other native breeds. Its milk has cultural significance as it is used for Ksheeraabhishekam in Tirupati Thirumala Temple. Hence statement 4 is correct.   Incorrect Solution (c) Punganur Cow is native to Punganur village in the Chittoor district of Andhra Pradesh. Hence statement 1 is incorrect. It has a high resilience to drought and can adapt to low-quality feed. They are considered eco-friendly, requiring less water, feed, and space compared to hybrid breeds. Hence statement 2 is correct. It is white, grey, or light brown to dark brown or red. Sometimes, animals with white colour mixed with red, brown, or black patches are also seen. Its milk is rich in nutrients such as Omega fatty acids, calcium, potassium, and magnesium. Hence statement 3 is correct. It is also prized for its milk, which boasts a higher fat content, making it ideal for producing ghee. A Punganur cow can give around 1 to 3 litres a day, and the milk fat content is 8 percent as compared to 3 to 4 percent in other native breeds. Its milk has cultural significance as it is used for Ksheeraabhishekam in Tirupati Thirumala Temple. Hence statement 4 is correct.   Question 31 of 35 31. Question The number of ways of arranging n students in a row such that no two boys sit together and no two girls sit together is m(m>100). If one more student is added, then number of ways of arranging as above increases by 200%. The value of n is: a) 8 b) 12 c) 10 d) 9 Correct Solution (c) If n is even, then the number of boys should be equal to number of girls, let each be a. ⇒ n = 2a Then the number of arrangements = 2 × a! × a! If one more students is added, then number of arrangements, = a! × (a + 1)! But this is 200% more than the earlier ⇒ 3 × (2 × a! × a!) = a! × (a + 1)! ⇒ a + 1 = 6 and a = 5 ⇒ n = 10 But if n is odd, then number of arrangements, = a!(a + 1)! Where, n = 2a + 1 When one student is included, number of arrangements, = 2(a + 1)! (a + 1)! By the given condition, 2(a + 1) = 3, which is not possible   Incorrect Solution (c) If n is even, then the number of boys should be equal to number of girls, let each be a. ⇒ n = 2a Then the number of arrangements = 2 × a! × a! If one more students is added, then number of arrangements, = a! × (a + 1)! But this is 200% more than the earlier ⇒ 3 × (2 × a! × a!) = a! × (a + 1)! ⇒ a + 1 = 6 and a = 5 ⇒ n = 10 But if n is odd, then number of arrangements, = a!(a + 1)! Where, n = 2a + 1 When one student is included, number of arrangements, = 2(a + 1)! (a + 1)! By the given condition, 2(a + 1) = 3, which is not possible   Question 32 of 35 32. Question A gardener plants his garden with 5550 trees and arranged them so that there is one tree more per row as there are rows. Then the number of trees in a row are? a) 74 b) 56 c) 66 d) 75 Correct Solution (d) Let there be n rows, then number of trees in each row = (n + 1) Thus, total number of trees, n × (n +1) = 5550 Now, at this moment this problem can be solved in two ways. First by finding the roots of quadratic equation. Second, by using the values from options. 74 × 75 = 5550 i.e. (n + 1) = 75. Incorrect Solution (d) Let there be n rows, then number of trees in each row = (n + 1) Thus, total number of trees, n × (n +1) = 5550 Now, at this moment this problem can be solved in two ways. First by finding the roots of quadratic equation. Second, by using the values from options. 74 × 75 = 5550 i.e. (n + 1) = 75. Question 33 of 35 33. Question In a camp, there is a meal for 120 men or 200 children. If 150 children have taken the meal, how many men will be catered to with remaining meal? a) 30 b) 20 c) 50 d) 40 Correct Solution (a) There is a meal for 200 children 150 children have taken the meal Remaining meal is to be catered to 50 children Now, 200 children = 120 men ∴ 50 children = ((120/200*50)) men = 30 men Incorrect Solution (a) There is a meal for 200 children 150 children have taken the meal Remaining meal is to be catered to 50 children Now, 200 children = 120 men ∴ 50 children = ((120/200*50)) men = 30 men Question 34 of 35 34. Question Two friends A and B simultaneously start running around a circular track. They run in the same direction. A travels at 6 m/s and B runs at b m/s. If they cross each other at exactly two points on the circular track and b is a natural number less than 30, how many values can b take? a) 3 b) 4 c) 7 d) 5 Correct Solution (a) Let the total length of the track be equal to L . Relative speed of A with respect to B =b−6 Hence time when they meet for the first time =L/b−6 Time taken by A to complete one full lap of the track =L/6 Time taken by B to complete one full lap of the track =L/b Thus time when they meet at the starting point for the first time =L/HCF(b,6) Number of times they meet on starting point = time taken to meet at the starting point / time taken for meeting the first time =b−6/HCF(b,6) This is equal to 2 according to the question Therefore, b−6HCF(b,6)=2 Since less than 30, only values of b that satisfy the above equation are 2,10,18 . Hence there are 3 values that b can take. Incorrect Solution (a) Let the total length of the track be equal to L . Relative speed of A with respect to B =b−6 Hence time when they meet for the first time =L/b−6 Time taken by A to complete one full lap of the track =L/6 Time taken by B to complete one full lap of the track =L/b Thus time when they meet at the starting point for the first time =L/HCF(b,6) Number of times they meet on starting point = time taken to meet at the starting point / time taken for meeting the first time =b−6/HCF(b,6) This is equal to 2 according to the question Therefore, b−6HCF(b,6)=2 Since less than 30, only values of b that satisfy the above equation are 2,10,18 . Hence there are 3 values that b can take. Question 35 of 35 35. Question Abhishek divides some mangoes among four boys. He gives 1/4th to the first boy, 1/3rd of the remaining to the second boy, 1⁄2 of the remaining to the third boy and the remaining 20 to the fourth boy. How many mangoes he had in the beginning?   a) 120 b) 40 c) 160 d) 80 Correct Solution (d) Let the initial number of mangoes be ‘x’   Then, x – (1/4)x = 3x/4 3x/4 – 1/3 * 3x/ 4 = x/2   x/2 – 1/2 * x/2 = x/4 x/4 = 20 x = 80 Incorrect Solution (d) Let the initial number of mangoes be ‘x’   Then, x – (1/4)x = 3x/4 3x/4 – 1/3 * 3x/ 4 = x/2   x/2 – 1/2 * x/2 = x/4 x/4 = 20 x = 80 window.wpProQuizInitList = window.wpProQuizInitList || []; window.wpProQuizInitList.push({ id: '#wpProQuiz_3636', init: { quizId: 3636, mode: 1, globalPoints: 70, timelimit: 1800, resultsGrade: [0], bo: 704, qpp: 0, catPoints: [70], formPos: 0, lbn: "Test-summary", json: {"32244":{"type":"single","id":32244,"catId":0,"points":2,"correct":[0,1,0,0]},"32245":{"type":"single","id":32245,"catId":0,"points":2,"correct":[0,1,0,0]},"32247":{"type":"single","id":32247,"catId":0,"points":2,"correct":[0,0,0,1]},"32248":{"type":"single","id":32248,"catId":0,"points":2,"correct":[0,0,0,1]},"32249":{"type":"single","id":32249,"catId":0,"points":2,"correct":[1,0,0,0]},"32251":{"type":"single","id":32251,"catId":0,"points":2,"correct":[0,0,1,0]},"32252":{"type":"single","id":32252,"catId":0,"points":2,"correct":[1,0,0,0]},"32253":{"type":"single","id":32253,"catId":0,"points":2,"correct":[0,1,0,0]},"32255":{"type":"single","id":32255,"catId":0,"points":2,"correct":[1,0,0,0]},"32257":{"type":"single","id":32257,"catId":0,"points":2,"correct":[1,0,0,0]},"32258":{"type":"single","id":32258,"catId":0,"points":2,"correct":[0,0,0,1]},"32259":{"type":"single","id":32259,"catId":0,"points":2,"correct":[0,0,1,0]},"32262":{"type":"single","id":32262,"catId":0,"points":2,"correct":[0,0,0,1]},"32264":{"type":"single","id":32264,"catId":0,"points":2,"correct":[0,0,1,0]},"32265":{"type":"single","id":32265,"catId":0,"points":2,"correct":[0,1,0,0]},"32268":{"type":"single","id":32268,"catId":0,"points":2,"correct":[0,0,0,1]},"32269":{"type":"single","id":32269,"catId":0,"points":2,"correct":[1,0,0,0]},"32271":{"type":"single","id":32271,"catId":0,"points":2,"correct":[1,0,0,0]},"32272":{"type":"single","id":32272,"catId":0,"points":2,"correct":[0,0,1,0]},"32274":{"type":"single","id":32274,"catId":0,"points":2,"correct":[0,1,0,0]},"32277":{"type":"single","id":32277,"catId":0,"points":2,"correct":[0,0,0,1]},"32280":{"type":"single","id":32280,"catId":0,"points":2,"correct":[0,1,0,0]},"32282":{"type":"single","id":32282,"catId":0,"points":2,"correct":[1,0,0,0]},"32284":{"type":"single","id":32284,"catId":0,"points":2,"correct":[0,0,1,0]},"32286":{"type":"single","id":32286,"catId":0,"points":2,"correct":[0,1,0,0]},"32289":{"type":"single","id":32289,"catId":0,"points":2,"correct":[0,1,0,0]},"32292":{"type":"single","id":32292,"catId":0,"points":2,"correct":[0,0,1,0]},"32293":{"type":"single","id":32293,"catId":0,"points":2,"correct":[0,1,0,0]},"32296":{"type":"single","id":32296,"catId":0,"points":2,"correct":[1,0,0,0]},"32299":{"type":"single","id":32299,"catId":0,"points":2,"correct":[0,0,1,0]},"32300":{"type":"single","id":32300,"catId":0,"points":2,"correct":[0,0,1,0]},"32303":{"type":"single","id":32303,"catId":0,"points":2,"correct":[0,0,0,1]},"32306":{"type":"single","id":32306,"catId":0,"points":2,"correct":[1,0,0,0]},"32307":{"type":"single","id":32307,"catId":0,"points":2,"correct":[1,0,0,0]},"32309":{"type":"single","id":32309,"catId":0,"points":2,"correct":[0,0,0,1]}} } }); All the Best IASbaba

Daily Prelims CA Quiz

UPSC Quiz – 2024 : IASbaba’s Daily Current Affairs Quiz 13th April 2024

For Previous Daily Quiz (ARCHIVES) – CLICK HERE The Current Affairs questions are based on sources like ‘The Hindu’, ‘Indian Express’ and ‘PIB’, which are very important sources for UPSC Prelims Exam. The questions are focused on both the concepts and facts. The topics covered here are generally different from what is being covered under ‘Daily Current Affairs/Daily News Analysis (DNA) and Daily Static Quiz’ to avoid duplication. The questions would be published from Monday to Saturday before 2 PM. One should not spend more than 10 minutes on this initiative. Gear up and Make the Best Use of this initiative. Do remember that, “the difference between Ordinary and EXTRA-Ordinary is PRACTICE!!” Important Note: Don’t forget to post your marks in the comment section. Also, let us know if you enjoyed today’s test 🙂After completing the 5 questions, click on ‘View Questions’ to check your score, time taken, and solutions.To take the Test Click Here

DAILY CURRENT AFFAIRS IAS | UPSC Prelims and Mains Exam – 12th April 2024

Archives (PRELIMS & MAINS Focus)   Time Standard for Moon Syllabus Prelims – Science Context: The United States officially directed the National Aeronautics and Space Administration (NASA) to create a time standard for the Moon, which different international bodies and private companies can use to coordinate their activities on the lunar surface. Background:- According to Reuters, the head of the White House Office of Science and Technology Policy (OSTP) told the space agency to work with other parts of the US government to finalise the strategy by the end of 2026 for establishing what it called a Coordinated Lunar Time (LTC). Time Standard for Moon: Most of the clocks and time zones are based on coordinated Universal Time (UTC), which is set by the International Bureau of Weights and Measures in Paris, France. UTC is essentially an internationally agreed upon standard for world time. It is tracked by a weighted average of more than 400 atomic clocks placed in different parts of the globe. If a country lies on the west of the Greenwich meridian, it has to subtract from the UTC, and if a country is located on the east of the meridian, it has to add. UTC cannot be used to determine time on the Moon because time on the Moon flows differently than it does on the Earth. Why do we need a time standard for the Moon? Time flows differently on the Moon, and cannot be determined using UTC. Time is not absolute and it ticks slightly faster on the Moon as compared to the Earth. It is due to Einstein’s theory of general relativity which tells us that gravity bends space and time. On the Moon, the Earth-based clock will appear to lose on average 58.7 microseconds per Earth day with additional periodic variations. Improper time standards can create problems for spacecraft trying to dock on the Moon, transfer of data at a specific time, navigation, and communication. There may be a problem with multiple spacecraft working together at the same time. For example, NASA’s Artemis program aims to send humans back to the Moon after September 2026, and China and India intend to send their own astronauts to the lunar surface by 2030 and 2040 respectively. According to a 2023 report by the journal Nature, there will be a need to place at least three atomic clocks on the lunar surface that will tick at the Moon’s natural pace, and whose output will be combined by an algorithm to generate a more accurate virtual timepiece. These clocks have to be placed on the Moon at different locations since the Moon’s rotation and even local lumps of mass, called mascons, beneath the crust of the Moon affect the flow of time ever so slightly. These effects are minor but the output from these clocks can be synthesised to give the Moon its own independent time, which can be tied back to UTC for seamless operations from Earth as well. What are atomic clocks? Atomic clocks measure time in terms of the resonant frequencies — the natural frequency of an object where it tends to vibrate at a higher amplitude — of atoms such as cesium-133. In atomic time, a second is defined as the period in which a caesium atom vibrates 9,192,631,770 times. Source: Indian Express The Azad Hind government Syllabus Prelims & Mains – History Context:  Kangana Ranaut, in a recent interview, claimed that Subhas Chandra Bose, not Jawaharlal Nehru, was the first prime minister of independent India. After being criticised for the historicity (or lack thereof) of her comments, Kangana doubled down, citing the provisional government setup by Bose in 1943 as evidence of her claim. Background: Notably, 28 years before the Azad Hind government came into existence, the Provisional Government of India was formed in Kabul by a group known as the Indian Independence Committee (IIC). About the Azad Hind government: Subhas Chandra Bose proclaimed the formation of the Provisional Government of Azad Hind (“Free India”) in Singapore on October 21, 1943. Bose was the Head of State of this provisional government, and held the foreign affairs and war portfolios. A C Chatterjee was in charge of finance, S A Ayer became minister of publicity and propaganda, and Lakshmi Swaminathan was given the ministry of women’s affairs. A number of officers from Bose’s Azad Hind Fauj were also given cabinet posts. The Azad Hind government claimed authority over all Indian civilian and military personnel in Britain’s Southeast Asian colonies (primarily Burma, Singapore, and Malaya) which had fallen into Japanese hands during World War II. It also claimed prospective authority over all Indian territory that would be taken by Japanese forces, and Bose’s Azad Hind Fauj, as they attacked British India’s northeastern frontier. To give legitimacy to his government, much like Charles de Gaulle had declared sovereignty over some islands in the Atlantic for the Free French, Bose chose the Andamans. “It [the Azad Hind government] obtained de jure control over a piece of Indian territory when the Japanese handed over the Andaman and Nicobar islands in late December 1943, though de facto military control was not relinquished by the Japanese admiralty,” Sugata Bose wrote. Diplomatically, Bose’s government was recognised by the Axis powers and their satellites: Germany, Japan, and Italy, as well as Nazi and Japanese puppet states in Croatia, China, Thailand, Burma, Manchuria, and the Philippines. Immediately after its formation, the Azad Hind government declared war on Britain and the United States. Notably, 28 years before the Azad Hind government came into existence, the Provisional Government of India was formed in Kabul by a group known as the Indian Independence Committee (IIC). Much like Bose allied with the Axis powers during World War II to fight the British, during World War I, Indian nationalists abroad (mostly in Germany and the US), as well as revolutionaries and Pan-Islamists from India, attempted to further the cause of Indian independence with aid from the Central Powers. The IIC, with the help of the Ottoman Caliph and the Germans, tried to foment insurrection in India, mainly among Muslim tribes in Kashmir and the British India’s northwestern frontier. To further this cause, the IIC established a government-in-exile in Kabul under the presidency of Raja Mahendra Pratap, and prime ministership of Maulana Barkatullah. Source: Indian Express Food Waste Index Report 2024 Syllabus Mains – GS 3 Context: The Food Waste Index Report 2024 was released jointly by the United Nations Environment Programme (UNEP) and WRAP (Waste and Resources Action Programme), a UK-based non-profit organisation, stressed the importance of expanding and strengthening data infrastructure to enable the tracking and monitoring of food waste. Background: Contrary to popular perception, food waste was not a ‘rich country problem’, the report stated, with observed average levels of household food waste for high-income, upper-middle, and lower-middle-income countries differing by just 7 kg per capita. The Key Highlights of the Report: In 2022, the world wasted 1.05 billion tonnes of food, amounting to one-fifth (19%) of food available to consumers being wasted, at the retail, food service, and household level. That is in addition to the 13% of the world’s food lost in the supply chain, as estimated by FAO (Food and Agricultural Organization), from post-harvest up to and excluding retail. Food loss and waste generate 8-10% of Global Greenhouse Gas (GHG) emissions – almost five times the total emissions from the aviation sector. It occurs while a third of humanity faces food insecurity. Since the release of the 2021 Food Waste Index Report, there has been a significant expansion in data coverage, resulting in a notable reduction in disparities in average per capita household food waste. Across high-income, upper-middle-income, and lower-middle-income countries, the observed average levels of household food waste differ by just 7 kg per capita per year. Hotter countries appear to have more food waste per capita in households, potentially due to increased consumption of fresh foods with substantial inedible parts and lack of a robust cold chain. Higher seasonal temperatures, extreme heat events, and droughts make it more challenging to store, process, transport, and sell food safely, often leading to a significant volume of food being wasted or lost. Middle-income countries display variations between urban and rural populations, with rural areas generally wasting less. Possible explanations include greater diversion of food scraps to pets, animal feed, and home composting in rural areas. Many low- and middle-income countries continue to lack adequate systems for tracking progress to meet Sustainable Development Goal 12.3 of halving food waste by 2030, particularly in retail and food services. At present, only four G-20 countries (Australia, Japan, the UK, and the US) and the European Union have food waste estimates suitable for tracking progress to 2030. Countries like India, Indonesia, and South Korea have only subnational estimates regarding food waste, highlighting a gap in comprehensive national data. The report suggests that this variance necessitates more inclusive studies to capture a clearer picture of the food waste landscape. Key Recommendations of the Food Waste Index Report 2024: Encourage G20 countries to take a leading role in international cooperation and policy development to achieve Sustainable Development Goal (SDG) 12.3, leveraging their influence on global consumer trends to promote awareness and education about food waste domestically and internationally. Encourage the embrace of Public-Private Partnerships (PPPs) to reduce food waste and its impacts on climate and water stress, bringing together governments, and regional and industry groups to collaborate and deliver a shared goal through a Target-Measure-Act approach. Advocate for countries to use the Food Waste Index to consistently measure food waste, develop robust national baselines, and track progress towards SDG 12.3. This includes addressing the lack of comprehensive food waste data collection, especially in the retail and food service sectors. Highlight the necessity for representative national food waste studies in key countries such as India, China, South Africa, Indonesia, and Mexico to address the variance in data and effectively tackle food waste at both individual and systemic levels. There is a need to urge governments, cities, food businesses, and researchers to collaborate in efforts to reduce food waste, emphasising the importance of accurate measurement, innovative solutions, and collective action to achieve SDG 12.3 by halving global food waste by 2030. Source: UNEP NATIONAL INVESTMENT AND INFRASTRUCTURE FUND (NIIF) Syllabus Prelims – Economy Context: The National Investment and Infrastructure Fund (NIIF) recently made a significant investment of $200 million (approximately ₹1,660 crore) in iBUS Network and Infrastructure, a player in the digital infrastructure space. Background: This investment reflects NIIF’s commitment to supporting digital technology’s transformative power for India’s economy. It also aligns with the projected capital expenditure of $35-40 billion in the digital infrastructure sector over the next five years. About NATIONAL INVESTMENT AND INFRASTRUCTURE FUND (NIIF) The National Investment and Infrastructure Fund (NIIF) is India’s first sovereign wealth fund. NIIF is registered under SEBI Regulations as a Category II Alternative Investment Fund (AIF). Established in 2015, NIIF serves as a platform for both national and international investors to invest equity capital in domestic economic and social infrastructure. NIIF aims to enhance infrastructure financing by investing in greenfield (new), brownfield (existing), and stalled projects. NIIF is an investor-owned fund manager, anchored by the Government of India (GoI) in collaboration with leading global and domestic institutional investors. Types of NIIF Funds: Master Fund: This fund primarily invests in infrastructure-related projects such as roads, ports, airports, and power. It also invests in well-established enterprises operating in regulated environments. Fund of Funds: This fund invests in funds managed by renowned fund managers with excellent track records. It acts as an anchor investor, enabling fund managers to attract more institutional investments. Strategic Fund: Registered as an Alternative Fund II under SEBI, this fund primarily invests in equity and equity-linked instruments. Source: Hindu Previous Year Question Q1. With reference to ‘National Investment and Infrastructure Fund’, which of the following statements is/are correct? It is an organ of NITI Aayog. It has a corpus of Rs 4,00,000 crore at present. Select the correct answer using the code given below: 1 only 2 only Both 1 and 2 Neither 1 nor 2 GLOBAL POSITIONING SYSTEM (GPS) Syllabus Prelims – Science Context: Cargo ships transiting the Mediterranean and the Black Sea are encountering a growing number of incidents related to GPS jamming. Background: This trend has become more common since the Hamas attacks on Israel in October 2023. This situation is concerning, and some merchant ships have been additionally fitted with the GLONASS satellite system (Russian) as an emergency backup when onboard GPS systems (US) are jammed. About GLOBAL POSITIONING SYSTEM (GPS) : The Global Positioning System (GPS) is a satellite-based navigation system that provides precise location and time information to users worldwide. GPS is owned by the United States government and operated by the United States Space Force. GPS consists of a constellation of satellites orbiting Earth. These satellites transmit signals that allow GPS receivers on the ground to calculate their position accurately. GPS technology is widely used for navigation, mapping, surveying, and various other applications. Satellite-based navigation systems, also known as satnav systems, utilize satellites to provide autonomous geo-positioning. Global Systems: GPS (Global Positioning System): Operated by the United States. GLONASS (Global Navigation Satellite System): Russia’s navigation system. BeiDou (BDS): China’s navigation system. Galileo: Managed by the European Space Agency. Regional System: NavIC (Indian Regional Navigation Satellite System or IRNSS): India’s regional system. Source: Hindu Businessline NEW FEATURES OF UPI Syllabus Prelims – Economy Context: The Reserve Bank of India (RBI) has recently introduced two new features for the Unified Payments Interface (UPI). Background: These new features are poised to make UPI even more versatile and user-friendly, benefiting both consumers and financial institutions. About Unified Payments Interface (UPI): The Unified Payments Interface (UPI) is a real-time payment system that allows instant transfer of funds between two bank accounts through a mobile platform. It was developed by the National Payments Corporation of India (NPCI) and is regulated by the Reserve Bank of India (RBI). UPI was launched on April 11, 2016, as an advanced version of the Immediate Payment Service (IMPS). It integrates multiple bank accounts into a single mobile application, offering features like seamless fund routing and merchant payments. Some of the top UPI apps include PhonePe, Paytm, Google Pay, Amazon Pay, and BHIM, with BHIM being the government’s offering. New Features UPI Access for Prepaid Payment Instruments (PPIs) through Third Party Apps: Previously, UPI payments from Prepaid Payment Instruments (PPIs) could only be made using the web or mobile app provided by the PPI issuer. Now, RBI proposes to permit the use of third-party UPI apps for making UPI payments from PPI wallets. Enabling UPI for Cash Deposit Facility: Traditionally, depositing cash at Cash Deposit Machines (CDMs) primarily required the use of debit cards. Building on the success of card-less cash withdrawal using UPI at ATMs, RBI now proposes to facilitate cash deposits in CDMs using UPI. This measure aims to enhance customer convenience and streamline the currency handling process at banks. Soon, customers will be able to deposit cash at CDMs in banks and ATMs through the UPI app. Source: Times of India Practice MCQs Daily Practice MCQs Q1.) With reference to the Unified Payments Interface (UPI), consider the following statements: It allows instant transfer of funds between two bank accounts through a mobile platform. It was developed by the National Payments Corporation of India (NPCI). It is regulated by the Securities and Exchange Board of India (SEBI). How many of the statements given above are correct? Only one Only two All three None Q2.)Consider the following pairs: Satellite Navigation System         Country GPS –                                           India GLONASS –                               Russia BeiDou –                                     China How many of the pairs given above are correctly matched? Only one Only two Only three None Q3.)With reference to the National Investment and Infrastructure Fund (NIIF), consider the following statements: NIIF aims to enhance infrastructure financing by investing in greenfield, brownfield and stalled projects. It operates as an Alternative Investment Fund under SEBI regulations. Which of the statements given above are correct? 1 only 2 only Both 1 and 2 Neither 1 nor 2 Comment the answers to the above questions in the comment section below!! ANSWERS FOR ’  12th April  2024 – Daily Practice MCQs’ will be updated along with tomorrow’s Daily Current Affairs.st ANSWERS FOR  11th April – Daily Practice MCQs Answers- Daily Practice MCQs Q.1) – d Q.2) – c Q.3) – c

[DAY 34] 60 DAY RAPID REVISION (RaRe) SERIES for UPSC Prelims 2024 – SCIENCE AND TECHNOLOGY, CURRENT AFFAIRS & CSAT TEST SERIES!

Archives Hello Friends The 60 Days Rapid Revision (RaRe) Series is IASbaba’s Flagship Initiative recommended by Toppers and loved by the aspirants’ community every year. It is the most comprehensive program which will help you complete the syllabus, revise and practice tests on a daily basis. The Programme on a daily basis includes Daily Prelims MCQs from Static (Monday – Saturday) Daily Static Quiz will cover all the topics of static subjects – Polity, History, Geography, Economics, Environment and Science and technology. 20 questions will be posted daily and these questions are framed from the topics mentioned in the schedule. It will ensure timely and streamlined revision of your static subjects. Daily Current Affairs MCQs (Monday – Saturday) Daily 5 Current Affairs questions, based on sources like ‘The Hindu’, ‘Indian Express’ and ‘PIB’, would be published from Monday to Saturday according to the schedule. Daily CSAT Quiz (Monday – Friday) CSAT has been an Achilles heel for many aspirants. Daily 5 CSAT Questions will be published. Note – Daily Test of 20 static questions, 10 current affairs, and 5 CSAT questions. (35 Prelims Questions) in QUIZ FORMAT will be updated on a daily basis. To Know More about 60 Days Rapid Revision (RaRe) Series – CLICK HERE   60 Day Rapid Revision (RaRe) Series Schedule – CLICK HERE  Important Note Comment your Scores in the Comment Section. This will keep you accountable, responsible and sincere in days to come. It will help us come out with the Cut-Off on a Daily Basis. Let us know if you enjoyed today’s test 🙂  You can post your comments in the given format  (1) Your Score (2) Matrix Meter (3) New Learning from the Test Time limit: 0 Test-summary 0 of 35 questions completed Questions: 1 2 3 4 5 6 7 8 9 10 11 12 13 14 15 16 17 18 19 20 21 22 23 24 25 26 27 28 29 30 31 32 33 34 35 Information The following Test is based on the syllabus of 60 Days Plan-2023 for UPSC IAS Prelims 2022. To view Solutions, follow these instructions: Click on – ‘Start Test’ button Solve Questions Click on ‘Test Summary’ button Click on ‘Finish Test’ button Now click on ‘View Questions’ button – here you will see solutions and links. You have already completed the test before. Hence you can not start it again. Test is loading... You must sign in or sign up to start the test. You have to finish following test, to start this test: Results 0 of 35 questions answered correctly Your time: Time has elapsed You have scored 0 points out of 0 points, (0) Average score     Your score     Categories Not categorized 0% Your result has been entered into leaderboard Loading Name: E-Mail: Captcha: maximum of 70 points Pos. Name Entered on Points Result Table is loading No data available 1 2 3 4 5 6 7 8 9 10 11 12 13 14 15 16 17 18 19 20 21 22 23 24 25 26 27 28 29 30 31 32 33 34 35 Answered Review Question 1 of 35 1. Question Consider the following statements regarding Palliative Care: It is an approach that improves the quality of life of patients facing the problem associated with a life-threatening illness. Palliative Care is part of the ‘Mission Flexipool’ under National Health Mission (NHM). Which of the statements given above is/are correct? a) 1 only b) 2 only c) Both 1 and 2 d) Neither 1 nor 2 Correct Solution (c) Palliative care is also known as supportive care which is required in the terminal cases of Cancer, AIDS etc. and can be provided relatively simply and inexpensively. (Hence statement 1 is correct) Effective palliative care requires a broad multidisciplinary approach that includes the family and makes use of available community resources. The Ministry of Health and Family Welfare, Government of India constituted an expert group on Palliative care which submitted its report ‘Proposal of Strategies for Palliative Care in India’ in November, 2012. On the basis of the Report, an EPC note for 12th Five Year Plan was formulated. No separate budget is allocated for the implementation of National Palliative Care Program. However, the Palliative Care is part of the ‘Mission Flexipool’ under National Health Mission (NHM). (Hence statement 2 is correct) Incorrect Solution (c) Palliative care is also known as supportive care which is required in the terminal cases of Cancer, AIDS etc. and can be provided relatively simply and inexpensively. (Hence statement 1 is correct) Effective palliative care requires a broad multidisciplinary approach that includes the family and makes use of available community resources. The Ministry of Health and Family Welfare, Government of India constituted an expert group on Palliative care which submitted its report ‘Proposal of Strategies for Palliative Care in India’ in November, 2012. On the basis of the Report, an EPC note for 12th Five Year Plan was formulated. No separate budget is allocated for the implementation of National Palliative Care Program. However, the Palliative Care is part of the ‘Mission Flexipool’ under National Health Mission (NHM). (Hence statement 2 is correct) Question 2 of 35 2. Question Consider the following statements regarding Meningitis: It is an inflammation of the protective membranes covering the brain and spinal cord. Both Bacterial and Viral infections can cause Meningitis. Which of the statements given above is/are correct? a) 1 only b) 2 only c) Both 1 and 2 d) Neither 1 nor 2 Correct Solution (c) Meningitis is an inflammation (swelling) of the protective membranes covering the brain and spinal cord. These membranes are called meninges. The most common causes of meningitis are viral and bacterial infections. Other causes may include: cancer fungi drug-induced reactions Transmission: Infections that cause meningitis can be spread through sneezing and coughing.  (Hence option (c) is the correct answer) Symptoms: Common symptoms of meningitis are neck stiffness, fever, confusion or altered mental status, headaches, nausea and vomiting. Less frequent symptoms are seizures, coma and neurological deficits (for example hearing or vision loss, cognitive impairment, or weakness of the limbs). Types of meningitis caused by viruses or bacteria can have similar symptoms. Symptoms may be stronger in some types of meningitis than in others and require different treatment. Treatment and Prevention: Meningitis is a medical emergency. It is potentially fatal within 24 hours and requires urgent medical attention. Meningitis can vary in severity, appropriate treatment and care depending on the cause. Meningitis caused by bacteria requires immediate antibiotic treatment. Vaccines offer the best protection against common types of bacterial meningitis. Antibiotics for close contacts of those with meningococcal disease, when given promptly, decreases the risk of transmission. Hence both the statements are correct. Incorrect Solution (c) Meningitis is an inflammation (swelling) of the protective membranes covering the brain and spinal cord. These membranes are called meninges. The most common causes of meningitis are viral and bacterial infections. Other causes may include: cancer fungi drug-induced reactions Transmission: Infections that cause meningitis can be spread through sneezing and coughing.  (Hence option (c) is the correct answer) Symptoms: Common symptoms of meningitis are neck stiffness, fever, confusion or altered mental status, headaches, nausea and vomiting. Less frequent symptoms are seizures, coma and neurological deficits (for example hearing or vision loss, cognitive impairment, or weakness of the limbs). Types of meningitis caused by viruses or bacteria can have similar symptoms. Symptoms may be stronger in some types of meningitis than in others and require different treatment. Treatment and Prevention: Meningitis is a medical emergency. It is potentially fatal within 24 hours and requires urgent medical attention. Meningitis can vary in severity, appropriate treatment and care depending on the cause. Meningitis caused by bacteria requires immediate antibiotic treatment. Vaccines offer the best protection against common types of bacterial meningitis. Antibiotics for close contacts of those with meningococcal disease, when given promptly, decreases the risk of transmission. Hence both the statements are correct. Question 3 of 35 3. Question Consider the following statements: Most of the bacteria in human body are harmless. More than one billion people worldwide suffer from fungus infections each year, which are widespread and prevalent. A virus can replicate itself and infect cells. How many of the above statements are correct? a) Only one b) Only two c) All three d) None Correct Solution (b) Bacteria The human body is inhabited by millions of tiny living organisms, which, all together, are called the human microbiota. Bacteria are microbes found on the skin, in the nose, mouth, and especially in the gut. We acquire these bacteria during birth and the first years of life, and they live with us throughout our lives. The human microbiota is involved in healthy growth, in protecting the body from invaders, in helping digestion, and in regulating moods. Some changes in the microbiota may occur during our growth, depending on the foods we eat, the environment in which we live, the people and animals that interact with us, or medicines that we take, such as antibiotics. The human microbiota helps us to keep us healthy, but sometimes these bacteria can also be harmful. We need to take good care of our microbiota to avoid the development of some diseases, such as obesity and asthma. We should eat healthy foods that contribute to the development of a healthy microbiota. (Hence statement 1 is correct) Fungi The fungal disease sometimes referred to as mycosis, is a condition brought on by fungi. More than one billion people worldwide suffer from fungus infections each year, which are widespread and prevalent. (Hence statement 2 is correct) Other animals can develop a variety of fungal illnesses, some of which can be transferred from animals to people. Fungal infections are quite common and rare as well- they can be categorized by site of infection, geography, and immuno-comprised target as well as in biological classification. Most common: Fungal nail infections Vaginal candidiasis or vaginal yeast infection Ringworm Candida infections of the mouth, throat, and esophagus Fungal diseases based on geographic location Blastomycosis- caused by moist soil fungus in US and Canada Cryptococcus Gattai infection- tropical and sub-tropical areas Para coccidioidomycosis- central and south America Coccidioidomycosis (valley fever)- southwestern US, Mexico, central and south America Histoplasmosis- due to bird or bat droppings.   Virus Infected host cells (eukaryotic or prokaryotic) can be cultured and grown, and then the growth medium can be harvested as a source of the virus. Viruses replicate only within living cells. Some viruses are restricted in the kinds of cells in which they replicate, and a few have not yet been cultivated at all under laboratory conditions. However, most viruses are grown in cultured cells, embryonated hen’s eggs, or laboratory animals. (Hence statement 3 is incorrect) Incorrect Solution (b) Bacteria The human body is inhabited by millions of tiny living organisms, which, all together, are called the human microbiota. Bacteria are microbes found on the skin, in the nose, mouth, and especially in the gut. We acquire these bacteria during birth and the first years of life, and they live with us throughout our lives. The human microbiota is involved in healthy growth, in protecting the body from invaders, in helping digestion, and in regulating moods. Some changes in the microbiota may occur during our growth, depending on the foods we eat, the environment in which we live, the people and animals that interact with us, or medicines that we take, such as antibiotics. The human microbiota helps us to keep us healthy, but sometimes these bacteria can also be harmful. We need to take good care of our microbiota to avoid the development of some diseases, such as obesity and asthma. We should eat healthy foods that contribute to the development of a healthy microbiota. (Hence statement 1 is correct) Fungi The fungal disease sometimes referred to as mycosis, is a condition brought on by fungi. More than one billion people worldwide suffer from fungus infections each year, which are widespread and prevalent. (Hence statement 2 is correct) Other animals can develop a variety of fungal illnesses, some of which can be transferred from animals to people. Fungal infections are quite common and rare as well- they can be categorized by site of infection, geography, and immuno-comprised target as well as in biological classification. Most common: Fungal nail infections Vaginal candidiasis or vaginal yeast infection Ringworm Candida infections of the mouth, throat, and esophagus Fungal diseases based on geographic location Blastomycosis- caused by moist soil fungus in US and Canada Cryptococcus Gattai infection- tropical and sub-tropical areas Para coccidioidomycosis- central and south America Coccidioidomycosis (valley fever)- southwestern US, Mexico, central and south America Histoplasmosis- due to bird or bat droppings.   Virus Infected host cells (eukaryotic or prokaryotic) can be cultured and grown, and then the growth medium can be harvested as a source of the virus. Viruses replicate only within living cells. Some viruses are restricted in the kinds of cells in which they replicate, and a few have not yet been cultivated at all under laboratory conditions. However, most viruses are grown in cultured cells, embryonated hen’s eggs, or laboratory animals. (Hence statement 3 is incorrect) Question 4 of 35 4. Question Consider following statements regarding H5N1 virus: It is a highly pathogenic subtype of avian influenza (AI), a virus that causes severe disease and death in birds. It can cause illness in humans and many other animal species. Choose the correct code: a) 1 only b) 2 only c) Both 1 and 2 d) Neither 1 nor 2 Correct Solution (c) H5N1 is a highly pathogenic subtype of avian influenza (AI) that causes severe disease and death in birds. This subtype has caused a number of human infections through close contact with infected birds, or contaminated environments and is often fatal. It can cause illness in humans and many other animal species. A bird-adapted strain of H5N1, called HPAI A(H5N1) for highly pathogenic avian influenza virus of type A of subtype H5N1, is the highly pathogenic causative agent of H5N1 flu, commonly known as avian influenza (“bird flu”). It is enzootic (maintained in the population) in many bird populations, especially in Southeast Asia. (Hence both statements 1 and 2 are correct) Incorrect Solution (c) H5N1 is a highly pathogenic subtype of avian influenza (AI) that causes severe disease and death in birds. This subtype has caused a number of human infections through close contact with infected birds, or contaminated environments and is often fatal. It can cause illness in humans and many other animal species. A bird-adapted strain of H5N1, called HPAI A(H5N1) for highly pathogenic avian influenza virus of type A of subtype H5N1, is the highly pathogenic causative agent of H5N1 flu, commonly known as avian influenza (“bird flu”). It is enzootic (maintained in the population) in many bird populations, especially in Southeast Asia. (Hence both statements 1 and 2 are correct) Question 5 of 35 5. Question Which among the following cells has the ability to develop into any type of cell? a) Muscle cell b) Endodermal cell c) Ectodermal cell d) Stem cell Correct Solution (d) A stem cell is a cell with the unique ability to develop into specialized cell types in the body. (Hence option (d) is the correct answer) Blood stem cells are produced in the bone marrow and can become any kind of blood cell the body needs. Stem cells are constantly dividing and maturing into different types of blood cells, replacing older and worn-out blood cells in the body. They produce billions of new blood cells every day. If the stem cells cannot make enough new blood cells, many serious health problems can occur. These problems may include infections, anemia, or bleeding. Muscle cell A muscle cell is also known as a myocyte when referring to either a cardiac muscle cell (cardiomyocyte) or a smooth muscle cell, as these are both small cells. A skeletal muscle cell is long and threadlike with many nuclei and is called a muscle fiber. A muscle cell, known technically as a myocyte, is a specialized animal cell which can shorten its length using a series of motor proteins specially arranged within the cell. While several associated proteins help, actin and myosin form thick and thin filaments which slide past each other to contract small units of a muscle cell. These units are called sarcomeres, and many of them run end-to-end within a larger fiber called a myofibril. A single muscle cell contains many nuclei, which are pressed against the cell membrane. A muscle cell is a long cell compared to other forms of cells, and many muscle cells connect together to form the long fibers found in muscle tissue. Ectodermal cell The ectoderm is one of the primary layers of cells that exists in an embryo. The ectoderm cells differentiate into cells that form a number of external structures such as skin, sweat glands, skin sensor receptors, and hair follicles. Endodermal cells The endodermal cells generate only the lining of the digestive tube and its glands; mesodermal mesenchyme cells will surround this tube to provide the muscles for peristalsis. The ectoderm gives rise to the skin and the nervous system. The mesoderm specifies the development of several cell types such as bone, muscle, and connective tissue. Cells in the endoderm layer become the linings of the digestive and respiratory system, and form organs such as the liver and pancreas. Incorrect Solution (d) A stem cell is a cell with the unique ability to develop into specialized cell types in the body. (Hence option (d) is the correct answer) Blood stem cells are produced in the bone marrow and can become any kind of blood cell the body needs. Stem cells are constantly dividing and maturing into different types of blood cells, replacing older and worn-out blood cells in the body. They produce billions of new blood cells every day. If the stem cells cannot make enough new blood cells, many serious health problems can occur. These problems may include infections, anemia, or bleeding. Muscle cell A muscle cell is also known as a myocyte when referring to either a cardiac muscle cell (cardiomyocyte) or a smooth muscle cell, as these are both small cells. A skeletal muscle cell is long and threadlike with many nuclei and is called a muscle fiber. A muscle cell, known technically as a myocyte, is a specialized animal cell which can shorten its length using a series of motor proteins specially arranged within the cell. While several associated proteins help, actin and myosin form thick and thin filaments which slide past each other to contract small units of a muscle cell. These units are called sarcomeres, and many of them run end-to-end within a larger fiber called a myofibril. A single muscle cell contains many nuclei, which are pressed against the cell membrane. A muscle cell is a long cell compared to other forms of cells, and many muscle cells connect together to form the long fibers found in muscle tissue. Ectodermal cell The ectoderm is one of the primary layers of cells that exists in an embryo. The ectoderm cells differentiate into cells that form a number of external structures such as skin, sweat glands, skin sensor receptors, and hair follicles. Endodermal cells The endodermal cells generate only the lining of the digestive tube and its glands; mesodermal mesenchyme cells will surround this tube to provide the muscles for peristalsis. The ectoderm gives rise to the skin and the nervous system. The mesoderm specifies the development of several cell types such as bone, muscle, and connective tissue. Cells in the endoderm layer become the linings of the digestive and respiratory system, and form organs such as the liver and pancreas. Question 6 of 35 6. Question Which of the following best describes the Blastomycosis which was recently seen in the news? a) It is a fungal disease b) It is a bacterial disease c) It spreads through Mosquitoes d) None of these Correct Solution (a) Recently, at least one person died and around 100 were confirmed or suspected to be infected by blastomycosis in the United States. It is an infection caused by a fungus of the genus Blastomyces. It is found in moist soil and decomposing wood and leaves in the mid-western, south-central and south-eastern states of the US. Transmission: The microscopic spores from the fungus can get dispersed in the air and travel freely. Human beings can contract blastomycosis by inhaling the spores. It is not contagious between animals and people through the air. The symptoms of blastomycosis in animals are similar to the symptoms in humans. Symptoms: Fever, Cough, Breathing difficulty and muscle aches etc. Treatment: Itraconazole is a type of antifungal medication that is typically used to treat mild to moderate blastomycosis. Hence option (a) is the correct answer. Incorrect Solution (a) Recently, at least one person died and around 100 were confirmed or suspected to be infected by blastomycosis in the United States. It is an infection caused by a fungus of the genus Blastomyces. It is found in moist soil and decomposing wood and leaves in the mid-western, south-central and south-eastern states of the US. Transmission: The microscopic spores from the fungus can get dispersed in the air and travel freely. Human beings can contract blastomycosis by inhaling the spores. It is not contagious between animals and people through the air. The symptoms of blastomycosis in animals are similar to the symptoms in humans. Symptoms: Fever, Cough, Breathing difficulty and muscle aches etc. Treatment: Itraconazole is a type of antifungal medication that is typically used to treat mild to moderate blastomycosis. Hence option (a) is the correct answer. Question 7 of 35 7. Question Which of the following organizations recently launched “One Health Global Leaders Group on Antimicrobial Resistance (AMR)”? a) World Health Organization, UN Food and Agriculture Organization, and the International Organization on Animal Health. b) World Health Organization, Ministry of Health and Family Welfare, and the International Organization on Animal Health. c) World Health Organization, UN Food and Agriculture Organization, and the Ministry of Family Welfare d) World Health Organization, UN Environmental Programme, and the International Organization on Animal Health Correct Solution (a) Global Leaders Group on Antimicrobial Resistance The Global Leaders Group on Antimicrobial Resistance (AMR Leaders) consists of world leaders and experts from across sectors working together to accelerate political action on antimicrobial resistance (AMR). It was established in November 2020 following the recommendation of the Interagency Coordination Group (IACG) on Antimicrobial Resistance to strengthen global political momentum and leadership on AMR. The inaugural meeting of the Group took place in January 2021. One Health Global Leaders Group on Antimicrobial Resistance (AMR) launched by United Nations Tripartite organizations. World Health Organization (WHO), UN Food and Agriculture Organization (FAO), and the International Organization on Animal Health (OIE). (Hence, statement (a) is correct). The group performs an independent global advisory and advocacy role and works to maintain urgency, public support, political momentum and visibility of the AMR challenge on the global health and development agenda. India is a member of the Initiative.   Incorrect Solution (a) Global Leaders Group on Antimicrobial Resistance The Global Leaders Group on Antimicrobial Resistance (AMR Leaders) consists of world leaders and experts from across sectors working together to accelerate political action on antimicrobial resistance (AMR). It was established in November 2020 following the recommendation of the Interagency Coordination Group (IACG) on Antimicrobial Resistance to strengthen global political momentum and leadership on AMR. The inaugural meeting of the Group took place in January 2021. One Health Global Leaders Group on Antimicrobial Resistance (AMR) launched by United Nations Tripartite organizations. World Health Organization (WHO), UN Food and Agriculture Organization (FAO), and the International Organization on Animal Health (OIE). (Hence, statement (a) is correct). The group performs an independent global advisory and advocacy role and works to maintain urgency, public support, political momentum and visibility of the AMR challenge on the global health and development agenda. India is a member of the Initiative.   Question 8 of 35 8. Question Consider the following statements regarding National Centre for Disease Control (NCDC): It is the nodal agency in the country for disease surveillance, facilitating prevention and control of communicable diseases. It functions under the administrative control of Indian Council of Medical Research (ICMR). Which of the statements given above is/are correct? a) 1 only b) 2 only c) Both 1 and 2 d) Neither 1 nor 2 Correct Solution (a) The National Centre for Disease Control (NCDC) recently alerted states about smaller outbreaks for five diseases—typhoid, malaria, dengue, scrub typhus and hepatitis A. It is an institute under the Indian Directorate General of Health Services, Ministry of Health and Family Welfare. (Hence statement 2 is incorrect) The Director, an officer of the Public Health sub-cadre of Central Health Service, is the administrative and technical head of the Institute. History: The National Centre for Disease Control (NCDC), formerly National Institute of Communicable Diseases (NICD) had its origin as Central Malaria Bureau, established at Kasauli (Himachal Pradesh) in 1909 and following expansion was renamed in 1927 as the Malaria Survey of India. In view of the drastic reduction achieved in the incidence of malaria under National Malaria Eradication Programme (NMEP), Government of India decided to reorganize and expand the activities of the institute to cover other communicable diseases. Thus in 1963 the erstwhile MII was renamed as National Institute of Communicable Diseases (NICD) to shoulder these additional responsibilities. In year 2009, NICD transforms into National Centre for Disease Control (NCDC) with a larger mandate of controlling emerging and re-emerging diseases. Headquarters: New Delhi. Functions: It functions as the nodal agency in the country for disease surveillance facilitating prevention and control of communicable diseases. (Hence statement 1 is correct) In coordination with the State Governments, NCDC has the capacity and capability for disease surveillance, outbreak investigation, and rapid response to contain and combat outbreaks. It also deals with Anti-Microbial Resistance (AMR), an emerging area of concern with far-reaching consequences. It also provides referral diagnostic support, capacity building and technical support to States/UTs in the country. Incorrect Solution (a) The National Centre for Disease Control (NCDC) recently alerted states about smaller outbreaks for five diseases—typhoid, malaria, dengue, scrub typhus and hepatitis A. It is an institute under the Indian Directorate General of Health Services, Ministry of Health and Family Welfare. (Hence statement 2 is incorrect) The Director, an officer of the Public Health sub-cadre of Central Health Service, is the administrative and technical head of the Institute. History: The National Centre for Disease Control (NCDC), formerly National Institute of Communicable Diseases (NICD) had its origin as Central Malaria Bureau, established at Kasauli (Himachal Pradesh) in 1909 and following expansion was renamed in 1927 as the Malaria Survey of India. In view of the drastic reduction achieved in the incidence of malaria under National Malaria Eradication Programme (NMEP), Government of India decided to reorganize and expand the activities of the institute to cover other communicable diseases. Thus in 1963 the erstwhile MII was renamed as National Institute of Communicable Diseases (NICD) to shoulder these additional responsibilities. In year 2009, NICD transforms into National Centre for Disease Control (NCDC) with a larger mandate of controlling emerging and re-emerging diseases. Headquarters: New Delhi. Functions: It functions as the nodal agency in the country for disease surveillance facilitating prevention and control of communicable diseases. (Hence statement 1 is correct) In coordination with the State Governments, NCDC has the capacity and capability for disease surveillance, outbreak investigation, and rapid response to contain and combat outbreaks. It also deals with Anti-Microbial Resistance (AMR), an emerging area of concern with far-reaching consequences. It also provides referral diagnostic support, capacity building and technical support to States/UTs in the country. Question 9 of 35 9. Question Which of the following statements regarding Central Drugs Standard Control Organisation (CDSCO) is incorrect? a) It works under the Department of Pharmaceuticals, Ministry of Chemicals and Fertilizers. b) Drugs Controller General of India (DCGI) is the head of the department of the CDSCO of the Government of India. c) It is the Central Drug Authority for discharging functions assigned under the Drugs and Cosmetics Act of 1940. d) CDSCO along with state regulators, is jointly responsible for the grant of licenses to certain specialized categories of critical Drugs such as vaccines and sera, etc. Correct Solution (a) Central Drugs Standard Control Organisation (CDSCO) has recently started conducting joint inspections of identified Drug Manufacturing Units along with the State Drugs Control Administration. It is the Central Drug Authority for discharging functions assigned to the Central Government under the Drugs and Cosmetics Act of 1940. (Hence statement 3 is correct) It works under the Ministry of Health & Family Welfare, the National Regulatory Authority (NRA) of India. (Hence statement 1 is incorrect) Under the Drugs and Cosmetics Act, CDSCO is responsible for – Approval of Drugs. Conduct Clinical Trials. Laying down the standards for Drugs. Control over the quality of imported Drugs in the country. Coordination of the activities of State Drug Control Organizations. Further CDSCO along with state regulators, is jointly responsible for the grant of licenses to certain specialised categories of critical Drugs such as vaccines and sera, etc. (Hence statement 4 is correct) The Indian government has announced plans to subject all medical devices, including implants and contraception, to CDSCO scrutiny. Drugs Controller General of India (DCGI): DCGI is the head of the department of the CDSCO of the Government of India responsible for approval of licenses of specified categories of drugs such as blood and blood products, IV fluids, vaccines and sera in India. (Hence statement 2 is correct) DCGI also sets standards for the manufacturing, sales, import, and distribution of drugs in India. Incorrect Solution (a) Central Drugs Standard Control Organisation (CDSCO) has recently started conducting joint inspections of identified Drug Manufacturing Units along with the State Drugs Control Administration. It is the Central Drug Authority for discharging functions assigned to the Central Government under the Drugs and Cosmetics Act of 1940. (Hence statement 3 is correct) It works under the Ministry of Health & Family Welfare, the National Regulatory Authority (NRA) of India. (Hence statement 1 is incorrect) Under the Drugs and Cosmetics Act, CDSCO is responsible for – Approval of Drugs. Conduct Clinical Trials. Laying down the standards for Drugs. Control over the quality of imported Drugs in the country. Coordination of the activities of State Drug Control Organizations. Further CDSCO along with state regulators, is jointly responsible for the grant of licenses to certain specialised categories of critical Drugs such as vaccines and sera, etc. (Hence statement 4 is correct) The Indian government has announced plans to subject all medical devices, including implants and contraception, to CDSCO scrutiny. Drugs Controller General of India (DCGI): DCGI is the head of the department of the CDSCO of the Government of India responsible for approval of licenses of specified categories of drugs such as blood and blood products, IV fluids, vaccines and sera in India. (Hence statement 2 is correct) DCGI also sets standards for the manufacturing, sales, import, and distribution of drugs in India. Question 10 of 35 10. Question Consider the following statements regarding Monkey Pox Virus: It is a fungal zoonotic disease that occurs primarily in tropical rainforest areas of Central and West Africa. The first case of monkeypox in humans was recorded in 1970 in the Democratic Republic of Congo. Monkeypox can be eradicated using vaccine completely. How many of the above statements are correct? a) Only one b) Only two c) All three d) None Correct Solution (a) Recently, the World Health Organisation has declared the global monkeypox outbreak a ‘public health emergency of international concern’ (PHEIC). Monkeypox is a viral zoonotic disease that occurs primarily in tropical rainforest areas of Central and West Africa and is occasionally exported to other regions. (Hence statement 1 is incorrect) Monkeypox virus is an enveloped double-stranded DNA virus that belongs to the Orthopoxvirus genus of the Poxviridae family. The first case of monkeypox in humans was recorded in 1970 in the Democratic Republic of Congo during a period of intensified effort to eliminate smallpox. (Hence statement 2 is correct) Scientists have so far discovered two distinct genetic groups of monkeypox virus—Central African and West African. Cause It is caused by monkeypox virus, a member of the Orthopoxvirus genus in the family Poxviridae. Monkeypox belongs to the same family of viruses as smallpox. But Smallpox had a higher fatality rate than monkeypox. Smallpox was declared eradicated worldwide in 1980. Symptoms Monkeypox is characterised by flu-like symptoms and swelling of the lymph nodes, which gradually progresses to a widespread rash all over the face and body. Transmission Monkeypox virus is mostly transmitted to people from wild animals such as rodents and primates, but human-to-human transmission also occurs. It spreads from one person to another via respiratory droplets. Treatment There is currently no specific treatment recommended for monkeypox. (Hence statement 3 is incorrect) Vaccination against smallpox was about 85% effective in preventing monkeypox. Thus, prior childhood smallpox vaccination may result in a milder disease course. Incorrect Solution (a) Recently, the World Health Organisation has declared the global monkeypox outbreak a ‘public health emergency of international concern’ (PHEIC). Monkeypox is a viral zoonotic disease that occurs primarily in tropical rainforest areas of Central and West Africa and is occasionally exported to other regions. (Hence statement 1 is incorrect) Monkeypox virus is an enveloped double-stranded DNA virus that belongs to the Orthopoxvirus genus of the Poxviridae family. The first case of monkeypox in humans was recorded in 1970 in the Democratic Republic of Congo during a period of intensified effort to eliminate smallpox. (Hence statement 2 is correct) Scientists have so far discovered two distinct genetic groups of monkeypox virus—Central African and West African. Cause It is caused by monkeypox virus, a member of the Orthopoxvirus genus in the family Poxviridae. Monkeypox belongs to the same family of viruses as smallpox. But Smallpox had a higher fatality rate than monkeypox. Smallpox was declared eradicated worldwide in 1980. Symptoms Monkeypox is characterised by flu-like symptoms and swelling of the lymph nodes, which gradually progresses to a widespread rash all over the face and body. Transmission Monkeypox virus is mostly transmitted to people from wild animals such as rodents and primates, but human-to-human transmission also occurs. It spreads from one person to another via respiratory droplets. Treatment There is currently no specific treatment recommended for monkeypox. (Hence statement 3 is incorrect) Vaccination against smallpox was about 85% effective in preventing monkeypox. Thus, prior childhood smallpox vaccination may result in a milder disease course. Question 11 of 35 11. Question Which of the following are the applications of CRISPR Technology? Genetic sequence of disease-causing organisms can be altered to make them ineffective. Diseases and disorders including some forms of cancer caused by an undesired genetic mutation can be fixed. Till date genes of plants cannot be edited to make them pest resistant. How many of the above statements are correct? a) Only one b) Only two c) All three d) None Correct Solution (b) The CRISPR (Clustered Regularly Interspaced Short Palindromic Repeats) technology works in a simple way — it locates the specific area in the genetic sequence which has been diagnosed to be the cause of the problem, cuts it out, and replaces it with a new and correct sequence that no longer causes the problem. The technology replicates a natural defense mechanism in some bacteria that uses a similar method to protect itself from virus attacks. Because the entire process is programmable, it has a remarkable efficiency, and has already brought almost miraculous results. There are a whole lot of diseases and disorders, including some forms of cancer, that are caused by an undesired genetic mutation. These can all be fixed with this technology. (Hence statement 2 is correct) There are vast applications elsewhere as well. Genetic sequence of disease-causing organisms can be altered to make them ineffective. (Hence statement 1 is correct) Genes of plants can be edited to make them withstand pests, or improve their tolerance to drought or temperature. (Hence statement 3 is incorrect) Incorrect Solution (b) The CRISPR (Clustered Regularly Interspaced Short Palindromic Repeats) technology works in a simple way — it locates the specific area in the genetic sequence which has been diagnosed to be the cause of the problem, cuts it out, and replaces it with a new and correct sequence that no longer causes the problem. The technology replicates a natural defense mechanism in some bacteria that uses a similar method to protect itself from virus attacks. Because the entire process is programmable, it has a remarkable efficiency, and has already brought almost miraculous results. There are a whole lot of diseases and disorders, including some forms of cancer, that are caused by an undesired genetic mutation. These can all be fixed with this technology. (Hence statement 2 is correct) There are vast applications elsewhere as well. Genetic sequence of disease-causing organisms can be altered to make them ineffective. (Hence statement 1 is correct) Genes of plants can be edited to make them withstand pests, or improve their tolerance to drought or temperature. (Hence statement 3 is incorrect) Question 12 of 35 12. Question Consider the following statements regarding Alzheimer’s disease. It is a progressive heart disorder that typically affects older people. It affects the ability to perform day-to-day activities of human. Alzheimer’s disease can be treated using vaccines. How many of the above statements are correct? a) Only one b) Only two c) All three d) None Correct Solution (a) Alzheimer’s disease is a progressive brain disorder that typically affects people older than 65. (Hence statement 1 is incorrect) When it affects younger individuals, it is considered early onset. The disease destroys brain cells and nerves, and disrupts the message-carrying neurotransmitters. Eventually, a person with Alzheimer’s loses the ability to perform day-to-day activities. (Hence statement 2 is correct) Symptoms include memory loss, difficulty in completing familiar tasks, confusion with time or place, problems in speaking and writing, decreased or poor judgment, and changes in mood and personality. Alzheimer’s disease is also the most common cause of dementia — which is a syndrome and not a disease in itself, and whose symptoms include loss of memory, thinking skills, problems with language, changes in mood and deterioration in behaviour. Treatment: There is no cure for Alzheimer’s, because its exact causes are not known. (Hence statement 3 is incorrect) Most drugs being developed try to slow down or stop the progression of the disease. Incorrect Solution (a) Alzheimer’s disease is a progressive brain disorder that typically affects people older than 65. (Hence statement 1 is incorrect) When it affects younger individuals, it is considered early onset. The disease destroys brain cells and nerves, and disrupts the message-carrying neurotransmitters. Eventually, a person with Alzheimer’s loses the ability to perform day-to-day activities. (Hence statement 2 is correct) Symptoms include memory loss, difficulty in completing familiar tasks, confusion with time or place, problems in speaking and writing, decreased or poor judgment, and changes in mood and personality. Alzheimer’s disease is also the most common cause of dementia — which is a syndrome and not a disease in itself, and whose symptoms include loss of memory, thinking skills, problems with language, changes in mood and deterioration in behaviour. Treatment: There is no cure for Alzheimer’s, because its exact causes are not known. (Hence statement 3 is incorrect) Most drugs being developed try to slow down or stop the progression of the disease. Question 13 of 35 13. Question Consider the following statements: Human papillomavirus (HPV) is a viral infection of the reproductive tract. Majority of cervical cancer is due to HPV. India has recently developed its first human papillomavirus vaccine. How many of the above statements are correct? a) Only one b) Only two c) All three d) None Correct Solution (c) Cervavac, a vaccine developed by the Serum Institute of India (SII), has recently acquired market authorization from the Drugs Controller General of India (DGCI). It is India’s first Quadrivalent Human Papillomavirus Vaccination (qHPV) designed to protect women from cervical cancer. Cervical cancer is a prevalent sexually transmitted infection. It is a type of cancer that occurs in the cells of the cervix, the lower part of the uterus that connects to the vagina. It is mostly caused by long-term infection with particular forms of HPV. It is the second most prevalent cancer form and the second leading cause of cancer death in women of reproductive age (15-44). Hence all the statements are correct. Incorrect Solution (c) Cervavac, a vaccine developed by the Serum Institute of India (SII), has recently acquired market authorization from the Drugs Controller General of India (DGCI). It is India’s first Quadrivalent Human Papillomavirus Vaccination (qHPV) designed to protect women from cervical cancer. Cervical cancer is a prevalent sexually transmitted infection. It is a type of cancer that occurs in the cells of the cervix, the lower part of the uterus that connects to the vagina. It is mostly caused by long-term infection with particular forms of HPV. It is the second most prevalent cancer form and the second leading cause of cancer death in women of reproductive age (15-44). Hence all the statements are correct. Question 14 of 35 14. Question Consider the following statements regarding Artificial insemination and In-Vitro Fertilization: In-Vitro Fertilization technique introduces previously selected semen into the woman’s uterus that has been prepared by stimulating ovulation. Artificial insemination technique consists of retrieving a woman’s eggs to be fertilised in the laboratory. Which of the statements given above is/are correct? a) 1 only b) 2 only c) Both 1 and 2 d) Neither 1 nor 2 Correct Solution (d) Statement 1 Statement 2 Incorrect Incorrect In Vitro Fertilization- This technique consists of retrieving a woman’s eggs to be fertilised in the laboratory and later introducing the obtained embryos inside the uterus. Artificial insemination- This technique introduces previously selected semen into the woman’s uterus that has been prepared by stimulating ovulation. Incorrect Solution (d) Statement 1 Statement 2 Incorrect Incorrect In Vitro Fertilization- This technique consists of retrieving a woman’s eggs to be fertilised in the laboratory and later introducing the obtained embryos inside the uterus. Artificial insemination- This technique introduces previously selected semen into the woman’s uterus that has been prepared by stimulating ovulation. Question 15 of 35 15. Question Solution (b) Statement 1 Statement 2 Statement 3 Incorrect Correct Correct The strength of inactivated vaccines tends to wear off over time, resulting in less long-lasting immunity. Hence multiple doses are required to provide long-lasting immunity. Live-attenuated vaccines cannot be administered to persons with impaired immune response system. In a recombinant vaccine, only a protein from disease causing organism that generates protective immune response is inserted into gene of another cell. a) Only one b) Only two c) All three d) None Correct Test explained Incorrect Test explained Question 16 of 35 16. Question With reference to advancement in technology, consider the following statements: Viruses can be cultured without living cells. Fungi can be cultured only in natural medium. Bacteria can be cultured both naturally and artificial mediums. How many of the above statements are correct? a) Only one b) Only two c) All three d) None Correct Solution (a) Bacteria and Fungi can be cultured in an artificial/ synthetic medium. Whereas viruses require a living host cell for replication. Viruses replicate only within living cells. Some viruses are restricted in the kinds of cells in which they replicate, and a few have not yet been cultivated at all under laboratory conditions. Infected host cells (eukaryotic or prokaryotic) can be cultured and grown, and then the growth medium can be harvested as a source of the virus. (Hence statement 1 and 2 are incorrect and 3 is correct)   Incorrect Solution (a) Bacteria and Fungi can be cultured in an artificial/ synthetic medium. Whereas viruses require a living host cell for replication. Viruses replicate only within living cells. Some viruses are restricted in the kinds of cells in which they replicate, and a few have not yet been cultivated at all under laboratory conditions. Infected host cells (eukaryotic or prokaryotic) can be cultured and grown, and then the growth medium can be harvested as a source of the virus. (Hence statement 1 and 2 are incorrect and 3 is correct)   Question 17 of 35 17. Question Consider the following with respect to E.coli bacteria: coli bacteria are found in the gut of some animals. Most types of E. coli are harmless. They help in keeping the digestive tract healthy. How many of the above statements are correct? a) Only one b) Only two c) All three d) None Correct Solution (c) Statement 1 Statement 2 Statement 3 Correct Correct Correct E. coli (Escherichia coli), is a type of bacteria that normally lives in your intestines. It’s also found in the gut of some animals.  Hence, statement 1 is correct. Most types of E. coli are harmless and even help keep your digestive tract healthy. But some strains can cause diarrhea if you eat contaminated food or drink fouled water. Hence, statement 2 is correct. Most types of E. coli are harmless and even help keep your digestive tract healthy. But some strains can cause diarrhea if you eat contaminated food or drink fouled water. Hence, statement 3 is correct. Incorrect Solution (c) Statement 1 Statement 2 Statement 3 Correct Correct Correct E. coli (Escherichia coli), is a type of bacteria that normally lives in your intestines. It’s also found in the gut of some animals.  Hence, statement 1 is correct. Most types of E. coli are harmless and even help keep your digestive tract healthy. But some strains can cause diarrhea if you eat contaminated food or drink fouled water. Hence, statement 2 is correct. Most types of E. coli are harmless and even help keep your digestive tract healthy. But some strains can cause diarrhea if you eat contaminated food or drink fouled water. Hence, statement 3 is correct. Question 18 of 35 18. Question Consider the following: Antimicrobials are medicines used to prevent and treat infections in humans, animals and plants. Medical procedures, such as surgery, cancer chemotherapy, and organ transplantation, will not be affected by antimicrobial resistance. Which of the statements given above is/are correct? a) 1 only b) 2 only c) Both 1 and 2 d) Neither 1 nor 2 Correct Solution (a) Statement 1 Statement 2 Correct Incorrect Antimicrobials – including antibiotics, antivirals, antifungals and antiparasitics – are medicines used to prevent and treat infections in humans, animals and plants. Hence, statement 1 is true. Without effective tools for the prevention and adequate treatment of drug-resistant infections and improved access to existing and new quality-assured antimicrobials, the number of people for whom treatment is failing or who die of infections will increase. Medical procedures, such as surgery, including caesarean sections or hip replacements, cancer chemotherapy, and organ transplantation, will become riskier. Hence, statement 2 is not true. Incorrect Solution (a) Statement 1 Statement 2 Correct Incorrect Antimicrobials – including antibiotics, antivirals, antifungals and antiparasitics – are medicines used to prevent and treat infections in humans, animals and plants. Hence, statement 1 is true. Without effective tools for the prevention and adequate treatment of drug-resistant infections and improved access to existing and new quality-assured antimicrobials, the number of people for whom treatment is failing or who die of infections will increase. Medical procedures, such as surgery, including caesarean sections or hip replacements, cancer chemotherapy, and organ transplantation, will become riskier. Hence, statement 2 is not true. Question 19 of 35 19. Question Consider the following with respect to Spinal Muscular Atrophy (SMA): It is a neurological condition caused by a defect in the SMN1 gene. There are broadly three types of SMA depending on the age at which weakness starts and severity of weakness. SMA Type 1 being most severe. How many of the statements given above is/are correct? a) Only one b) Only two c) All three d) None Correct Solution (c) Statement 1,2,3 Correct Spinal Muscular Atrophy is a neurological condition caused by a defect in the SMN1 gene. Normally, every person is born with a gene called SMN1 which produces a protein called SMN protein, in many cells in our body. This protein is essential for normal functioning of nerve cells in the spinal cord called the ‘anterior horn cells’. The anterior horn cells control the skeletal muscles essential for all our movements (of limbs, neck, back, breathing and swallowing etc,). However, absence of SMN1 gene causes reduction in the amount of SMN protein produced in anterior horn cells. The reduced quantity of SMN protein causes gradual death of anterior horn cells, and thus progressive weakness of muscles of limbs, trunk and breathing and swallowing muscles. Depending on the age of child and severity of SMA, the speed with which the muscles become weaker varies. There are broadly three types of SMA depending on the age at which weakness starts and severity of weakness, with type 1 being most severe. Incorrect Solution (c) Statement 1,2,3 Correct Spinal Muscular Atrophy is a neurological condition caused by a defect in the SMN1 gene. Normally, every person is born with a gene called SMN1 which produces a protein called SMN protein, in many cells in our body. This protein is essential for normal functioning of nerve cells in the spinal cord called the ‘anterior horn cells’. The anterior horn cells control the skeletal muscles essential for all our movements (of limbs, neck, back, breathing and swallowing etc,). However, absence of SMN1 gene causes reduction in the amount of SMN protein produced in anterior horn cells. The reduced quantity of SMN protein causes gradual death of anterior horn cells, and thus progressive weakness of muscles of limbs, trunk and breathing and swallowing muscles. Depending on the age of child and severity of SMA, the speed with which the muscles become weaker varies. There are broadly three types of SMA depending on the age at which weakness starts and severity of weakness, with type 1 being most severe. Question 20 of 35 20. Question Consider the following statements: Adenoviruses have single-stranded DNA genomes whereas retroviruses have double-stranded DNA genomes. Common cold is mostly caused by an adenovirus whereas AIDS is caused by a retrovirus. Which of the statements given above is/are correct? a) 1 only b) 2 only c) Both 1 and 2 d) Neither 1 nor 2 Correct Solution (b) Statement 1 Statement 2 Incorrect Correct Adenoviruses are medium-sized, nonenveloped viruses with an icosahedral nucleocapsid containing a double-stranded DNA genome. Retroviruses (Rv) are double-stranded RNA viruses that integrate in to host the chromosome, and thus provide ‘permanent’ transfer of genes to a cell. Adenoviruses are common viruses that typically cause mild cold- or flu-like illness. Adenoviruses can cause illness in people of all ages any time of year.   HIV is classified as a retrovirus because it contains reverse transcriptase. Retroviruses are a family of viruses that are grouped together based on how they are structured and how they replicate within a host. Besides human immunodeficiency virus (HIV), the virus that causes AIDS, there two other retroviruses that can cause human illness. One is called human T-lymphotropic virus type 1 (HTLV-1) and the other is called human T-lymphotropic virus type 2 (HTLV-II). Both of these viruses are transmitted between people through sexual contact, infected blood or tissue exposure, or during pregnancy or childbirth from an infected person to their child. Incorrect Solution (b) Statement 1 Statement 2 Incorrect Correct Adenoviruses are medium-sized, nonenveloped viruses with an icosahedral nucleocapsid containing a double-stranded DNA genome. Retroviruses (Rv) are double-stranded RNA viruses that integrate in to host the chromosome, and thus provide ‘permanent’ transfer of genes to a cell. Adenoviruses are common viruses that typically cause mild cold- or flu-like illness. Adenoviruses can cause illness in people of all ages any time of year.   HIV is classified as a retrovirus because it contains reverse transcriptase. Retroviruses are a family of viruses that are grouped together based on how they are structured and how they replicate within a host. Besides human immunodeficiency virus (HIV), the virus that causes AIDS, there two other retroviruses that can cause human illness. One is called human T-lymphotropic virus type 1 (HTLV-1) and the other is called human T-lymphotropic virus type 2 (HTLV-II). Both of these viruses are transmitted between people through sexual contact, infected blood or tissue exposure, or during pregnancy or childbirth from an infected person to their child. Question 21 of 35 21. Question Consider the following statements regarding Valmiki Tiger Reserve (VTR): It forms the easternmost limit of the Himalayan Terai forests in India. It is the only tiger reserve in Jharkhand. It is surrounded by the Royal Chitwan National Park of Nepal in the north. Its vegetation is dominated by the moist mixed deciduous forest. How many of the above statements are correct? a) Only one b) Only two c) Only three d) All four Correct Solution (c) Valmiki Tiger Reserve (VTR) forms the easternmost limit of the Himalayan Terai forests in India. Hence statement 1 is correct. It is situated in the Gangetic plains bio-geographic zone of the country. The forest of this region has a combination of bhabar and terai tracts. Rivers Gandak, Pandai, Manor, Harha, Masan, and Bhapsa flow through various parts of the reserve. It is the only tiger reserve in the West Champaran district of Bihar. Palamu Tiger Reserve is the only tiger reserve in Jharkhand. Hence statement 2 is incorrect. It is surrounded by the Royal Chitwan National Park of Nepal in the north. Hence statement 3 is correct. Its vegetation is dominated by the moist mixed deciduous forest. Hence statement 4 is correct. It also includes open-land vegetation, sub-mountainous semi-evergreen formations, freshwater swamps, riparian fringes, alluvial grasslands, high hill savannahs, and wetlands. Incorrect Solution (c) Valmiki Tiger Reserve (VTR) forms the easternmost limit of the Himalayan Terai forests in India. Hence statement 1 is correct. It is situated in the Gangetic plains bio-geographic zone of the country. The forest of this region has a combination of bhabar and terai tracts. Rivers Gandak, Pandai, Manor, Harha, Masan, and Bhapsa flow through various parts of the reserve. It is the only tiger reserve in the West Champaran district of Bihar. Palamu Tiger Reserve is the only tiger reserve in Jharkhand. Hence statement 2 is incorrect. It is surrounded by the Royal Chitwan National Park of Nepal in the north. Hence statement 3 is correct. Its vegetation is dominated by the moist mixed deciduous forest. Hence statement 4 is correct. It also includes open-land vegetation, sub-mountainous semi-evergreen formations, freshwater swamps, riparian fringes, alluvial grasslands, high hill savannahs, and wetlands. Question 22 of 35 22. Question Consider the following statements about the Strait of Gibraltar: It is a narrow waterway separating Europe from Asia. It is bordered by Morocco in the north. It connects the Mediterranean Sea to the Atlantic Ocean. How many of the above statements are correct? a) Only one b) Only two c) All three d) None Correct Solution (a) The Strait of Gibraltar is a narrow waterway separating Europe from Africa. Hence statement 1 is incorrect. It is one of the busiest waterways in the world. Approximately 300 ships cross the Strait every day, about one ship every 5 minutes. It is bordered by Spain and the British Overseas Territory of Gibraltar in the north. It is bordered by Morocco and the Spanish exclave of Ceuta in the south. Hence statement 2 is incorrect. Its depth ranges from 300 to 900 meters, and it forms a significant gap between the high plateau of Spain and the Atlas Mountains of Northern Africa. It connects the Mediterranean Sea to the Atlantic Ocean. Hence statement 3 is correct. An important port located on the strait is the Moroccan port of Tanger-Med, near Tangier. Incorrect Solution (a) The Strait of Gibraltar is a narrow waterway separating Europe from Africa. Hence statement 1 is incorrect. It is one of the busiest waterways in the world. Approximately 300 ships cross the Strait every day, about one ship every 5 minutes. It is bordered by Spain and the British Overseas Territory of Gibraltar in the north. It is bordered by Morocco and the Spanish exclave of Ceuta in the south. Hence statement 2 is incorrect. Its depth ranges from 300 to 900 meters, and it forms a significant gap between the high plateau of Spain and the Atlas Mountains of Northern Africa. It connects the Mediterranean Sea to the Atlantic Ocean. Hence statement 3 is correct. An important port located on the strait is the Moroccan port of Tanger-Med, near Tangier. Question 23 of 35 23. Question Consider the following statements about Reindeers: They are known to see ultraviolet light. They are spotted in Europe, Asia, and North America. They are classified as critically endangered on the IUCN List. How many of the above statements are correct? a) Only one b) Only two c) All three d) None Correct Solution (b) Reindeers are known to see ultraviolet light. It helps them to spot predators whose white fur is more visible in UV light. Hence statement 1 is correct. They can use their nose to warm the air they breathe in before it enters their lungs. They are spotted in Europe, Asia, and North America. Hence statement 2 is correct. Both male and female reindeer grow antlers. However, male reindeer antlers tend to be twice as big. They are herbivores that are found in polar and arctic climates. They are classified as vulnerable on the IUCN List. Hence statement 3 is incorrect. Note: Researchers have discovered that the more time reindeer spend ruminating, the less time they spend in non-rapid eye movement (non-REM) sleep. It might help reindeer get enough sleep during the summer months when food is abundant and reindeer feed almost 24/7.   Incorrect Solution (b) Reindeers are known to see ultraviolet light. It helps them to spot predators whose white fur is more visible in UV light. Hence statement 1 is correct. They can use their nose to warm the air they breathe in before it enters their lungs. They are spotted in Europe, Asia, and North America. Hence statement 2 is correct. Both male and female reindeer grow antlers. However, male reindeer antlers tend to be twice as big. They are herbivores that are found in polar and arctic climates. They are classified as vulnerable on the IUCN List. Hence statement 3 is incorrect. Note: Researchers have discovered that the more time reindeer spend ruminating, the less time they spend in non-rapid eye movement (non-REM) sleep. It might help reindeer get enough sleep during the summer months when food is abundant and reindeer feed almost 24/7.   Question 24 of 35 24. Question Consider the following statements: Rights of Persons with Disabilities Act of 2016 includes mental illness, cerebral palsy, and muscular dystrophy as types of disabilities. India has ratified the United Nations Convention on the Rights of Persons with Disabilities. The subject of ‘relief of the disabled and unemployable’ is specified in the union list of the seventh schedule of the Indian constitution. How many of the above statements are correct? a) Only one b) Only two c) All three d) None Correct Solution (b) Rights of Persons with Disabilities Act of 2016 includes mental illness, cerebral palsy, autism, chronic neurological conditions, speech and language disability, thalassemia, hemophilia, sickle cell disease, multiple disabilities including deaf blindness, acid attack victims, Parkinson’s disease, and muscular dystrophy as types of disabilities. Hence statement 1 is correct. It increases the quantum of reservations for people suffering from disabilities from 3% to 4% in government jobs and from 3% to 5% in higher education institutes. Every child with a benchmark disability between the age group of 6 and 18 years shall have the right to free education. “Person with a disability” means a person with long-term physical, mental, intellectual, or sensory impairments which, in interaction with barriers, hinder his full and effective participation in society equally with others. India has ratified the United Nations Convention on Rights of Persons with Disability in 2007. Hence statement 2 is correct. It was adopted in 2006, and defines persons with disabilities as those “who have long-term physical, mental, intellectual or sensory impairments which in interaction with various barriers may hinder their full and effective participation in society on an equal basis with others”. The subject of ‘relief of the disabled and unemployable’ is specified in the state list of the seventh schedule of the Indian constitution. Hence statement 3 is incorrect. Article 41 of the Directive Principles of State Policy (DPSP) states that the State shall make effective provisions for securing the right to work, education, and public assistance in cases of unemployment, old age, sickness, and disablement, within the limits of its economic capacity and development.   Incorrect Solution (b) Rights of Persons with Disabilities Act of 2016 includes mental illness, cerebral palsy, autism, chronic neurological conditions, speech and language disability, thalassemia, hemophilia, sickle cell disease, multiple disabilities including deaf blindness, acid attack victims, Parkinson’s disease, and muscular dystrophy as types of disabilities. Hence statement 1 is correct. It increases the quantum of reservations for people suffering from disabilities from 3% to 4% in government jobs and from 3% to 5% in higher education institutes. Every child with a benchmark disability between the age group of 6 and 18 years shall have the right to free education. “Person with a disability” means a person with long-term physical, mental, intellectual, or sensory impairments which, in interaction with barriers, hinder his full and effective participation in society equally with others. India has ratified the United Nations Convention on Rights of Persons with Disability in 2007. Hence statement 2 is correct. It was adopted in 2006, and defines persons with disabilities as those “who have long-term physical, mental, intellectual or sensory impairments which in interaction with various barriers may hinder their full and effective participation in society on an equal basis with others”. The subject of ‘relief of the disabled and unemployable’ is specified in the state list of the seventh schedule of the Indian constitution. Hence statement 3 is incorrect. Article 41 of the Directive Principles of State Policy (DPSP) states that the State shall make effective provisions for securing the right to work, education, and public assistance in cases of unemployment, old age, sickness, and disablement, within the limits of its economic capacity and development.   Question 25 of 35 25. Question Consider the following statements regarding Huntington’s Disease (HD): It is a hereditary neurodegenerative disorder characterized by the progressive breakdown of nerve cells in the brain. It is caused by a mutation in the huntingtin gene (HTT), leading to the production of a defective form of the huntingtin protein. Its symptoms include involuntary movements, cognitive decline, and emotional disturbances. How many of the above statements are correct? a) Only one b) Only two c) All three d) None Correct Solution (c) Huntington’s Disease (HD) is a hereditary neurodegenerative disorder characterized by the progressive breakdown of nerve cells in the brain. Hence statement 1 is correct. It is caused by a mutation in the huntingtin gene (HTT), leading to the production of a defective form of the huntingtin protein. Hence statement 2 is correct. Its symptoms include involuntary movements, cognitive decline, and emotional disturbances. Hence statement 3 is correct. It is currently incurable and treatment cannot reverse its progression or slow it down. However, healthcare providers can offer medications to help with certain symptoms. Incorrect Solution (c) Huntington’s Disease (HD) is a hereditary neurodegenerative disorder characterized by the progressive breakdown of nerve cells in the brain. Hence statement 1 is correct. It is caused by a mutation in the huntingtin gene (HTT), leading to the production of a defective form of the huntingtin protein. Hence statement 2 is correct. Its symptoms include involuntary movements, cognitive decline, and emotional disturbances. Hence statement 3 is correct. It is currently incurable and treatment cannot reverse its progression or slow it down. However, healthcare providers can offer medications to help with certain symptoms. Question 26 of 35 26. Question Consider the following statements: Bikaji Cama established the Paris Indian Society with co-founders Munchershah Burjorji Godrej and S.R. Rana. Sucheta Kriplani founded the All-India Mahila Congress and was the first woman Chief Minister of Uttar Pradesh. Kanaklata Barua led a group of freedom fighters called Mukti Bahini during the Swadeshi Movement. How many of the above statements are correct? a) Only one b) Only two c) All three d) None Correct Solution (b) Bikaji Cama (1861-1936) established the Paris Indian Society with co-founders Munchershah Burjorji Godrej and S.R. Rana. Hence statement 1 is correct. She actively participated in the Indian freedom movement and advocated for Swaraj (self-rule). Sucheta Kriplani (1908-1974) founded the All India Mahila Congress in 1940 to advocate women’s rights and empowerment. She was the first woman Chief Minister of Uttar Pradesh. Hence statement 2 is correct. She worked closely with Mahatma Gandhi during the tumultuous time of the Partition riots, accompanying him to Noakhali in 1946. Kanaklata Barua (1924-1942) led a group of freedom fighters called Mukti Bahini during the Quit India Movement. She died during a procession to unfurl the Tricolour at Gohpur police station on September 20, 1942. Hence statement 3 is incorrect. Incorrect Solution (b) Bikaji Cama (1861-1936) established the Paris Indian Society with co-founders Munchershah Burjorji Godrej and S.R. Rana. Hence statement 1 is correct. She actively participated in the Indian freedom movement and advocated for Swaraj (self-rule). Sucheta Kriplani (1908-1974) founded the All India Mahila Congress in 1940 to advocate women’s rights and empowerment. She was the first woman Chief Minister of Uttar Pradesh. Hence statement 2 is correct. She worked closely with Mahatma Gandhi during the tumultuous time of the Partition riots, accompanying him to Noakhali in 1946. Kanaklata Barua (1924-1942) led a group of freedom fighters called Mukti Bahini during the Quit India Movement. She died during a procession to unfurl the Tricolour at Gohpur police station on September 20, 1942. Hence statement 3 is incorrect. Question 27 of 35 27. Question Consider the following statements about Polar Stratospheric Clouds (PSC): These are formed in very cold conditions over polar regions and within the troposphere. These clouds provide a surface for chemical reactions where benign forms of chlorine are converted into reactive, ozone-destroying forms. Choose the correct code: a) 1 only b) 2 only c) Both 1 and 2 d) Neither 1 nor 2 Correct Solution (b) Polar Stratospheric Clouds (PSC) are formed in very cold conditions over polar regions and within the stratosphere, around 12-19 miles (19-31km) high, far above our normal clouds. Hence statement 1 is incorrect. These clouds are made of smaller ice particles than those that form more common clouds. These small particles help to scatter light in a different way, which gives them their unique appearance. The frequency of these clouds is often a harbinger of ozone depletion, as they play a crucial role in stratospheric chemistry. These clouds provide a surface for chemical reactions where benign forms of chlorine are converted into reactive, ozone-destroying forms. Hence statement 2 is correct. They also remove nitrogen compounds that would otherwise moderate the destructive impact of chlorine on the ozone layer. Incorrect Solution (b) Polar Stratospheric Clouds (PSC) are formed in very cold conditions over polar regions and within the stratosphere, around 12-19 miles (19-31km) high, far above our normal clouds. Hence statement 1 is incorrect. These clouds are made of smaller ice particles than those that form more common clouds. These small particles help to scatter light in a different way, which gives them their unique appearance. The frequency of these clouds is often a harbinger of ozone depletion, as they play a crucial role in stratospheric chemistry. These clouds provide a surface for chemical reactions where benign forms of chlorine are converted into reactive, ozone-destroying forms. Hence statement 2 is correct. They also remove nitrogen compounds that would otherwise moderate the destructive impact of chlorine on the ozone layer. Question 28 of 35 28. Question Consider the following statements about the Bhoomi Rashi Portal: It is an e-governance initiative of the Ministry of Road Transport and Highways. It intends to expedite the process of land acquisition for National Highways. Choose the correct code: a) 1 only b) 2 only c) Both 1 and 2 d) Neither 1 nor 2 Correct Solution (c) The Bhoomi Rashi Portal is an e-governance initiative of the Ministry of Road Transport and Highways. Hence statement 1 is correct. It acts as a single-point platform for the online processing of land acquisition notifications to accelerate highway infrastructure development projects in India. It intends to expedite the process of land acquisition for National Highways. Hence statement 2 is correct. This portal would enhance the efficiency of the land acquisition process, ensure transparency and accountability, and result in the e-transfer of benefits directly to the accounts of the beneficiaries.   Incorrect Solution (c) The Bhoomi Rashi Portal is an e-governance initiative of the Ministry of Road Transport and Highways. Hence statement 1 is correct. It acts as a single-point platform for the online processing of land acquisition notifications to accelerate highway infrastructure development projects in India. It intends to expedite the process of land acquisition for National Highways. Hence statement 2 is correct. This portal would enhance the efficiency of the land acquisition process, ensure transparency and accountability, and result in the e-transfer of benefits directly to the accounts of the beneficiaries.   Question 29 of 35 29. Question Consider the following statements about Divya Kala Mela: It is a unique event showcasing the products and craftsmanship of Divyang entrepreneurs and artisans from across the country. It is organised by the Ministry of Culture through the National Divyangjan Finance and Development Corporation (NDFDC). Choose the correct code: a) 1 only b) 2 only c) Both 1 and 2 d) Neither 1 nor 2 Correct Solution (a) Divya Kala Mela is a unique event showcasing the products and craftsmanship of Divyang entrepreneurs and artisans from across the country. Hence statement 1 is correct. It aims to strengthen the economic empowerment of Divyangjan and provide a bigger platform for marketing and showcasing the products and skills of Divyangjan. It is organised by the Department of Empowerment of Persons with Disabilities (Divyangjan), Ministry of Social Justice and Empowerment (MoSJ&E) through the National Divyangjan Finance and Development Corporation (NDFDC). Hence statement 2 is incorrect. Incorrect Solution (a) Divya Kala Mela is a unique event showcasing the products and craftsmanship of Divyang entrepreneurs and artisans from across the country. Hence statement 1 is correct. It aims to strengthen the economic empowerment of Divyangjan and provide a bigger platform for marketing and showcasing the products and skills of Divyangjan. It is organised by the Department of Empowerment of Persons with Disabilities (Divyangjan), Ministry of Social Justice and Empowerment (MoSJ&E) through the National Divyangjan Finance and Development Corporation (NDFDC). Hence statement 2 is incorrect. Question 30 of 35 30. Question Consider the following statements about Kanheri Caves:  The name Kanheri occurs in the Nasik inscription of the Satavahana ruler Vasisthiputra Pulumavi. The earliest reference of Kanheri is ascribed to Megasthenes who visited India during the reign of Chandragupta Maurya. They are located within the forests of the Sanjay Gandhi National Park. How many of the above statements are correct? a) Only one b) Only two c) All three d) None Correct Solution (b) Kanheri Caves are a group of caves and rock-cut monuments located on the western outskirts of Mumbai. The name Kanheri is derived from ‘Kanhagiri’ in Prakrit. The name Kanheri occurs in the Nasik inscription of the Satavahana ruler Vasisthiputra Pulumavi. Hence statement 1 is correct. The Kanheri caves comprise more than 110 different rock-cut monolithic excavations and are one of the largest single excavations in the country. The earliest reference to Kanheri is ascribed to Fa-Hein who visited India during 399-411 CE. Hence statement 2 is incorrect. Kanheri Caves flourished under the patronage of Satavahana, Traikutakas, Vakatakas, and Silaharas. They are located within the forests of the Sanjay Gandhi National Park. Hence statement 3 is correct. Note: Grey Francolin (Ortygornis pondicerianus) was reported inside the Sanjay Gandhi National Park (SGNP) of Mumbai. Grey Francolin is a ground-dwelling bird known for its distinctive calls and cryptic plumage. They are carnivores (insectivores) and herbivores (granivores). Incorrect Solution (b) Kanheri Caves are a group of caves and rock-cut monuments located on the western outskirts of Mumbai. The name Kanheri is derived from ‘Kanhagiri’ in Prakrit. The name Kanheri occurs in the Nasik inscription of the Satavahana ruler Vasisthiputra Pulumavi. Hence statement 1 is correct. The Kanheri caves comprise more than 110 different rock-cut monolithic excavations and are one of the largest single excavations in the country. The earliest reference to Kanheri is ascribed to Fa-Hein who visited India during 399-411 CE. Hence statement 2 is incorrect. Kanheri Caves flourished under the patronage of Satavahana, Traikutakas, Vakatakas, and Silaharas. They are located within the forests of the Sanjay Gandhi National Park. Hence statement 3 is correct. Note: Grey Francolin (Ortygornis pondicerianus) was reported inside the Sanjay Gandhi National Park (SGNP) of Mumbai. Grey Francolin is a ground-dwelling bird known for its distinctive calls and cryptic plumage. They are carnivores (insectivores) and herbivores (granivores). Question 31 of 35 31. Question (1/9), (1/3), 1, ?, 9. What number should fill the Question mark? a) (2/3) b) 3 c) 3/9 d) 6 Correct Solution (b) This is a multiplication series; each number is 3 times the previous number. Incorrect Solution (b) This is a multiplication series; each number is 3 times the previous number. Question 32 of 35 32. Question The difference of two numbers is 1365. On dividing the larger number by the smaller, we get 6 as quotient and the 15 as remainder. What is the smaller number? a) 270 b) 295 c) 240 d) 360 Correct Solution (a) Let the smaller number be x Then larger number = (x + 1365) ∴ x + 1365 = 6x + 15 ⇒ 5x = 1350 ⇒ x = 270 ∴ Smaller number = 270 Incorrect Solution (a) Let the smaller number be x Then larger number = (x + 1365) ∴ x + 1365 = 6x + 15 ⇒ 5x = 1350 ⇒ x = 270 ∴ Smaller number = 270 Question 33 of 35 33. Question In a race of 200m, B can give a start of 10m to A and C can give a start of 20m to B. The start that C can give to A in the same race is? a) 27m b) 30m c) 25m d) 29m Correct Solution (d) B : A = 200 : 190 C : B = 200 : 180 = C/A = (C/B)*(B/A) = (200/180)*(200/190) = 200/171 ∴ C can give to A, a start of (200 – 171) m = 29 m Incorrect Solution (d) B : A = 200 : 190 C : B = 200 : 180 = C/A = (C/B)*(B/A) = (200/180)*(200/190) = 200/171 ∴ C can give to A, a start of (200 – 171) m = 29 m Question 34 of 35 34. Question Find the HCF of (3125-1) and (335-1) a) 312-1 b) 35-1 c) 34-1 d) None of these Correct Solution (b) The solution of this question is based on the rule, The HCF of (am – 1) and (an – 1) is given by (aHCF of m, n – 1) Thus for this question the answer is (35 – 1) Since, 5 is the HCF of 35 and 125. Incorrect Solution (b) The solution of this question is based on the rule, The HCF of (am – 1) and (an – 1) is given by (aHCF of m, n – 1) Thus for this question the answer is (35 – 1) Since, 5 is the HCF of 35 and 125. Question 35 of 35 35. Question Prof. Suman takes a number of quizzes for a course. All the quizzes are out of 100. A student can get an A grade in the course if the average of her scores is more than or equal to 90. Grade B is awarded to a student if the average of her scores is between 87 and 89 (both included). If the average is below 87, the student gets a C grade. Ramesh is preparing for his last quiz, and he realizes that he must score a minimum of 97 to get an A grade. After the quiz, he realizes that he will score 70, and he will just manage a B. How many quizzes did Prof. Suman take? a) 6 b) 7 c) 8 d) 9 Correct Solution (d) Let the number of quizzes before the last quiz be x. Let the average score of Ramesh be n. According to the question, ((nx+97)/x) = 9090x – nx = 97…….(1) And, ((nx+70)/x) = 8787x – nx = 70……..(2) Subtract both equations 3x = 27x = 9 Hence, the number of quizzes is 9. Incorrect Solution (d) Let the number of quizzes before the last quiz be x. Let the average score of Ramesh be n. According to the question, ((nx+97)/x) = 9090x – nx = 97…….(1) And, ((nx+70)/x) = 8787x – nx = 70……..(2) Subtract both equations 3x = 27x = 9 Hence, the number of quizzes is 9. window.wpProQuizInitList = window.wpProQuizInitList || []; window.wpProQuizInitList.push({ id: '#wpProQuiz_3631', init: { quizId: 3631, mode: 1, globalPoints: 70, timelimit: 1800, resultsGrade: [0], bo: 704, qpp: 0, catPoints: [70], formPos: 0, lbn: "Test-summary", json: {"32097":{"type":"single","id":32097,"catId":0,"points":2,"correct":[0,0,1,0]},"32098":{"type":"single","id":32098,"catId":0,"points":2,"correct":[0,0,1,0]},"32099":{"type":"single","id":32099,"catId":0,"points":2,"correct":[0,1,0,0]},"32102":{"type":"single","id":32102,"catId":0,"points":2,"correct":[0,0,1,0]},"32104":{"type":"single","id":32104,"catId":0,"points":2,"correct":[0,0,0,1]},"32105":{"type":"single","id":32105,"catId":0,"points":2,"correct":[1,0,0,0]},"32106":{"type":"single","id":32106,"catId":0,"points":2,"correct":[1,0,0,0]},"32109":{"type":"single","id":32109,"catId":0,"points":2,"correct":[1,0,0,0]},"32111":{"type":"single","id":32111,"catId":0,"points":2,"correct":[1,0,0,0]},"32113":{"type":"single","id":32113,"catId":0,"points":2,"correct":[1,0,0,0]},"32114":{"type":"single","id":32114,"catId":0,"points":2,"correct":[0,1,0,0]},"32115":{"type":"single","id":32115,"catId":0,"points":2,"correct":[1,0,0,0]},"32116":{"type":"single","id":32116,"catId":0,"points":2,"correct":[0,0,1,0]},"32118":{"type":"single","id":32118,"catId":0,"points":2,"correct":[0,0,0,1]},"32121":{"type":"single","id":32121,"catId":0,"points":2,"correct":[0,1,0,0]},"32123":{"type":"single","id":32123,"catId":0,"points":2,"correct":[1,0,0,0]},"32124":{"type":"single","id":32124,"catId":0,"points":2,"correct":[0,0,1,0]},"32127":{"type":"single","id":32127,"catId":0,"points":2,"correct":[1,0,0,0]},"32128":{"type":"single","id":32128,"catId":0,"points":2,"correct":[0,0,1,0]},"32130":{"type":"single","id":32130,"catId":0,"points":2,"correct":[0,1,0,0]},"32132":{"type":"single","id":32132,"catId":0,"points":2,"correct":[0,0,1,0]},"32134":{"type":"single","id":32134,"catId":0,"points":2,"correct":[1,0,0,0]},"32137":{"type":"single","id":32137,"catId":0,"points":2,"correct":[0,1,0,0]},"32138":{"type":"single","id":32138,"catId":0,"points":2,"correct":[0,1,0,0]},"32139":{"type":"single","id":32139,"catId":0,"points":2,"correct":[0,0,1,0]},"32141":{"type":"single","id":32141,"catId":0,"points":2,"correct":[0,1,0,0]},"32142":{"type":"single","id":32142,"catId":0,"points":2,"correct":[0,1,0,0]},"32145":{"type":"single","id":32145,"catId":0,"points":2,"correct":[0,0,1,0]},"32148":{"type":"single","id":32148,"catId":0,"points":2,"correct":[1,0,0,0]},"32151":{"type":"single","id":32151,"catId":0,"points":2,"correct":[0,1,0,0]},"32154":{"type":"single","id":32154,"catId":0,"points":2,"correct":[0,1,0,0]},"32156":{"type":"single","id":32156,"catId":0,"points":2,"correct":[1,0,0,0]},"32157":{"type":"single","id":32157,"catId":0,"points":2,"correct":[0,0,0,1]},"32160":{"type":"single","id":32160,"catId":0,"points":2,"correct":[0,1,0,0]},"32162":{"type":"single","id":32162,"catId":0,"points":2,"correct":[0,0,0,1]}} } }); All the Best IASbaba

Daily Prelims CA Quiz

UPSC Quiz – 2024 : IASbaba’s Daily Current Affairs Quiz 12th April 2024

For Previous Daily Quiz (ARCHIVES) – CLICK HERE The Current Affairs questions are based on sources like ‘The Hindu’, ‘Indian Express’ and ‘PIB’, which are very important sources for UPSC Prelims Exam. The questions are focused on both the concepts and facts. The topics covered here are generally different from what is being covered under ‘Daily Current Affairs/Daily News Analysis (DNA) and Daily Static Quiz’ to avoid duplication. The questions would be published from Monday to Saturday before 2 PM. One should not spend more than 10 minutes on this initiative. Gear up and Make the Best Use of this initiative. Do remember that, “the difference between Ordinary and EXTRA-Ordinary is PRACTICE!!” Important Note: Don’t forget to post your marks in the comment section. Also, let us know if you enjoyed today’s test 🙂After completing the 5 questions, click on ‘View Questions’ to check your score, time taken, and solutions.To take the Test Click Here

DAILY CURRENT AFFAIRS IAS | UPSC Prelims and Mains Exam – 11th April 2024

Archives (PRELIMS & MAINS Focus)   Hepatitis Syllabus Prelims – Science Context: India is one of the countries with the highest burden of viral hepatitis — with 2.9 crore people living with Hepatitis B infection and 0.55 crore living with Hepatitis C infection, according to the Global Hepatitis Report 2024 released by the World Health Organisation (WHO) Background:- There were over 50,000 new Hepatitis B cases and 1.4 lakh new Hepatitis C cases reported in 2022. And these infections killed 1.23 lakh people in India in 2022 as per the report. About Hepatitis: Hepatitis means inflammation of the liver. The liver is a vital organ that processes nutrients, filters the blood, and fights infections. When the liver is inflamed or damaged, its function can be affected. Heavy alcohol use, toxins, some medications, and certain medical conditions can cause hepatitis. However, hepatitis is often caused by a virus. There are five main strains of the hepatitis virus, referred to as types A, B, C, D and E. While they all cause liver disease, they differ in important ways including modes of transmission, severity of the illness, geographical distribution and prevention methods. Hepatitis B (HBV) is known to cause acute infection with nausea, vomitting and yellowing of the eye and skin for several weeks. Liver failure happens in severe cases. But the challenge is the chronic, life-long liver disease that it causes, especially when children get it. The chronic infection can lead to scarring of the liver called cirrhosis and increase the risk of liver cancer. Many people with the hepatitis C (HCV) virus don’t have symptoms or know they are infected. Those who do develop symptoms two to 12 weeks after exposure report yellow skin or eyes, loss of appetite, nausea, stomach ache, fever, dark urine, light-coloured stool, joint pain and exhaustion. Both infections are transmitted from mother to child during delivery, during transfusion of blood that hasn’t been screened properly, during contact with the blood of an infected person or while sharing needles by drug users.Both viruses can also be transmitted sexually. Since hepatitis B can be prevented through vaccination, the report highlights the need to ensure coverage. Hepatitis C is curable with medicines. Source: Indian Express Previous Year Question Q.1) Which of the following statements is not correct? (UPSC CSE 2015) Hepatitis B virus is transmitted much like HIV. Hepatitis B, unlike Hepatitis C, does not have a vaccine. Globally, the number of people infected with Hepatitis B and C viruses are several times more than those infected with HIV. Some of those infected with Hepatitis B and C viruses do not show the symptoms for many years. Lavender Cultivation Syllabus Prelims & Mains – Agriculture Context:When lavender cultivation was first introduced to Bhaderwah subdivision in Jammu and Kashmir’s Doda district in 2015, most farmers were sceptical and only a few adopted it. Today, these early adopters have significantly expanded cultivation, employ 30-40 people each on a full-time basis, have their own distillation units for making lavender oil, and market their products in different parts of the country. Background: The CSIR (Council Of Scientific and Industrial Research) – Aroma Mission is a flagship project of CSIR under which Lavender cultivation is being promoted in the temperate regions of J&K.The aim of the project is to increase the income of small and marginal farmers and develop agriculture-based Startups.Besides lavender, the Aroma Mission aims to inject scientific research and development into the cultivation of aromatic plants such as rose, lemon grass, rosemary and wild marigold. About Lavender Cultivation: Lavender, which can be grown in areas that experience snowfall and have a hilly terrain, blossoms and is ready for harvesting two-and-a-half years after it is planted. The same plant can bear flowers for 18-20 years, and does not require the amount of insecticides, pesticides, and other chemicals used on conventional crops like maize and paddy. The uses of lavender includes manufacturing perfumes, soaps, agarbatti, and room fresheners.Lavender oil has a calming effect and is a stress buster. CSIR developed an elite variety of lavender suitable for cultivation in temperate regions of J&K and provided free saplings and end-to-end agro-technologies to farmers and also installed distillation units for essential oil extraction in several regions of J&K. The success of Lavender cultivation in J&K earned it the sobriquet, ‘Purple Revolution’. The variety of Lavender is highly suitable for cultivation in the rainfed temperate regions of India. The crop was officially introduced in Bhaderwah by the Council Of Scientific and Industrial Research-Indian Institute of Integrative Medicine (CSIR-IIIM), Jammu, in 2015. According to officials, more than 700 acres of farmland has been brought under lavender cultivation in the area since 2017, and another 100 acres is set to be added. CSIR plans to distribute lavender plants to farmers in parts of Kathua, Rajouri, Poonch, and the Kashmir Valley, besides Uttrakhand, Himachal Pradesh and the Northeast. Source: Indian Express CREDIT-DEPOSIT RATIO Syllabus Prelims – Economy Context: Data from the Reserve Bank of India (RBI) reveals that the credit-deposit ratio has reached its highest level in at least 20 years. Background: This surge in the credit-deposit ratio indicates that more of the banks’ deposit base is being utilized for loans, including home loans and other consumption-related loans. About CREDIT-DEPOSIT RATIO The credit-deposit ratio is a financial metric that measures the proportion of a bank’s deposit base that is used for lending. It is calculated by dividing the total loans extended by the bank by its total deposits. A higher ratio indicates that a larger portion of the bank’s deposits is being lent out as credit, while a lower ratio suggests more deposits are being held in reserve or invested in other assets. For instance, if a bank has a credit-deposit ratio of 80%, it means that 80% of its deposits are being used to provide loans. This ratio is crucial for assessing a bank’s liquidity and lending capacity. Source: Livemint GULF COOPERATION COUNCIL (GCC) Syllabus Prelims – Current Event Context: Recently, for the first time since its inception, the Gulf Cooperation Council (GCC) announced its ‘Vision for Regional Security’ in Riyadh. Background: The GCC plays a significant role in regional affairs, promoting cooperation across various domains. About GULF COOPERATION COUNCIL (GCC) The Gulf Cooperation Council (GCC), also known as the Cooperation Council for the Arab States of the Gulf, is a regional, intergovernmental, political, and economic union. GCC comprising six countries in the Arabian Peninsula: Bahrain, Kuwait, Oman, Qatar, Saudi Arabia, United Arab Emirates. The GCC was established in 1981 with a formal charter. The charter was signed in Abu Dhabi City, United Arab Emirates, on 25 May 1981 (corresponding to 21 Rajab 1401 on the Islamic calendar). The GCC aims to foster economic, scientific, and business cooperation among its member states. It serves as a platform for regional collaboration and coordination. The main headquarters of the GCC is located in Riyadh, the capital of Saudi Arabia. The Peninsula Shield Force serves as the military arm of the GCC, formed in 1984. Source: Arab News TSAT-1A SATELLITE Syllabus Prelims – Science Context: Tata Advanced Systems Limited (TASL) has achieved a remarkable milestone by successfully deploying the TSAT-1A satellite into space. Background: This groundbreaking achievement resulted from a strategic collaboration between TASL and Satellogic Inc., a leader in sub-meter resolution Earth Observation (EO) data collection. About TSAT-1A SATELLITE: TSAT-1A is the first military spy satellite initiative in India’s private sector. It is an Optical sub-metre-resolution Earth observation satellite. It is Built by Tata Advanced Systems Limited (TASL) in collaboration with Satellogic Inc. SpaceX’s Falcon 9 rocket lauched it from Kennedy Space Center, Florida. Capabilities: Military-grade imagery with sub-meter resolution. Multispectral and hyperspectral imaging for detailed analysis of land, water, and natural resources. Applications: Used by Indian defence forces for intelligence gathering. Information sharing with friendly nations to enhance preparedness and strategic decision-making. Source: Hindustan Times Internet Shutdown in India Syllabus Mains – GS 2 Context: India has topped the global list of countries imposing internet bans, with about 60% of all blackouts recorded in the world, between 2016 and 2022 having been in India. Background: Freedom in cyberspace hinges on a freely accessible, functional, and affordable internet. The extent of the freedom can be measured based on availability of mobile and broadband services, internet speed, and access to websites and social media platforms. Provisions related to Internet Shutdown: Section 5(2) of the Indian Telegraph Act, 1885, read with Temporary Suspension of Telecom Services (Public Emergency and Public Safety) Rules, 2017 allows the union or state home secretary to order the suspension of any telegraph service (including internet) in case of public emergency or public safety. Such an order must be reviewed by a committee within five days and cannot last for more than 15 days. In an urgent situation, an officer of the joint secretary level or above, authorized by the union or state home secretary, can issue the order. Section 144 of the Code of Criminal Procedure empowers a district magistrate, a sub-divisional magistrate or any other executive magistrate specially empowered by the state government to issue orders to prevent or stop any nuisance or disturbance of public tranquility. Such orders can include the suspension of internet services in a particular area for a specified period. Significance/Arguments supporting Internet Shutdown: Internet shutdowns can help to prevent the spread of hate speech and fake news that could incite violence and riots.  For example, the government announced the internet shutdown in Delhi NCR following the farmers’ protest on Republic Day to combat misinformation and maintain law and order. Internet shutdowns can help curb the organization and mobilization of protests that could disrupt public order and security.  For example, the government imposed internet shutdowns in Kashmir and other parts of the country after the abrogation of Article 370 to prevent any anti-national activities and separatist movements. Internet shutdowns can help protect national security and sovereignty from external threats and cyberattacks.  For example, the government suspended internet services in some border areas during the standoff with China to prevent any espionage or sabotage. Internet shutdowns can help control the distribution and consumption of content that could be harmful or offensive to certain groups or individuals.  For example, the government blocks internet access in some regions to prevent the circulation of objectionable images or videos. Issues/Concerns related to Internet Shutdown: Internet shutdowns violate Fundamental Rights under Article 19(1)(a) and Article 19(1)(g). The freedom of speech and expression and freedom to practice any profession over the medium of the internet enjoy constitutional protection under Article 19(1)(a) and Article 19(1)(g) – the Supreme Court in Anuradha Bhasin vs Union of India case (2020). Internet Shutdown violates the Right to Information which has been declared as a Fundamental Right under Article 19 by the Supreme Court in the Raj Narain vs State of UP (1975) case. Internet Shutdowns violate the Right to Internet which was declared a Fundamental Right under Article 21 by the Kerala High Court in the Faheema Shirin v. State of Kerala case. Internet Shutdowns are often implemented without clear legal frameworks or oversight mechanisms, leading to arbitrary and disproportionate restrictions on internet access.  The absence of accountability mechanisms exacerbates the risk of abuse of power by authorities, who may impose shutdowns without adequate justification or recourse for affected individuals. Beyond the immediate social and political implications, internet shutdowns have significant economic repercussions. Disruptions to online commerce, communication, and financial transactions disrupt business operations, hinder economic growth, and deter investment, ultimately undermining long-term development objectives. According to Top10VPN, India lost Rs 2,091 crore ($255.2 million) in the first half of 2023 because of internet shutdowns. Internet Shutdowns disrupt the fabric of society by impeding communication networks, hindering access to vital services, and isolating individuals from their communities. Social cohesion may suffer as a result, as people are unable to connect, organize, or mobilize effectively, leading to feelings of isolation and alienation. Source: Hindu Practice MCQs Daily Practice MCQs Q1.) With reference to TSAT-1A satellite, consider the following statements: TSAT-1A is the first military spy satellite initiative in India’s private sector. It is an Earth observation satellite. It is Built by Tata Advanced Systems Limited. SpaceX’s Falcon 9 rocket lauched it from Kennedy Space Center, Florida. How many of the statements given above are correct? Only one Only two Only three All four Q2.)Consider the following countries: Iran Iraq Kuwait Oman Qatar Saudi Arabia How many of the above-mentioned countries are the members of the Gulf Cooperation Council (GCC)? Only two Only three Only four Only five Q3.) With reference to the credit-deposit ratio, consider the following statements: The credit-deposit ratio is a financial metric that measures the proportion of a bank’s deposit base that is used for lending. A higher ratio indicates that a larger portion of the bank’s deposits is being lent out as credit. Which of the statements given above is/ are correct? 1 only 2 only Both 1 and 2 Neither 1 nor 2 Comment the answers to the above questions in the comment section below!! ANSWERS FOR ’  11th April  2024 – Daily Practice MCQs’ will be updated along with tomorrow’s Daily Current Affairs.st ANSWERS FOR  4th April – Daily Practice MCQs Answers- Daily Practice MCQs Q.1) –  c Q.2) – c Q.3) – b

Daily Prelims CA Quiz

UPSC Quiz – 2024 : IASbaba’s Daily Current Affairs Quiz 11th April 2024

For Previous Daily Quiz (ARCHIVES) – CLICK HERE The Current Affairs questions are based on sources like ‘The Hindu’, ‘Indian Express’ and ‘PIB’, which are very important sources for UPSC Prelims Exam. The questions are focused on both the concepts and facts. The topics covered here are generally different from what is being covered under ‘Daily Current Affairs/Daily News Analysis (DNA) and Daily Static Quiz’ to avoid duplication. The questions would be published from Monday to Saturday before 2 PM. One should not spend more than 10 minutes on this initiative. Gear up and Make the Best Use of this initiative. Do remember that, “the difference between Ordinary and EXTRA-Ordinary is PRACTICE!!” Important Note: Don’t forget to post your marks in the comment section. Also, let us know if you enjoyed today’s test 🙂After completing the 5 questions, click on ‘View Questions’ to check your score, time taken, and solutions. To take the Test Click Here

[DAY 33] 60 DAY RAPID REVISION (RaRe) SERIES for UPSC Prelims 2024 – ECONOMY, CURRENT AFFAIRS & CSAT TEST SERIES!

Archives Hello Friends The 60 Days Rapid Revision (RaRe) Series is IASbaba’s Flagship Initiative recommended by Toppers and loved by the aspirants’ community every year. It is the most comprehensive program which will help you complete the syllabus, revise and practice tests on a daily basis. The Programme on a daily basis includes Daily Prelims MCQs from Static (Monday – Saturday) Daily Static Quiz will cover all the topics of static subjects – Polity, History, Geography, Economics, Environment and Science and technology. 20 questions will be posted daily and these questions are framed from the topics mentioned in the schedule. It will ensure timely and streamlined revision of your static subjects. Daily Current Affairs MCQs (Monday – Saturday) Daily 5 Current Affairs questions, based on sources like ‘The Hindu’, ‘Indian Express’ and ‘PIB’, would be published from Monday to Saturday according to the schedule. Daily CSAT Quiz (Monday – Friday) CSAT has been an Achilles heel for many aspirants. Daily 5 CSAT Questions will be published. Note – Daily Test of 20 static questions, 10 current affairs, and 5 CSAT questions. (35 Prelims Questions) in QUIZ FORMAT will be updated on a daily basis. To Know More about 60 Days Rapid Revision (RaRe) Series – CLICK HERE   60 Day Rapid Revision (RaRe) Series Schedule – CLICK HERE  Important Note Comment your Scores in the Comment Section. This will keep you accountable, responsible and sincere in days to come. It will help us come out with the Cut-Off on a Daily Basis. Let us know if you enjoyed today’s test 🙂  You can post your comments in the given format  (1) Your Score (2) Matrix Meter (3) New Learning from the Test Time limit: 0 Test-summary 0 of 35 questions completed Questions: 1 2 3 4 5 6 7 8 9 10 11 12 13 14 15 16 17 18 19 20 21 22 23 24 25 26 27 28 29 30 31 32 33 34 35 Information The following Test is based on the syllabus of 60 Days Plan-2023 for UPSC IAS Prelims 2022. To view Solutions, follow these instructions: Click on – ‘Start Test’ button Solve Questions Click on ‘Test Summary’ button Click on ‘Finish Test’ button Now click on ‘View Questions’ button – here you will see solutions and links. You have already completed the test before. Hence you can not start it again. Test is loading... You must sign in or sign up to start the test. You have to finish following test, to start this test: Results 0 of 35 questions answered correctly Your time: Time has elapsed You have scored 0 points out of 0 points, (0) Average score     Your score     Categories Not categorized 0% Your result has been entered into leaderboard Loading Name: E-Mail: Captcha: maximum of 70 points Pos. Name Entered on Points Result Table is loading No data available 1 2 3 4 5 6 7 8 9 10 11 12 13 14 15 16 17 18 19 20 21 22 23 24 25 26 27 28 29 30 31 32 33 34 35 Answered Review Question 1 of 35 1. Question Consider the following statements regarding Treasury Bills (T-Bills): They provide long-term capital to the government. They satisfy the CRR and SLR requirements of the banking institutions that invest in them. Which of the statements given above is/are correct? a) 1 only b) 2 only c) Both 1 and 2 d) Neither 1 nor 2 Correct Solution (b) Statement 1 Statement 2 Incorrect Correct Started in 1986, they are instruments of money market used by the Central Government to fulfil its short- term liquidity requirement unto the period of 364 days.   The TBS other than providing short-term cushion to the government, it also functions as short-term investment avenues for the banks and financial institutions, besides functioning as requirements of the CRR and SLR of the banking institutions. Incorrect Solution (b) Statement 1 Statement 2 Incorrect Correct Started in 1986, they are instruments of money market used by the Central Government to fulfil its short- term liquidity requirement unto the period of 364 days.   The TBS other than providing short-term cushion to the government, it also functions as short-term investment avenues for the banks and financial institutions, besides functioning as requirements of the CRR and SLR of the banking institutions. Question 2 of 35 2. Question With reference to ‘Chit Funds’, consider the following statements: Registration and regulation of Chit funds are carried out by State Governments. Chit funds are included in the definition of Non- Banking Financial Companies by RBI. Which of the statements given above is/are correct? a) 1 only b) 2 only c) Both 1 and 2 d) Neither 1 nor 2 Correct Solution (c) Statement 1 Statement 2 Correct Correct Central Government has not framed any Rules of operation for them. Thus, Registration and Regulation of Chit funds are carried out by State Governments under the Rules framed by them.   Functionally, Chit funds are included in the definition of Non- Banking Financial Companies by RBI under the sub-head miscellaneous non-banking company (MNBC). But RBI has not laid out any separate regulatory framework for them. Incorrect Solution (c) Statement 1 Statement 2 Correct Correct Central Government has not framed any Rules of operation for them. Thus, Registration and Regulation of Chit funds are carried out by State Governments under the Rules framed by them.   Functionally, Chit funds are included in the definition of Non- Banking Financial Companies by RBI under the sub-head miscellaneous non-banking company (MNBC). But RBI has not laid out any separate regulatory framework for them. Question 3 of 35 3. Question With reference to Venture Capital Fund (VCF), consider the following statements: Venture Capital Funds ensure that the money of the investors is used to fund projects which have a potential to grow. They are regulated by the guidelines issued by the Reserve Bank of India (RBI). Which of the statements given above is/are correct? a) 1 only b) 2 only c) Both 1 and 2 d) Neither 1 nor 2 Correct Solution (a) Statement 1 Statement 2 Correct Incorrect Venture Capital Funds ensure that the money of the investors is used to fund projects which have a potential to grow and the money provided in the process is known as Venture Capital. They are institutions that are dedicated to funding new ventures and are regulated by the guidelines issued by the Securities and Exchange Board of India (SEBI).   Incorrect Solution (a) Statement 1 Statement 2 Correct Incorrect Venture Capital Funds ensure that the money of the investors is used to fund projects which have a potential to grow and the money provided in the process is known as Venture Capital. They are institutions that are dedicated to funding new ventures and are regulated by the guidelines issued by the Securities and Exchange Board of India (SEBI).   Question 4 of 35 4. Question With reference to Alternative Investment Funds, consider the following statements: It refers to any privately pooled investment fund from domestic sources only. Currently they do not come under the jurisdiction of any regulatory agency in India. Choose the correct code: a) 1 only b) 2 only c) Both 1 and 2 d) Neither 1 nor 2 Correct Solution (d) Statement 1 Statement 2 Incorrect Incorrect It refers to any privately pooled investment fund, (whether from Indian or foreign sources), in the form of a trust or a company or a body corporate or a Limited Liability Partnership (LLP) which are not presently covered by any Regulation of SEBI governing fund management (like, Regulations governing Mutual Fund or Collective Investment Scheme) nor coming under the direct regulation of any other sectoral regulators in India-IRDA, PFRDA, RBI. In India, Alternative Investment Funds (AIFs) are defined in Regulation 2(1)(b) of Securities and Exchange Board of India (Alternative Investment Funds) Regulations, 2012. Incorrect Solution (d) Statement 1 Statement 2 Incorrect Incorrect It refers to any privately pooled investment fund, (whether from Indian or foreign sources), in the form of a trust or a company or a body corporate or a Limited Liability Partnership (LLP) which are not presently covered by any Regulation of SEBI governing fund management (like, Regulations governing Mutual Fund or Collective Investment Scheme) nor coming under the direct regulation of any other sectoral regulators in India-IRDA, PFRDA, RBI. In India, Alternative Investment Funds (AIFs) are defined in Regulation 2(1)(b) of Securities and Exchange Board of India (Alternative Investment Funds) Regulations, 2012. Question 5 of 35 5. Question Which of the following financial instruments can be classified as securities? Bonds Shares Fixed Deposits Choose the correct code: a) Only one b) Only two c) All three d) None Correct Solution (b) Statement 1 Statement 2 Statement 3 Correct Correct Incorrect Financial instruments which are tradable or that can be bought and sold are generally referred to as securities. For e.g., shares, bonds etc. A government bond is a debt instrument issued by the Central and State Governments of India. Issuance of such bonds occur when the issuing body (Central or State governments) faces a liquidity crisis and requires funds for the purpose of infrastructure development. The key aspect is that it should be transferable. For instance, In India, the word “securities” is defined in clause (h) of section 2 of the Securities Contracts (Regulation) Act, 1956 (SCRA). The term securities include the following in India: shares, scrips, stocks, bonds, debentures, debenture stock or other marketable securities of a like nature in or of any incorporated company or other body corporates derivatives Fixed deposits are low-risk investments as they aren’t dependent on market fluctuations. Therefore, these are not included in securities. Incorrect Solution (b) Statement 1 Statement 2 Statement 3 Correct Correct Incorrect Financial instruments which are tradable or that can be bought and sold are generally referred to as securities. For e.g., shares, bonds etc. A government bond is a debt instrument issued by the Central and State Governments of India. Issuance of such bonds occur when the issuing body (Central or State governments) faces a liquidity crisis and requires funds for the purpose of infrastructure development. The key aspect is that it should be transferable. For instance, In India, the word “securities” is defined in clause (h) of section 2 of the Securities Contracts (Regulation) Act, 1956 (SCRA). The term securities include the following in India: shares, scrips, stocks, bonds, debentures, debenture stock or other marketable securities of a like nature in or of any incorporated company or other body corporates derivatives Fixed deposits are low-risk investments as they aren’t dependent on market fluctuations. Therefore, these are not included in securities. Question 6 of 35 6. Question With reference to ‘Capital Gains Tax’, which of the following statements is/are correct? It is levied on long term gains only. Capital gains tax in India need not be paid in case the individual inherits the property and there is no sale. Which of the statements given above is/are correct? a) 1 only b) 2 only c) Both 1 and 2 d) Neither 1 nor 2 Correct Solution (b) Statement 1 Statement 2 Incorrect Correct Capital gain can be defined as any profit that is received through the sale of a capital asset. The profit that is received falls under the income category. Therefore, a tax needs to be paid on the income that is received. The tax that is paid is called capital gains tax and it can either be long term or short term. Under the Income Tax Act, capital gains tax in India need not be paid in case the individual inherits the property and there is no sale. However, if the person who has inherited the property decides to sell it, tax will have to be paid on the income that has been generated from the sale. Incorrect Solution (b) Statement 1 Statement 2 Incorrect Correct Capital gain can be defined as any profit that is received through the sale of a capital asset. The profit that is received falls under the income category. Therefore, a tax needs to be paid on the income that is received. The tax that is paid is called capital gains tax and it can either be long term or short term. Under the Income Tax Act, capital gains tax in India need not be paid in case the individual inherits the property and there is no sale. However, if the person who has inherited the property decides to sell it, tax will have to be paid on the income that has been generated from the sale. Question 7 of 35 7. Question With reference to debentures, consider the following statements: Debenture is a movable property. The debenture holders have the right to vote in meetings of the company. Select the correct answer using the code given below: a) 1 only b) 2 only c) Both 1 and 2 d) Neither 1 nor 2 Correct Solution (a) Statement 1 Statement 2 Correct Incorrect Denture is a movable property. It is in the form of a certificate of indebtedness of the company and issued by the company itself. It generally creates a charge on the undertaking or undertakings of the company. As the debenture holders are not the owner of the company so they are not entitled with the administration and management of the company. The debenture holders cannot claim the privilege to vote in any meeting of the company. Incorrect Solution (a) Statement 1 Statement 2 Correct Incorrect Denture is a movable property. It is in the form of a certificate of indebtedness of the company and issued by the company itself. It generally creates a charge on the undertaking or undertakings of the company. As the debenture holders are not the owner of the company so they are not entitled with the administration and management of the company. The debenture holders cannot claim the privilege to vote in any meeting of the company. Question 8 of 35 8. Question With reference to capital markets, consider the following statements: Capital market deals in financial instruments and commodities that are long-term securities. Secondary Market exclusively deals with the issue of new securities. Which of the statements given above is/are correct? a) 1 only b) 2 only c) Both 1 and 2 d) Neither 1 nor 2 Correct Solution (a) Statement 1 Statement 2 Correct Incorrect Capital market deals in financial instruments and commodities that are long-term securities. They have a maturity of at least more than one year.   The most important type of capital market is the primary market. It is what we call the new issue market. It exclusively deals with the issue of new securities, i.e., securities that are issued to investors for the very first time. Incorrect Solution (a) Statement 1 Statement 2 Correct Incorrect Capital market deals in financial instruments and commodities that are long-term securities. They have a maturity of at least more than one year.   The most important type of capital market is the primary market. It is what we call the new issue market. It exclusively deals with the issue of new securities, i.e., securities that are issued to investors for the very first time. Question 9 of 35 9. Question Qualified Foreign Investors (QFIs) are allowed to invest in which of the following segments of the capital market? Corporate bonds Commercial Papers Mutual funds Select the correct answer using the code given below a) 1 and 2 only b) 3 only c) 2 and 3 only d) 1, 2 and 3 Correct Solution (d) Statement 1 Statement 2 Statement 3 Correct Correct Correct The Qualified Foreign Investor (QFI) is sub-category of Foreign Portfolio Investor and refers to any foreign individuals, groups or associations, or resident, however, restricted to those from a country that is a member of Financial Action Task Force (FATF). QFIs are allowed to make investments in the Corporate bonds listed/to be listed on recognized stock exchanges G-Securities, T-Bills and Commercial Papers by opening a demat account in of the SEBI approved Qualified Depository Participant (QDP). QFIs are allowed to make investments in the Equity and Debt schemes of Indian mutual funds. Incorrect Solution (d) Statement 1 Statement 2 Statement 3 Correct Correct Correct The Qualified Foreign Investor (QFI) is sub-category of Foreign Portfolio Investor and refers to any foreign individuals, groups or associations, or resident, however, restricted to those from a country that is a member of Financial Action Task Force (FATF). QFIs are allowed to make investments in the Corporate bonds listed/to be listed on recognized stock exchanges G-Securities, T-Bills and Commercial Papers by opening a demat account in of the SEBI approved Qualified Depository Participant (QDP). QFIs are allowed to make investments in the Equity and Debt schemes of Indian mutual funds. Question 10 of 35 10. Question Which of the followings entities is/are allowed to participate in “Call Money Market”? Land Development Banks Regional Rural Banks Payment Banks Select the correct answer from the codes given below: a) 3 only b) 1 and 2 only c) 1, 2 and 3 d) None Correct Solution (a) Statement 1 Statement 2 Statement 3 Incorrect Incorrect Correct Participants in the call money market are banks and related entities specified by the RBI. Scheduled commercial banks (excluding RRBs), co-operative banks (other than Land Development Banks) and Primary  Dealers (PDs), are permitted to participate in call/notice money market both as borrowers and lenders. Financial markets broad basing is at the centre stage for the Reserve Bank of India with the inclusion of Regional Rural Banks into the money market. As per the new regulations, Payment Banks are also allowed to participate in CMM as both lenders and borrowers. Incorrect Solution (a) Statement 1 Statement 2 Statement 3 Incorrect Incorrect Correct Participants in the call money market are banks and related entities specified by the RBI. Scheduled commercial banks (excluding RRBs), co-operative banks (other than Land Development Banks) and Primary  Dealers (PDs), are permitted to participate in call/notice money market both as borrowers and lenders. Financial markets broad basing is at the centre stage for the Reserve Bank of India with the inclusion of Regional Rural Banks into the money market. As per the new regulations, Payment Banks are also allowed to participate in CMM as both lenders and borrowers. Question 11 of 35 11. Question Consider the following statements about the Foreign Portfolio Investment (FPI): It consists of securities and other financial assets held by the investors in another country. It provides the investor with direct ownership of a company’s assets. The American Depositary Receipt’s and the Global Depository Receipts form part of Foreign Portfolio Investment . It is a part of a country’s Capital Account. How many of the above statements are correct? a) Only one b) Only two c) Only three d) All four Correct Solution (c) Statement 1 Statement 2 Statement 3 Statement 4 Correct Incorrect Correct Correct The Foreign Portfolio Investment (FPI) consists of securities and other financial assets held by the investors in another country. It does not provide the investor with direct ownership of a company’s assets and is relatively liquid, depending on the volatility of the market. The FPI holdings can include stocks, ADRs (the American Depositary Receipts), GDRs (the Global Depositary Receipts), bonds, mutual funds and exchange traded funds. On a macro-level, the FPI is a part of a country’s Capital Account and is shown on its Balance of Payments (BOP). The BOP calculates the amount of money flowing from one country to other countries over a financial year. Note: Foreign Portfolio Investment (FPI): The Foreign Portfolio Investment (FPI) consists of securities and other financial assets held by the investors in another country. It does not provide the investor with direct ownership of a company’s assets and is relatively liquid, depending on the volatility of the market. Along with the Foreign Direct Investment (FDI), the FPI is one of the common ways to invest in an overseas economy. The FDI and the FPI are both important sources of funding for most economies. On a macro-level, the FPI is a part of a country’s Capital Account and is shown on its Balance of Payments (BOP). The BOP calculates the amount of money flowing from one country to other countries over a financial year. The FPI holdings can include stocks, ADRs (the American Depositary Receipts), GDRs (the Global Depositary Receipts), bonds, mutual funds and exchange traded funds. Incorrect Solution (c) Statement 1 Statement 2 Statement 3 Statement 4 Correct Incorrect Correct Correct The Foreign Portfolio Investment (FPI) consists of securities and other financial assets held by the investors in another country. It does not provide the investor with direct ownership of a company’s assets and is relatively liquid, depending on the volatility of the market. The FPI holdings can include stocks, ADRs (the American Depositary Receipts), GDRs (the Global Depositary Receipts), bonds, mutual funds and exchange traded funds. On a macro-level, the FPI is a part of a country’s Capital Account and is shown on its Balance of Payments (BOP). The BOP calculates the amount of money flowing from one country to other countries over a financial year. Note: Foreign Portfolio Investment (FPI): The Foreign Portfolio Investment (FPI) consists of securities and other financial assets held by the investors in another country. It does not provide the investor with direct ownership of a company’s assets and is relatively liquid, depending on the volatility of the market. Along with the Foreign Direct Investment (FDI), the FPI is one of the common ways to invest in an overseas economy. The FDI and the FPI are both important sources of funding for most economies. On a macro-level, the FPI is a part of a country’s Capital Account and is shown on its Balance of Payments (BOP). The BOP calculates the amount of money flowing from one country to other countries over a financial year. The FPI holdings can include stocks, ADRs (the American Depositary Receipts), GDRs (the Global Depositary Receipts), bonds, mutual funds and exchange traded funds. Question 12 of 35 12. Question Foreign exchange reserves are important for the Economies around the world for which of the following reasons? To keep the value of their currencies at a fixed rate. To maintain liquidity in case of an economic crisis. To provide confidence to the investors. To fund sectors like infrastructure. To meet external obligations. Select the correct answer using the code given below: a) 1, 2, 3 and 5 only b) 1, 2 and 4 only c) 1, 2, 3 and 4 only d) 1, 2, 3, 4 and 5 Correct Solution (d) There are seven ways the central banks use the foreign exchange reserves. First, the countries use their foreign exchange reserves to keep the value of their currencies at a fixed rate. A good example is China, which pegs the value of its currency, the yuan, to the dollar. When China stockpiles dollars, it raises the dollar value compared to that of the yuan. That makes Chinese exports cheaper than the American-made goods, increasing the sales. Second, those with a floating exchange rate system use reserves to keep the value of their currency lower than the dollar. They do this for the same reasons as those with fixed-rate systems. Even though Japan’s currency, the yen, is a floating system, the Central Bank of Japan buys U.S. Treasurys to keep its value lower than the dollar. Like China, this keeps Japan’s exports relatively cheaper, boosting trade and economic growth. Such currency trading takes place in the foreign exchange market. A third and critical function is to maintain liquidity in case of an economic crisis. For example, a flood or volcano might temporarily suspend the local exporters’ ability to produce goods. That cuts off their supply of foreign currency to pay for imports. In that case, the central bank can exchange its foreign currency for their local currency, allowing them to pay for and receive the imports. Similarly, the foreign investors will get spooked if a country has a war, military coup, or other blow to confidence. They withdraw their deposits from the country’s banks, creating a severe shortage in foreign currency. This pushes down the value of the local currency, since fewer people want it. That makes imports more expensive, creating inflation. The central bank supplies foreign currency to keep the markets steady. It also buys the local currency to support its value and prevent inflation. This reassures the foreign investors, who return to the economy. A fourth reason is to provide confidence. The central bank assures the foreign investors that it is ready to take action to protect their investments. It will also prevent a sudden flight to safety and loss of capital for the country. In that way, a strong position in the foreign currency reserves can prevent economic crises caused when an event triggers a flight to safety. Fifth, reserves are always needed to make sure a country will meet its external obligations. These include international payment obligations, including sovereign and commercial debts. They also include the financing of imports and the ability to absorb any unexpected capital movements. Sixth, some countries use their reserves to fund sectors, such as infrastructure. China, for instance, has used part of its forex reserves for recapitalizing some of its state-owned banks. Seventh, most central banks want to boost returns without compromising safety. They know the best way to do that is to diversify their portfolios. They will often hold gold and other safe, interest-bearing investments.   Incorrect Solution (d) There are seven ways the central banks use the foreign exchange reserves. First, the countries use their foreign exchange reserves to keep the value of their currencies at a fixed rate. A good example is China, which pegs the value of its currency, the yuan, to the dollar. When China stockpiles dollars, it raises the dollar value compared to that of the yuan. That makes Chinese exports cheaper than the American-made goods, increasing the sales. Second, those with a floating exchange rate system use reserves to keep the value of their currency lower than the dollar. They do this for the same reasons as those with fixed-rate systems. Even though Japan’s currency, the yen, is a floating system, the Central Bank of Japan buys U.S. Treasurys to keep its value lower than the dollar. Like China, this keeps Japan’s exports relatively cheaper, boosting trade and economic growth. Such currency trading takes place in the foreign exchange market. A third and critical function is to maintain liquidity in case of an economic crisis. For example, a flood or volcano might temporarily suspend the local exporters’ ability to produce goods. That cuts off their supply of foreign currency to pay for imports. In that case, the central bank can exchange its foreign currency for their local currency, allowing them to pay for and receive the imports. Similarly, the foreign investors will get spooked if a country has a war, military coup, or other blow to confidence. They withdraw their deposits from the country’s banks, creating a severe shortage in foreign currency. This pushes down the value of the local currency, since fewer people want it. That makes imports more expensive, creating inflation. The central bank supplies foreign currency to keep the markets steady. It also buys the local currency to support its value and prevent inflation. This reassures the foreign investors, who return to the economy. A fourth reason is to provide confidence. The central bank assures the foreign investors that it is ready to take action to protect their investments. It will also prevent a sudden flight to safety and loss of capital for the country. In that way, a strong position in the foreign currency reserves can prevent economic crises caused when an event triggers a flight to safety. Fifth, reserves are always needed to make sure a country will meet its external obligations. These include international payment obligations, including sovereign and commercial debts. They also include the financing of imports and the ability to absorb any unexpected capital movements. Sixth, some countries use their reserves to fund sectors, such as infrastructure. China, for instance, has used part of its forex reserves for recapitalizing some of its state-owned banks. Seventh, most central banks want to boost returns without compromising safety. They know the best way to do that is to diversify their portfolios. They will often hold gold and other safe, interest-bearing investments.   Question 13 of 35 13. Question Which of the following correctly describes an Escrow Account? a) A bank account through which NRIs can operate their finances in India. b) An overseas bank account used by MNCs for routing the funds through tax havens for the purpose of tax evasion. c) A third party bank account for safeguarding the seller against its buyer from the payment risk. d) A bank account which earns regular interest in return for a fixed monthly deposits. Correct Solution (c) An escrow account in simple terms is a third-party account. It is a separate bank account to hold money which belongs to others and where the money parked will be released only under the fulfilment of certain conditions of a contract. An escrow account is an arrangement for safeguarding the seller against its buyer from the payment risk for the goods or services sold by the former to the latter. This is done by removing the control over cash flows from the hands of the buyer to an independent agent. The independent agent, i.e, the holder of the escrow account would ensure that the appropriation of cash flows is as per the agreed terms and conditions between the transacting parties. Escrow account has become the standard in various transactions and business deals. In India escrow account is widely used in public private partnership projects in infrastructure. RBI has also permitted Banks (Authorized Dealer Category I) to open escrow accounts on behalf of Non-Resident corporates for acquisition / transfer of shares / convertible shares of an Indian company. Incorrect Solution (c) An escrow account in simple terms is a third-party account. It is a separate bank account to hold money which belongs to others and where the money parked will be released only under the fulfilment of certain conditions of a contract. An escrow account is an arrangement for safeguarding the seller against its buyer from the payment risk for the goods or services sold by the former to the latter. This is done by removing the control over cash flows from the hands of the buyer to an independent agent. The independent agent, i.e, the holder of the escrow account would ensure that the appropriation of cash flows is as per the agreed terms and conditions between the transacting parties. Escrow account has become the standard in various transactions and business deals. In India escrow account is widely used in public private partnership projects in infrastructure. RBI has also permitted Banks (Authorized Dealer Category I) to open escrow accounts on behalf of Non-Resident corporates for acquisition / transfer of shares / convertible shares of an Indian company. Question 14 of 35 14. Question With reference to Primary Deficit, consider the following statements: The primary deficit is the sum of Fiscal deficit and interest payments. It includes the burden of the past debt. Which of the statements given above is/are correct? a) 1 only b) 2 only c) Both 1 and 2 d) Neither 1 nor 2 Correct Solution (d) Statement 1 Statement 2 Incorrect Incorrect Gross Primary deficit is defined as gross fiscal deficit minus net interest payments. Net primary deficit, is gross primary deficit minus net domestic lending. It excludes the burden of the past debt and shows the net increase in the government’s indebtedness due to the current year’s fiscal operations. A reduction in primary deficit is reflective of government’s efforts at bridging the fiscal gap during a financial year. Incorrect Solution (d) Statement 1 Statement 2 Incorrect Incorrect Gross Primary deficit is defined as gross fiscal deficit minus net interest payments. Net primary deficit, is gross primary deficit minus net domestic lending. It excludes the burden of the past debt and shows the net increase in the government’s indebtedness due to the current year’s fiscal operations. A reduction in primary deficit is reflective of government’s efforts at bridging the fiscal gap during a financial year. Question 15 of 35 15. Question Which of the following is/are the factors determining the exchange rate of a country? Interest rates Money supply Financial stability Choose the correct code: a) Only one b) Only two c) All three d) None Correct Solution (c) Three Factors Affecting the Exchange Rates- Interest rates: The interest rate paid by a country’s central bank is a big factor. The higher interest rate makes that currency more valuable. The investors will exchange their currency for the higher-paying one. They then save it in that country’s bank to receive the higher interest rate. Money supply: The money supply that is created by the country’s central bank. If the government prints too much currency, then there is too much of it chasing too few goods. The currency holders will bid up the prices of goods and services. That creates inflation. If way too much money is printed, it causes hyperinflation. Some cash holders will invest overseas where there is no inflation, but they will find that there is not as much demand for their currency, since there is so much of it. That is why inflation can push the value of a currency down. Financial stability: Country’s economic growth and financial stability impact its currency exchange rates. If the country has a strong, growing economy, then the investors will buy its goods and services. They will need more of its currency to do so. If the financial stability looks bad, they will be less willing to invest in that country. They want to be sure they will get paid back if they hold the government bonds in that currency. Because of these factors, the demand for a country’s currency depends on what is happening in that country. Incorrect Solution (c) Three Factors Affecting the Exchange Rates- Interest rates: The interest rate paid by a country’s central bank is a big factor. The higher interest rate makes that currency more valuable. The investors will exchange their currency for the higher-paying one. They then save it in that country’s bank to receive the higher interest rate. Money supply: The money supply that is created by the country’s central bank. If the government prints too much currency, then there is too much of it chasing too few goods. The currency holders will bid up the prices of goods and services. That creates inflation. If way too much money is printed, it causes hyperinflation. Some cash holders will invest overseas where there is no inflation, but they will find that there is not as much demand for their currency, since there is so much of it. That is why inflation can push the value of a currency down. Financial stability: Country’s economic growth and financial stability impact its currency exchange rates. If the country has a strong, growing economy, then the investors will buy its goods and services. They will need more of its currency to do so. If the financial stability looks bad, they will be less willing to invest in that country. They want to be sure they will get paid back if they hold the government bonds in that currency. Because of these factors, the demand for a country’s currency depends on what is happening in that country. Question 16 of 35 16. Question Which of the following are the features of a Contestable Market? There are no barriers to entry or exit barriers. There are sunk costs involved in such markets. Both the incumbent firms and new entrants have access to the same level of technology. How many of the above statements are correct? a) Only one b) Only two c) All three d) None Correct Solution (b) Statement 1 Statement 2 Statement 3 Correct Incorrect Correct There are no barriers to entry or exit barriers is one of the features of a Contestable Market. There are no sunk costs is one of the features of a Contestable Market. Both the incumbent firms and new entrants have access to the same level of technology is one of the features of a Contestable Market.   Note: Contestable Market Theory: ‘The Contestable Market Theory‘ is an economic concept stating that the companies with a few rivals behave in a competitive manner, when the market they operate in has weak barriers to entry. ‘Contestable‘ in economics means that a company can be challenged or contested by the rival companies looking to enter the industry or the market. In other words, a Contestable Market is a market whereby the companies can enter and leave freely with low sunk costs. Sunk costs are major irrecoverable costs to enter an industry, such as the purchase of a manufacturing plant or equipment. The Contestable Market Theory assumes that even in a monopoly or oligopoly (It is a market form wherein a market or an industry is dominated by a small group of large sellers), the dominant companies will act competitively when there is a lack of barriers for the competitors. Dominant players in an industry will do everything to reduce the contestability of their industry by preventing new entrants from driving them out of business. The features of a Contestable Market include: There are no barriers to entry or exit barriers. There are no sunk costs, i.e.; the costs that have already been incurred and cannot be recovered. Both the incumbent firms and new entrants have access to the same level of technology. The continuous risk of new entrants emerging and stealing market share leads the incumbents to focus more on maximizing sales rather than profits. They realize that if they are too profitable, an entrant could easily come and undercut their business. Incorrect Solution (b) Statement 1 Statement 2 Statement 3 Correct Incorrect Correct There are no barriers to entry or exit barriers is one of the features of a Contestable Market. There are no sunk costs is one of the features of a Contestable Market. Both the incumbent firms and new entrants have access to the same level of technology is one of the features of a Contestable Market.   Note: Contestable Market Theory: ‘The Contestable Market Theory‘ is an economic concept stating that the companies with a few rivals behave in a competitive manner, when the market they operate in has weak barriers to entry. ‘Contestable‘ in economics means that a company can be challenged or contested by the rival companies looking to enter the industry or the market. In other words, a Contestable Market is a market whereby the companies can enter and leave freely with low sunk costs. Sunk costs are major irrecoverable costs to enter an industry, such as the purchase of a manufacturing plant or equipment. The Contestable Market Theory assumes that even in a monopoly or oligopoly (It is a market form wherein a market or an industry is dominated by a small group of large sellers), the dominant companies will act competitively when there is a lack of barriers for the competitors. Dominant players in an industry will do everything to reduce the contestability of their industry by preventing new entrants from driving them out of business. The features of a Contestable Market include: There are no barriers to entry or exit barriers. There are no sunk costs, i.e.; the costs that have already been incurred and cannot be recovered. Both the incumbent firms and new entrants have access to the same level of technology. The continuous risk of new entrants emerging and stealing market share leads the incumbents to focus more on maximizing sales rather than profits. They realize that if they are too profitable, an entrant could easily come and undercut their business. Question 17 of 35 17. Question Consider the following statements: The Securities and Exchange Board of India (SEBI) was set up in 1988 and was given statutory recognition in 1992. Capital market refers to the market for short to medium-term funds for investment purposes. Bombay Stock Exchange is the largest stock market in the country. How many of the above statements are correct? a) Only one b) Only two c) All three d) None Correct Solution (a) Statement 1 Statement 2 Statement 3 Correct Incorrect Incorrect The Securities and Exchange Board of India (SEBI) was set up in 1988 and was given statutory recognition in 1992. Capital market refers to the market for long-term funds for investment purposes. The capital market is the source of funds for the corporates and the governments, and provides opportunities to the savers to park their long-term savings. The capital market comprises two segments: the primary and the secondary markets. The setting up to the NSE (The National Stock Exchange of India Ltd) is a landmark in the Indian capital markets. At present, the NSE is the largest stock market in the country. Trading on the NSE can be done throughout the country through the network of satellite terminals. The NSE has introduced inter-regional clearing facilities. Incorrect Solution (a) Statement 1 Statement 2 Statement 3 Correct Incorrect Incorrect The Securities and Exchange Board of India (SEBI) was set up in 1988 and was given statutory recognition in 1992. Capital market refers to the market for long-term funds for investment purposes. The capital market is the source of funds for the corporates and the governments, and provides opportunities to the savers to park their long-term savings. The capital market comprises two segments: the primary and the secondary markets. The setting up to the NSE (The National Stock Exchange of India Ltd) is a landmark in the Indian capital markets. At present, the NSE is the largest stock market in the country. Trading on the NSE can be done throughout the country through the network of satellite terminals. The NSE has introduced inter-regional clearing facilities. Question 18 of 35 18. Question With reference to the Interim Budget, consider the following statements: The Interim Budget is a budget which is presented in an election year and is applicable for the entire financial year. Usually, the Interim Budget does not contain the Finance Bill, but includes the Vote-on-Account. Which of the statements given above is/are correct? a) 1 only b) 2 only c) Both 1 and 2 d) Neither 1 nor 2 Correct Solution (b) Statement 1 Statement 2 Incorrect Correct The Interim Budget is essentially a budget which is announced in an election year by the incumbent government. The Interim Budget would be applicable until the new government takes over after the General Elections. Hence, unlike the Union Budget, the Interim Budget would be applicable for a part of the financial year. The Interim Budget does not contain the Finance Bill, but includes the Vote-on-Account. The government needs approval from the Parliament to withdraw money from the Consolidated Fund of India to meet its day-to-day expenses. Hence, the incumbent government presents the Vote-on-Account in order to seek approval from the Parliament for withdrawing money to meet its expenditure needs until the new government takes over. Incorrect Solution (b) Statement 1 Statement 2 Incorrect Correct The Interim Budget is essentially a budget which is announced in an election year by the incumbent government. The Interim Budget would be applicable until the new government takes over after the General Elections. Hence, unlike the Union Budget, the Interim Budget would be applicable for a part of the financial year. The Interim Budget does not contain the Finance Bill, but includes the Vote-on-Account. The government needs approval from the Parliament to withdraw money from the Consolidated Fund of India to meet its day-to-day expenses. Hence, the incumbent government presents the Vote-on-Account in order to seek approval from the Parliament for withdrawing money to meet its expenditure needs until the new government takes over. Question 19 of 35 19. Question Which among the following is the most important reason(s) for the government to prefer the Exchange Traded Fund (ETF) route for undertaking disinvestment in the PSUs? Ability to undertake disinvestment in multiple PSUs at a single time. Enables the government to reduce its ownership in the PSUs to below 51% and yet retain management control. Enables the government to sell its entire stake to a single private sector entity. How many of the statements given above is/are correct? a) Only one b) Only two c) All three d) None Correct Solution (a) Exchange Traded Fund (ETF): The Exchange Traded Fund (ETF) is a fund that is created by pooling together the assets and then dividing this cumulated asset into individual units that are traded on the stock exchange. The value of the ETF comes from the value of the underlying assets (shares of stock, bonds, foreign currency, etc.). These ETFs, listed in the stock exchanges, are similar to shares and can be traded like ordinary shares. In nature, the ETFs are Index Funds, because they comprise shares of different companies. The share-based ETFs, issued by the government, include CPSE ETF and Bharat-22 ETF. These share-based ETFs enable the government to undertake the disinvestment in multiple PSUs at a single time and hence, reduces the time for undertaking disinvestment. This is the main reason for the higher preference of the government towards the ETF route for undertaking disinvestment. Incorrect Solution (a) Exchange Traded Fund (ETF): The Exchange Traded Fund (ETF) is a fund that is created by pooling together the assets and then dividing this cumulated asset into individual units that are traded on the stock exchange. The value of the ETF comes from the value of the underlying assets (shares of stock, bonds, foreign currency, etc.). These ETFs, listed in the stock exchanges, are similar to shares and can be traded like ordinary shares. In nature, the ETFs are Index Funds, because they comprise shares of different companies. The share-based ETFs, issued by the government, include CPSE ETF and Bharat-22 ETF. These share-based ETFs enable the government to undertake the disinvestment in multiple PSUs at a single time and hence, reduces the time for undertaking disinvestment. This is the main reason for the higher preference of the government towards the ETF route for undertaking disinvestment. Question 20 of 35 20. Question The Panda Bonds, sometimes seen in the news, refer to which of the following? a) Yuan-denominated bonds issued in Chinese mainland by an overseas entity. b) Yuan-denominated bonds issued in Chinese mainland by the Chinese companies. c) Rupee denominated bonds issued in Chinese mainland. d) Yuan denominated bonds issued in India. Correct Solution (a) Panda Bonds: The Panda Bonds are the Yuan-denominated bonds issued in the Chinese mainland market by an overseas entity. They are usually issued by the foreign companies and the Chinese companies operating overseas to raise capital. The International Finance Corporation and the Asian Development Bank have issued the Panda Bonds in 2005. Besides, countries like the Philippines and most recently, Pakistan have issued the Panda Bonds. Incorrect Solution (a) Panda Bonds: The Panda Bonds are the Yuan-denominated bonds issued in the Chinese mainland market by an overseas entity. They are usually issued by the foreign companies and the Chinese companies operating overseas to raise capital. The International Finance Corporation and the Asian Development Bank have issued the Panda Bonds in 2005. Besides, countries like the Philippines and most recently, Pakistan have issued the Panda Bonds. Question 21 of 35 21. Question Consider the following statements about ‘Iyothee Thass Pandithar’: He was a prominent anti-caste activist and a practitioner of Siddha medicine. He established the Advaidananda Sabha and launched a magazine called Dravida Pandian. He organized the Todas and other tribes of the Nilgiri Hills into a formidable force for the freedom movement. He established the Sakya Buddhist Society in Bengal. How many of the above statements are correct? a) Only one b) Only two c) Only three d) All four Correct Solution (c) Iyothee Thass Pandithar was a prominent anti-caste activist and a practitioner of Siddha medicine. Hence statement 1 is correct. He was born on 20 May 1845 in Madras presidency. He established the Advaidananda Sabha and launched a magazine called Dravida Pandian. Hence statement 2 is correct. He established the “Dravida Mahajana Sabha ” in 1891 along with Rettamalai Srinivasan. He organized the Todas and other tribes of the Nilgiri Hills into a formidable force for the freedom movement. Hence statement 3 is correct. He established the Sakya Buddhist Society in Madras with branches all over South India. Hence statement 4 is incorrect. The Sakya Buddhist Society was also known as the Indian Buddhist Association and was established in the year 1898. To manage and coordinate the functioning of the society, he began a weekly magazine, Tamizhan in 1907. Incorrect Solution (c) Iyothee Thass Pandithar was a prominent anti-caste activist and a practitioner of Siddha medicine. Hence statement 1 is correct. He was born on 20 May 1845 in Madras presidency. He established the Advaidananda Sabha and launched a magazine called Dravida Pandian. Hence statement 2 is correct. He established the “Dravida Mahajana Sabha ” in 1891 along with Rettamalai Srinivasan. He organized the Todas and other tribes of the Nilgiri Hills into a formidable force for the freedom movement. Hence statement 3 is correct. He established the Sakya Buddhist Society in Madras with branches all over South India. Hence statement 4 is incorrect. The Sakya Buddhist Society was also known as the Indian Buddhist Association and was established in the year 1898. To manage and coordinate the functioning of the society, he began a weekly magazine, Tamizhan in 1907. Question 22 of 35 22. Question Consider the following statements about Mumps: It is a contagious disease caused by a paramyxovirus. It is an acute disease that infects children and young adults. There is no specific treatment for mumps. How many of the above statements are correct? a) Only one b) Only two c) All three d) None Correct Solution (c) Mumps is a contagious disease caused by a paramyxovirus. Hence statement 1 is correct. It can cause the parotid glands, the ones on each side of the face to swell and they may also be tender or painful. It is an acute disease that infects children and young adults. Hence statement 2 is correct. Humans are the only known host for the mumps virus, which is spread via direct contact or by airborne droplets from the upper respiratory tract of infected individuals. There is no specific treatment for mumps. Hence statement 3 is correct. The various symptoms may be relieved with medicines. The best way to protect kids is to make sure they’re immunized against mumps. Incorrect Solution (c) Mumps is a contagious disease caused by a paramyxovirus. Hence statement 1 is correct. It can cause the parotid glands, the ones on each side of the face to swell and they may also be tender or painful. It is an acute disease that infects children and young adults. Hence statement 2 is correct. Humans are the only known host for the mumps virus, which is spread via direct contact or by airborne droplets from the upper respiratory tract of infected individuals. There is no specific treatment for mumps. Hence statement 3 is correct. The various symptoms may be relieved with medicines. The best way to protect kids is to make sure they’re immunized against mumps. Question 23 of 35 23. Question Consider the following statements about different types of ‘Otoliths’ that aid fish in balance and hearing: Sagitta is involved in the detection of sound and the process of hearing, or converting sound waves into electrical signals. Asteriscus is involved in the detection of sound and the process of hearing. Lapillus is involved in the detection of gravitational force and sound. How many of the above statements are correct? a) Only one b) Only two c) All three d) None Correct Solution (c) The otolith is a stony lump in the fish ear. These are much like tree rings which reveal fish’s age. Different forms or isotopes of oxygen in the otolith indicate the temperature the fish experienced when it was alive. Carbon isotopes reveal how quickly food is converted into energy. Fish carry their fitness trackers in their ears. They are commonly known as “ear stones,” which are hard, calcium carbonate structures located directly behind the brains of bony fishes. The different types of otoliths that aid fish in balance and hearing are: Sagitta is involved in the detection of sound and the process of hearing, or converting sound waves into electrical signals. Hence statement 1 is correct. Asteriscus is involved in the detection of sound and the process of hearing. Hence statement 2 is correct. Lapillus is involved in the detection of gravitational force and sound. Hence statement 3 is correct.   Incorrect Solution (c) The otolith is a stony lump in the fish ear. These are much like tree rings which reveal fish’s age. Different forms or isotopes of oxygen in the otolith indicate the temperature the fish experienced when it was alive. Carbon isotopes reveal how quickly food is converted into energy. Fish carry their fitness trackers in their ears. They are commonly known as “ear stones,” which are hard, calcium carbonate structures located directly behind the brains of bony fishes. The different types of otoliths that aid fish in balance and hearing are: Sagitta is involved in the detection of sound and the process of hearing, or converting sound waves into electrical signals. Hence statement 1 is correct. Asteriscus is involved in the detection of sound and the process of hearing. Hence statement 2 is correct. Lapillus is involved in the detection of gravitational force and sound. Hence statement 3 is correct.   Question 24 of 35 24. Question Consider the following statements about the Codex Alimentarius Commission: It is an international food safety and quality standard-setting body. It was created by the World Health Organisation and the Food and Agriculture Organization of the United Nations. It meets in regular sessions once in three years alternating between Geneva and Rome. How many of the above statements are correct? a) Only one b) Only two c) All three d) None Correct Solution (b) The Codex Alimentarius Commission is an international food safety and quality standard-setting body. Hence statement 1 is correct. Its objective is protecting consumer’s health and ensuring fair practices in the food trade. It was created by the World Health Organisation and the Food and Agriculture Organization of the United Nations. Hence statement 2 is correct. It consists of 189 member countries. Its membership is open to all Member Nations and Associate Members of FAO and WHO which are interested in international food standards. It meets in regular sessions once a year alternating between Geneva and Rome. Hence statement 3 is incorrect. Incorrect Solution (b) The Codex Alimentarius Commission is an international food safety and quality standard-setting body. Hence statement 1 is correct. Its objective is protecting consumer’s health and ensuring fair practices in the food trade. It was created by the World Health Organisation and the Food and Agriculture Organization of the United Nations. Hence statement 2 is correct. It consists of 189 member countries. Its membership is open to all Member Nations and Associate Members of FAO and WHO which are interested in international food standards. It meets in regular sessions once a year alternating between Geneva and Rome. Hence statement 3 is incorrect. Question 25 of 35 25. Question Consider the following statements about the Indian Tent Turtle: It is listed as critically endangered on the IUCN Red List. It is protected under Schedule I of the Wildlife Protection Act of 1972. It is listed under the Appendix I of the CITES. How many of the above statements are correct? a) Only one b) Only two c) All three d) None Correct Solution (a) The Indian tent turtle (Pangshura tentoria) is a species of turtle in the family Geoemydidae. It is an active swimmer and a mainly herbivorous species. Its habitats include still water pools on the riverside and slow-running water near the river banks. It is native to India, Nepal, and Bangladesh. The Indian Tent Turtle is listed as least concern on the IUCN Red List. Hence statement 1 is incorrect. It is protected under Schedule I of the Wildlife Protection Act of 1972. Hence statement 2 is correct. It is listed under the Appendix II of the CITES. Hence statement 3 is incorrect. Incorrect Solution (a) The Indian tent turtle (Pangshura tentoria) is a species of turtle in the family Geoemydidae. It is an active swimmer and a mainly herbivorous species. Its habitats include still water pools on the riverside and slow-running water near the river banks. It is native to India, Nepal, and Bangladesh. The Indian Tent Turtle is listed as least concern on the IUCN Red List. Hence statement 1 is incorrect. It is protected under Schedule I of the Wildlife Protection Act of 1972. Hence statement 2 is correct. It is listed under the Appendix II of the CITES. Hence statement 3 is incorrect. Question 26 of 35 26. Question Consider the following statements about De Winton’s Golden Mole: It is an elusive blind mole endemic to Australia that swims through sand and lives in inaccessible burrows. Its natural habitats are subtropical dry shrubland, Mediterranean-type shrubby vegetation, and sandy shores. It is currently the only golden mole species listed as critically endangered on the IUCN List. How many of the above statements are correct? a) Only one b) Only two c) All three d) None Correct Solution (b) De Winton’s Golden Mole is an elusive blind mole endemic to South Africa that swims through sand and lives in inaccessible burrows. Hence statement 1 is incorrect. It has only ever been recorded in one location – Port Nolloth, on South Africa’s northwest coast – where it was last sighted in 1937. Its natural habitats are subtropical dry shrubland, Mediterranean-type shrubby vegetation, and sandy shores. Hence statement 2 is correct. The upper parts have short, dense fur that is slate grey with a yellowish tinge. Individual hairs have grey bases, whitish shafts, and fawn tips. The face, cheeks, and lips have a more intense yellowish tinge. The underparts are rather paler than the upper parts, with individual hairs having white tips. It is currently the only golden mole species listed as critically endangered on the IUCN List. Hence statement 3 is correct. It is among the 25 “most wanted lost” species that are the focus of Global Wildlife Conservation’s “Search for Lost Species” initiative. Incorrect Solution (b) De Winton’s Golden Mole is an elusive blind mole endemic to South Africa that swims through sand and lives in inaccessible burrows. Hence statement 1 is incorrect. It has only ever been recorded in one location – Port Nolloth, on South Africa’s northwest coast – where it was last sighted in 1937. Its natural habitats are subtropical dry shrubland, Mediterranean-type shrubby vegetation, and sandy shores. Hence statement 2 is correct. The upper parts have short, dense fur that is slate grey with a yellowish tinge. Individual hairs have grey bases, whitish shafts, and fawn tips. The face, cheeks, and lips have a more intense yellowish tinge. The underparts are rather paler than the upper parts, with individual hairs having white tips. It is currently the only golden mole species listed as critically endangered on the IUCN List. Hence statement 3 is correct. It is among the 25 “most wanted lost” species that are the focus of Global Wildlife Conservation’s “Search for Lost Species” initiative. Question 27 of 35 27. Question Consider the following statements about Operation Storm Makers II: It was conducted by Interpol in which India was a participant. It aims to target fraud schemes fuelled by victims of human trafficking. Choose the correct code: a) 1 only b) 2 only c) Both 1 and 2 d) Neither 1 nor 2 Correct Solution (c) Operation Storm Makers II was conducted by Interpol in which India was a participant. Hence statement 1 is correct. Human trafficking-fuelled frauds are expanding beyond Southeast Asia. Victims are lured by fake job ads and forced to commit online fraud such as – Fake cryptocurrency investments, Work-from-home scams, and Lottery and online gambling scams. It aims to target fraud schemes fuelled by victims of human trafficking. Hence statement 2 is correct.   Incorrect Solution (c) Operation Storm Makers II was conducted by Interpol in which India was a participant. Hence statement 1 is correct. Human trafficking-fuelled frauds are expanding beyond Southeast Asia. Victims are lured by fake job ads and forced to commit online fraud such as – Fake cryptocurrency investments, Work-from-home scams, and Lottery and online gambling scams. It aims to target fraud schemes fuelled by victims of human trafficking. Hence statement 2 is correct.   Question 28 of 35 28. Question Consider the following statements: Abelmoschus Odishae is a new frog species discovered in a moist deciduous forest in Odisha. Indian Council of Agricultural Research (ICAR) is an autonomous organisation under the Department of Agricultural Research and Education (DARE). Choose the correct code: a) 1 only b) 2 only c) Both 1 and 2 d) Neither 1 nor 2 Correct Solution (b) Abelmoschus Odishae is a new plant species of ‘wild okra’ discovered in a moist deciduous forest in Odisha. Hence statement 1 is incorrect. It can be used for hybridisation to create a better variety with high disease resistance. It can also play a vital role in widening the genetic base of okra. Indian Council of Agricultural Research (ICAR) is an autonomous organisation under the Department of Agricultural Research and Education (DARE), Ministry of Agriculture, and Farmers Welfare. Hence statement 2 is correct. It was formerly known as the Imperial Council of Agricultural Research, it was established on 16 July 1929, as a registered society under the Societies Registration Act, 1860. It is responsible for coordinating agricultural education and research in India.   Incorrect Solution (b) Abelmoschus Odishae is a new plant species of ‘wild okra’ discovered in a moist deciduous forest in Odisha. Hence statement 1 is incorrect. It can be used for hybridisation to create a better variety with high disease resistance. It can also play a vital role in widening the genetic base of okra. Indian Council of Agricultural Research (ICAR) is an autonomous organisation under the Department of Agricultural Research and Education (DARE), Ministry of Agriculture, and Farmers Welfare. Hence statement 2 is correct. It was formerly known as the Imperial Council of Agricultural Research, it was established on 16 July 1929, as a registered society under the Societies Registration Act, 1860. It is responsible for coordinating agricultural education and research in India.   Question 29 of 35 29. Question Consider the following statements about the Red Sea: It is the world’s northernmost tropical sea which lies between Asia and Africa. Reef systems are better developed along the sea because of their greater depths and efficient water circulation patterns. It is bordered by Egypt to the north and west and by Sudan, Eritrea, and Djibouti to the west. Choose the correct code: a) Only one b) Only two c) All three d) None Correct Solution (c) Red Sea is the world’s northernmost tropical sea which lies between Asia and Africa. Hence statement 1 is correct. It is connected to the Arabian Sea and the Indian Ocean to the south through the Gulf of Aden and the narrow strait of Bab el-Mandeb. Reef systems are better developed along the sea because of their greater depths and efficient water circulation patterns. Hence statement 2 is correct. Its well-known islands include Tiran Island, which is located near the mouth of the Gulf of Aqaba, and Shadwan Island, which is located at the entrance of the Gulf of Suez. Yemen and Saudi Arabia border the Red Sea to the east. It is bordered by Egypt to the north and west and by Sudan, Eritrea, and Djibouti to the west. Hence statement 3 is correct. Incorrect Solution (c) Red Sea is the world’s northernmost tropical sea which lies between Asia and Africa. Hence statement 1 is correct. It is connected to the Arabian Sea and the Indian Ocean to the south through the Gulf of Aden and the narrow strait of Bab el-Mandeb. Reef systems are better developed along the sea because of their greater depths and efficient water circulation patterns. Hence statement 2 is correct. Its well-known islands include Tiran Island, which is located near the mouth of the Gulf of Aqaba, and Shadwan Island, which is located at the entrance of the Gulf of Suez. Yemen and Saudi Arabia border the Red Sea to the east. It is bordered by Egypt to the north and west and by Sudan, Eritrea, and Djibouti to the west. Hence statement 3 is correct. Question 30 of 35 30. Question Consider the following statements about Mohenjo Daro: It lies in Pakistan’s Sindh province, on the bank of the Indus River. It was divided into two parts – the citadel and the lower town. Its ruins were designated a UNESCO World Heritage Site. How many of the above statements are correct? a) Only one b) Only two c) All three d) None Correct Solution (c) Mohenjo Daro lies in Pakistan’s Sindh province, on the bank of the Indus River. Hence statement 1 is correct. The people of Mohenjo-Daro lived in houses that had similar features throughout the town. They consisted of a central courtyard surrounded by rooms. Almost every household had individual toilets and bathrooms. It was divided into two parts – the citadel and the lower town. Hence statement 2 is correct. The western mound, or Citadel was built on a raised platform of mud brick and consisted of all the important administrative structures like The Great Bath, granaries, and the College of Priests. It was fortified by a thick mud-brick retaining wall. The eastern or low-lying part of the town was also fortified and was meant for the settlement of commoners. Numerous streets and small alleys ran across this part of the city. Within this section, many sub-parts were built according to the guild systems. As people settled here, a large number of artifacts and information regarding the burial practices were found in this area. Its ruins were designated a UNESCO World Heritage Site in 1980. Hence statement 3 is correct. It was laid out in a rectilinear grid and built out of baked bricks, the city featured a complex water management system, complete with a sophisticated drainage and covered sewer system, and baths in nearly every house. Incorrect Solution (c) Mohenjo Daro lies in Pakistan’s Sindh province, on the bank of the Indus River. Hence statement 1 is correct. The people of Mohenjo-Daro lived in houses that had similar features throughout the town. They consisted of a central courtyard surrounded by rooms. Almost every household had individual toilets and bathrooms. It was divided into two parts – the citadel and the lower town. Hence statement 2 is correct. The western mound, or Citadel was built on a raised platform of mud brick and consisted of all the important administrative structures like The Great Bath, granaries, and the College of Priests. It was fortified by a thick mud-brick retaining wall. The eastern or low-lying part of the town was also fortified and was meant for the settlement of commoners. Numerous streets and small alleys ran across this part of the city. Within this section, many sub-parts were built according to the guild systems. As people settled here, a large number of artifacts and information regarding the burial practices were found in this area. Its ruins were designated a UNESCO World Heritage Site in 1980. Hence statement 3 is correct. It was laid out in a rectilinear grid and built out of baked bricks, the city featured a complex water management system, complete with a sophisticated drainage and covered sewer system, and baths in nearly every house. Question 31 of 35 31. Question The sum of 3 numbers is 85. If third number be one fifth of the first and the first number be twice the second, find the second number   a) 20 b) 25 c) 10 d) 15 Correct Solution (b) We know from the details of the question that the ratio of the numbers = 1: 1/2: 1/5 Taking LCM we get the ratio = 10: 5: 2 The total of the ratios = 17 Second number = 5/17 × 85 = 25   Incorrect Solution (b) We know from the details of the question that the ratio of the numbers = 1: 1/2: 1/5 Taking LCM we get the ratio = 10: 5: 2 The total of the ratios = 17 Second number = 5/17 × 85 = 25   Question 32 of 35 32. Question A contract is to be completed in 60 days if 105 persons work, each working at 8 hours a day. After 30 days, 2/5 of the work is completed. How many additional persons should be deployed so that the work will be completed in the scheduled time, each person now working 9 hours a day?   a) 35 b) 40 c) 28 d) 43 Correct Solution (a) Persons worked (M1) = 105 Number of hours each person worked per day (H1) = 8 Number of days they worked (D1) = 30 Work completed (W1)= 2/5 Remaining days (D2)= 60 – 30 = 30 Remaining Work to be completed (W2) = (1 – 2/5 ) = 3/5 Let the total number of persons who do the remaining work (M2) = x Number of hours each person needs to be work per day (H2) = 9 = M1D1H1/W1 = M2D2H2/W2 = (105×30×8)/ (2/5) = (x×30×9)/ (3/5) => x = 140 So, additional person = 140 – 105 = 35 Incorrect Solution (a) Persons worked (M1) = 105 Number of hours each person worked per day (H1) = 8 Number of days they worked (D1) = 30 Work completed (W1)= 2/5 Remaining days (D2)= 60 – 30 = 30 Remaining Work to be completed (W2) = (1 – 2/5 ) = 3/5 Let the total number of persons who do the remaining work (M2) = x Number of hours each person needs to be work per day (H2) = 9 = M1D1H1/W1 = M2D2H2/W2 = (105×30×8)/ (2/5) = (x×30×9)/ (3/5) => x = 140 So, additional person = 140 – 105 = 35 Question 33 of 35 33. Question It is not only the untreated sewage water and industrial effluents, but also the solid wastes and construction material discharged by individuals, companies and municipal bodies that have caused the suffocation of the Yamuna. Also, floodplains have been encroached upon by settlements. Hence, ensuring supply of quality drinking water is not only expensive, it also needs improvement in governance. It needs technical knowledge on measurement and regulation of water quality. It is not the fault of the DJB or the Delhi government alone that they have not been able to ensure 100% supply of quality water to the citizens of Delhi, considering the constraints they face, especially those concerning the water resources management and laws in the country. Which one of the following is the most logical and rational inference that can be made from the above passage? a) The water quality of Yamuna water can be improved only by clearing the encroachment on the flood plains. b) Many factors are at play that need to be addressed for improving the quality of water of a river. c) Improper management of resources and lack of proper laws have no effect on the water quality of a river. d) Climate change and unplanned development is taking a toll on the quality of water of a river. Correct Solution (b) Many factors like control of effluents and dumping of construction material, removing the encroachments from the flood plains, proper monitoring of water quality etc. are needed to improve the quality of the water of a river. The impact of climate change has not been discussed in the passage. Keeping these things in the mind, the most appropriate among the given answer options is option B. Incorrect Solution (b) Many factors like control of effluents and dumping of construction material, removing the encroachments from the flood plains, proper monitoring of water quality etc. are needed to improve the quality of the water of a river. The impact of climate change has not been discussed in the passage. Keeping these things in the mind, the most appropriate among the given answer options is option B. Question 34 of 35 34. Question The structure of India‘s population—that has been famously described as a “pyramid” is changing to that of an “unhealthy middle-aged man with a big head and fat belly”. The heavy belly comes from a dramatic increase in the size of the lower middle class and the big head comes from rising income inequality. The richest 1% of India now hold 58% of wealth, and the richest 10% over 80%. This structural shift will have profound consequences for politics and economics, with an aspiring class that is very large and growing, and with abject poverty declining to a much smaller percentage of the population. The terms “big head” and “fat belly”, as used in the passage, refer to a) Changing lifestyle of Indian people and the ensuing physical and mental health issues b) Uneven patterns of income distribution across different strata of population. c) Change in population profile of Indian population. d) Unhealthy dietary habits leading to an increase in average waist size of Indians Correct Solution (b) The passage begins by mentioning the structure of Indian population vis-a-vis its income, which has a pyramidal pattern. The terms “big head” and “fat belly” refer to the change in this pyramidal pattern. “Heavy belly” refers to a dramatic increase in the size of the lower middle class, and “big head” refers to rising income inequality. Option C might seem a little confusing. The passage talks about income distribution, not the number of people as a function of their ages. Hence, option C is incorrect. Incorrect Solution (b) The passage begins by mentioning the structure of Indian population vis-a-vis its income, which has a pyramidal pattern. The terms “big head” and “fat belly” refer to the change in this pyramidal pattern. “Heavy belly” refers to a dramatic increase in the size of the lower middle class, and “big head” refers to rising income inequality. Option C might seem a little confusing. The passage talks about income distribution, not the number of people as a function of their ages. Hence, option C is incorrect. Question 35 of 35 35. Question As a poet, Kabir transcended many of the divisions that existed in India. He can be celebrated as Dalit hero or as a Brahmin. Kabir and the Kabir Panth are accepted as a part of Hinduism. A large corpus of his poems is included in the Guru Granth Sahib. His presence in Indian Islamic thought, Qawwali singing and architecture has also been well documented. Nineteenth century missionaries noted the similarity of his thoughts to Christianity. His indebtedness to Buddhist Siddhas has been a subject of scholarship in the last century. Jain poets emulated his style, so much so that the 17th century Anandghan was dubbed as the ‘Jain Kabir’. The passage seems to argue that a) Kabir‘s thoughts are reflected in almost all major religions in India. b) Kabir supported different religions at different phases of his life. c) Kabir denounced untouchability and favoured equality among different castes. d) Kabir‘s presence in Indian Islamic thought is more prominent as compared to other religions. Correct Solution (a) Option A is correct. Throughout the passage it can be seen that the author is explaining how Kabir‘s thoughts have been incorporated by various religions in India. For example, he is celebrated as a Dalit hero or a Brahmin. His poems which reflected his thoughts are included in Guru Granth Sahib (Sikhs). Similar connection with other religions like Christianity, Jainism and Islam is also established in the passage. Option B is incorrect. The passage only explains how Kabir‘s thoughts have been included in various religions around India. It nowhere mentions whether Kabir supported any religion or not. Hence, this statement is not correct. Option C is incorrect. Though the passage mentions in the beginning, “He can be celebrated as Dalit hero or as a Brahmin”, it cannot be conclusively established from this paragrapn that Kabir advocated equality between different castes. Also there is no mention of untouchability in the passage. Hence, this statement is not correct. Option D is incorrect. The passage mentions the presence of Kabir’s thoughts in Islam through the statement, “His presence in Indian Islamic thought, Qawwali singing and architecture has also been well documented”. But, the passage does not argue that his presence in Islamic thought is more prominent than that in other religions.   Incorrect Solution (a) Option A is correct. Throughout the passage it can be seen that the author is explaining how Kabir‘s thoughts have been incorporated by various religions in India. For example, he is celebrated as a Dalit hero or a Brahmin. His poems which reflected his thoughts are included in Guru Granth Sahib (Sikhs). Similar connection with other religions like Christianity, Jainism and Islam is also established in the passage. Option B is incorrect. The passage only explains how Kabir‘s thoughts have been included in various religions around India. It nowhere mentions whether Kabir supported any religion or not. Hence, this statement is not correct. Option C is incorrect. Though the passage mentions in the beginning, “He can be celebrated as Dalit hero or as a Brahmin”, it cannot be conclusively established from this paragrapn that Kabir advocated equality between different castes. Also there is no mention of untouchability in the passage. Hence, this statement is not correct. Option D is incorrect. The passage mentions the presence of Kabir’s thoughts in Islam through the statement, “His presence in Indian Islamic thought, Qawwali singing and architecture has also been well documented”. But, the passage does not argue that his presence in Islamic thought is more prominent than that in other religions.   window.wpProQuizInitList = window.wpProQuizInitList || []; window.wpProQuizInitList.push({ id: '#wpProQuiz_3627', init: { quizId: 3627, mode: 1, globalPoints: 70, timelimit: 1800, resultsGrade: [0], bo: 704, qpp: 0, catPoints: [70], formPos: 0, lbn: "Test-summary", json: {"32004":{"type":"single","id":32004,"catId":0,"points":2,"correct":[0,1,0,0]},"32006":{"type":"single","id":32006,"catId":0,"points":2,"correct":[0,0,1,0]},"32008":{"type":"single","id":32008,"catId":0,"points":2,"correct":[1,0,0,0]},"32009":{"type":"single","id":32009,"catId":0,"points":2,"correct":[0,0,0,1]},"32011":{"type":"single","id":32011,"catId":0,"points":2,"correct":[0,1,0,0]},"32014":{"type":"single","id":32014,"catId":0,"points":2,"correct":[0,1,0,0]},"32016":{"type":"single","id":32016,"catId":0,"points":2,"correct":[1,0,0,0]},"32019":{"type":"single","id":32019,"catId":0,"points":2,"correct":[1,0,0,0]},"32022":{"type":"single","id":32022,"catId":0,"points":2,"correct":[0,0,0,1]},"32024":{"type":"single","id":32024,"catId":0,"points":2,"correct":[1,0,0,0]},"32026":{"type":"single","id":32026,"catId":0,"points":2,"correct":[0,0,1,0]},"32028":{"type":"single","id":32028,"catId":0,"points":2,"correct":[0,0,0,1]},"32029":{"type":"single","id":32029,"catId":0,"points":2,"correct":[0,0,1,0]},"32031":{"type":"single","id":32031,"catId":0,"points":2,"correct":[0,0,0,1]},"32033":{"type":"single","id":32033,"catId":0,"points":2,"correct":[0,0,1,0]},"32034":{"type":"single","id":32034,"catId":0,"points":2,"correct":[0,1,0,0]},"32036":{"type":"single","id":32036,"catId":0,"points":2,"correct":[1,0,0,0]},"32038":{"type":"single","id":32038,"catId":0,"points":2,"correct":[0,1,0,0]},"32041":{"type":"single","id":32041,"catId":0,"points":2,"correct":[1,0,0,0]},"32043":{"type":"single","id":32043,"catId":0,"points":2,"correct":[1,0,0,0]},"32046":{"type":"single","id":32046,"catId":0,"points":2,"correct":[0,0,1,0]},"32047":{"type":"single","id":32047,"catId":0,"points":2,"correct":[0,0,1,0]},"32048":{"type":"single","id":32048,"catId":0,"points":2,"correct":[0,0,1,0]},"32050":{"type":"single","id":32050,"catId":0,"points":2,"correct":[0,1,0,0]},"32053":{"type":"single","id":32053,"catId":0,"points":2,"correct":[1,0,0,0]},"32054":{"type":"single","id":32054,"catId":0,"points":2,"correct":[0,1,0,0]},"32055":{"type":"single","id":32055,"catId":0,"points":2,"correct":[0,0,1,0]},"32057":{"type":"single","id":32057,"catId":0,"points":2,"correct":[0,1,0,0]},"32059":{"type":"single","id":32059,"catId":0,"points":2,"correct":[0,0,1,0]},"32061":{"type":"single","id":32061,"catId":0,"points":2,"correct":[0,0,1,0]},"32062":{"type":"single","id":32062,"catId":0,"points":2,"correct":[0,1,0,0]},"32064":{"type":"single","id":32064,"catId":0,"points":2,"correct":[1,0,0,0]},"32067":{"type":"single","id":32067,"catId":0,"points":2,"correct":[0,1,0,0]},"32069":{"type":"single","id":32069,"catId":0,"points":2,"correct":[0,1,0,0]},"32071":{"type":"single","id":32071,"catId":0,"points":2,"correct":[1,0,0,0]}} } }); All the Best IASbaba

Daily Prelims CA Quiz

UPSC Quiz – 2024 : IASbaba’s Daily Current Affairs Quiz 10th April 2024

For Previous Daily Quiz (ARCHIVES) – CLICK HERE The Current Affairs questions are based on sources like ‘The Hindu’, ‘Indian Express’ and ‘PIB’, which are very important sources for UPSC Prelims Exam. The questions are focused on both the concepts and facts. The topics covered here are generally different from what is being covered under ‘Daily Current Affairs/Daily News Analysis (DNA) and Daily Static Quiz’ to avoid duplication. The questions would be published from Monday to Saturday before 2 PM. One should not spend more than 10 minutes on this initiative. Gear up and Make the Best Use of this initiative. Do remember that, “the difference between Ordinary and EXTRA-Ordinary is PRACTICE!!” Important Note: Don’t forget to post your marks in the comment section. Also, let us know if you enjoyed today’s test 🙂After completing the 5 questions, click on ‘View Questions’ to check your score, time taken, and solutions. To take the Test Click Here

[DAY 32] 60 DAY RAPID REVISION (RaRe) SERIES for UPSC Prelims 2024 – GEOGRAPHY, CURRENT AFFAIRS & CSAT TEST SERIES!

Archives Hello Friends The 60 Days Rapid Revision (RaRe) Series is IASbaba’s Flagship Initiative recommended by Toppers and loved by the aspirants’ community every year. It is the most comprehensive program which will help you complete the syllabus, revise and practice tests on a daily basis. The Programme on a daily basis includes Daily Prelims MCQs from Static (Monday – Saturday) Daily Static Quiz will cover all the topics of static subjects – Polity, History, Geography, Economics, Environment and Science and technology. 20 questions will be posted daily and these questions are framed from the topics mentioned in the schedule. It will ensure timely and streamlined revision of your static subjects. Daily Current Affairs MCQs (Monday – Saturday) Daily 5 Current Affairs questions, based on sources like ‘The Hindu’, ‘Indian Express’ and ‘PIB’, would be published from Monday to Saturday according to the schedule. Daily CSAT Quiz (Monday – Friday) CSAT has been an Achilles heel for many aspirants. Daily 5 CSAT Questions will be published. Note – Daily Test of 20 static questions, 10 current affairs, and 5 CSAT questions. (35 Prelims Questions) in QUIZ FORMAT will be updated on a daily basis. To Know More about 60 Days Rapid Revision (RaRe) Series – CLICK HERE   60 Day Rapid Revision (RaRe) Series Schedule – CLICK HERE  Important Note Comment your Scores in the Comment Section. This will keep you accountable, responsible and sincere in days to come. It will help us come out with the Cut-Off on a Daily Basis. Let us know if you enjoyed today’s test 🙂  You can post your comments in the given format  (1) Your Score (2) Matrix Meter (3) New Learning from the Test Time limit: 0 Test-summary 0 of 35 questions completed Questions: 1 2 3 4 5 6 7 8 9 10 11 12 13 14 15 16 17 18 19 20 21 22 23 24 25 26 27 28 29 30 31 32 33 34 35 Information The following Test is based on the syllabus of 60 Days Plan-2023 for UPSC IAS Prelims 2022. To view Solutions, follow these instructions: Click on – ‘Start Test’ button Solve Questions Click on ‘Test Summary’ button Click on ‘Finish Test’ button Now click on ‘View Questions’ button – here you will see solutions and links. You have already completed the test before. Hence you can not start it again. Test is loading... You must sign in or sign up to start the test. You have to finish following test, to start this test: Results 0 of 35 questions answered correctly Your time: Time has elapsed You have scored 0 points out of 0 points, (0) Average score     Your score     Categories Not categorized 0% Your result has been entered into leaderboard Loading Name: E-Mail: Captcha: maximum of 70 points Pos. Name Entered on Points Result Table is loading No data available 1 2 3 4 5 6 7 8 9 10 11 12 13 14 15 16 17 18 19 20 21 22 23 24 25 26 27 28 29 30 31 32 33 34 35 Answered Review Question 1 of 35 1. Question Consider the following statements with respect to the Five-Year Planning (FYP) phase: Implementation of Family Planning Programs was amongst the major targets of the seventh FYP. The ninth FYP for the first time had a separate plan document on states. The concept of Inclusive growth was introduced for the first time in the eighth FYP. Select the correct answer using the code given below: a) Only one b) Only two c) All three d) None Correct Solution (d) Statement 1 Statement 2 Statement 3 Incorrect Incorrect Incorrect The objective of fourth FYP (1969-74) includes “growth with stability and progressive achievement of self- reliance”. Implementation of Family Planning Programs was amongst the major targets of the Plan. The tenth FYP (2002 – 2007) sought to double national per capita income and create a hundred million jobs in 10 years. The 10th FYP emphasized regional imbalance and for the first time had a separate plan document on states. The eleventh plan (2007 – 2012) introduced the concept of inclusive growth. It focused on human resources, especially health and skill development. Incorrect Solution (d) Statement 1 Statement 2 Statement 3 Incorrect Incorrect Incorrect The objective of fourth FYP (1969-74) includes “growth with stability and progressive achievement of self- reliance”. Implementation of Family Planning Programs was amongst the major targets of the Plan. The tenth FYP (2002 – 2007) sought to double national per capita income and create a hundred million jobs in 10 years. The 10th FYP emphasized regional imbalance and for the first time had a separate plan document on states. The eleventh plan (2007 – 2012) introduced the concept of inclusive growth. It focused on human resources, especially health and skill development. Question 2 of 35 2. Question Which of the following expenditures are considered as Non-Developmental expenditure? Expenditure on Administration Maintenance of law and order Cost of tax collection Select the correct answer using the code given below. a) 1 and 2 only b) 2 and 3 only c) 1 and 3 only d) 1, 2 and 3 Correct Solution (d) Statement 1 Statement 2 Statement 3 Correct Correct Correct Non-developmental expenditure includes expenditures made for administrative service, defence service, debt servicing, subsidies, etc. Non-developmental expenditure includes expenditure made on maintenance of law and order. Cost of tax collection, cost of an audit, printing of notes, internal law and order are also included under Non-development expenditure. Incorrect Solution (d) Statement 1 Statement 2 Statement 3 Correct Correct Correct Non-developmental expenditure includes expenditures made for administrative service, defence service, debt servicing, subsidies, etc. Non-developmental expenditure includes expenditure made on maintenance of law and order. Cost of tax collection, cost of an audit, printing of notes, internal law and order are also included under Non-development expenditure. Question 3 of 35 3. Question Consider the following pairs: Deficit                                                 Description Effective Revenue Deficit:: Fiscal deficit – Interest payments Primary Deficit::            Revenue Deficit – Grants for the creation of capital assets Monetized Deficit::            Government deficit financed by borrowing from the RBI How many of the above pairs is/are correctly matched? a) None of the pairs b) One pair only c) Two pairs only d) Three pairs only Correct Solution (b) Statement 1 Statement 2 Statement 3 Incorrect Incorrect Correct Revenue deficit is the difference between revenue receipts and revenue expenditure. Effective Revenue deficit is a new term introduced in the Union Budget 2011-12.   Effective Revenue Deficit = Revenue Deficit – Grants for the creation of capital assets. Primary Deficit is the difference between the current year’s fiscal deficit (Total income– Total expenditure of the government) and the interest paid on the borrowings of the previous year.   Primary Deficit = Fiscal deficit – Interest payments Monetized deficit is also known as the ‘net reserve bank credit to the government’. It is that part of the government deficit which is financed solely by borrowing from the RBI Incorrect Solution (b) Statement 1 Statement 2 Statement 3 Incorrect Incorrect Correct Revenue deficit is the difference between revenue receipts and revenue expenditure. Effective Revenue deficit is a new term introduced in the Union Budget 2011-12.   Effective Revenue Deficit = Revenue Deficit – Grants for the creation of capital assets. Primary Deficit is the difference between the current year’s fiscal deficit (Total income– Total expenditure of the government) and the interest paid on the borrowings of the previous year.   Primary Deficit = Fiscal deficit – Interest payments Monetized deficit is also known as the ‘net reserve bank credit to the government’. It is that part of the government deficit which is financed solely by borrowing from the RBI Question 4 of 35 4. Question With reference to the First Five-Year Plan in India, consider the following statement: It focused on rapid industrialization using heavy and basic industries. The actual growth rate exceeded the target growth rate set by the five-year plan. Choose the correct code: a) 1 only b) 2 only c) Both 1 and 2 d) Neither 1 nor 2 Correct Solution (b) Statement 1 Statement 2 Incorrect Correct An influx of refugees, severe food shortage & mounting inflation confronted the country at the onset of the First Five-year Plan. The Plan Focused on agriculture, price stability, power, and transport. The First Five-Year Plan (1951-56) kept the Growth rate target of 2.1%. Whereas, the actual Growth rate was 3.6%.   Incorrect Solution (b) Statement 1 Statement 2 Incorrect Correct An influx of refugees, severe food shortage & mounting inflation confronted the country at the onset of the First Five-year Plan. The Plan Focused on agriculture, price stability, power, and transport. The First Five-Year Plan (1951-56) kept the Growth rate target of 2.1%. Whereas, the actual Growth rate was 3.6%.   Question 5 of 35 5. Question Which of the following can lead to the condition of ‘Fiscal Slippage’ in an economy? Farm loan waivers Decline in tax collection Increase in disinvestment proceeds How many of the above statements are correct? a) Only one b) Only two c) All three d) None Correct Solution (b) Statement 1 Statement 2 Statement 3 Correct Correct Incorrect Fiscal slippage in simple terms is any deviation in expenditure from the expected. When the Government’s expenditure surpasses the expected or estimated levels, the nation might face the threat of fiscal slippage. Farm loan waivers increase the expenditure that will lead to the condition of fiscal slippage. Tax rate cuts lead to a decline in tax revenue that will lead to the condition of fiscal slippage.   Increase in disinvestment proceed rather prevents (not lead to) the situation of fiscal slippage Incorrect Solution (b) Statement 1 Statement 2 Statement 3 Correct Correct Incorrect Fiscal slippage in simple terms is any deviation in expenditure from the expected. When the Government’s expenditure surpasses the expected or estimated levels, the nation might face the threat of fiscal slippage. Farm loan waivers increase the expenditure that will lead to the condition of fiscal slippage. Tax rate cuts lead to a decline in tax revenue that will lead to the condition of fiscal slippage.   Increase in disinvestment proceed rather prevents (not lead to) the situation of fiscal slippage Question 6 of 35 6. Question With reference to Revenue Deficit, consider the following statements: It includes such transactions that affect the current income and expenditure of the government. It signifies that government is using up the savings to finance a part of its consumption expenditure. Which of the statements given above is/are correct? a) 1 only b) 2 only c) Both 1 and 2 d) Neither 1 nor 2 Correct Solution (c) Statement 1 Statement 2 Correct Correct The revenue deficit includes only such transactions that affect the current income and expenditure of the government. When the government incurs a revenue deficit, it implies that the government is dissaving and is using up the savings of the other sectors of the economy to finance a part of its consumption expenditure.   This situation means that the government will have to borrow not only to finance its investment but also its consumption requirements. This will lead to a build-up of stock of debt and interest liabilities and force the government, eventually, to cut expenditure. Since a major part of revenue expenditure is committed expenditure, it cannot be reduced. Often the government reduces productive capital expenditure or welfare expenditure. This would mean lower growth and adverse welfare implications. Incorrect Solution (c) Statement 1 Statement 2 Correct Correct The revenue deficit includes only such transactions that affect the current income and expenditure of the government. When the government incurs a revenue deficit, it implies that the government is dissaving and is using up the savings of the other sectors of the economy to finance a part of its consumption expenditure.   This situation means that the government will have to borrow not only to finance its investment but also its consumption requirements. This will lead to a build-up of stock of debt and interest liabilities and force the government, eventually, to cut expenditure. Since a major part of revenue expenditure is committed expenditure, it cannot be reduced. Often the government reduces productive capital expenditure or welfare expenditure. This would mean lower growth and adverse welfare implications. Question 7 of 35 7. Question Consider the following statements regarding ‘Short Term Capital Gains Tax’ in India: It refers to a tax on the gain that arises from the sale of an asset after holding it less than sixty months. Securities Transaction Tax (STT) is an indirect tax levied on sale and purchase of equities. Select the correct answer using the code given below: a) 1 only b) 2 only c) Both 1 and 2 d) Neither 1 nor 2 Correct Solution (d) Statement 1 Statement 2 Incorrect Incorrect Short Term Capital Gains Tax (STCG) is the tax levied on profits generated from the sale of an asset which is held for a short-term period which differs for various items – for example it is 36 months or less in the case of immovable property such as land and building and 12 months or less for equity, bonds, government securities etc. The Securities Transaction Tax (STT) is a type of direct tax which is levied at the time of purchase and sale of securities listed on stock exchanges in India. Securities are tradable investment instruments such as shares bonds, debentures, equity-oriented mutual funds (MFs) and so on and are issued either by companies or by the Indian government. This tax was introduced in the Union Budget of 2004. Incorrect Solution (d) Statement 1 Statement 2 Incorrect Incorrect Short Term Capital Gains Tax (STCG) is the tax levied on profits generated from the sale of an asset which is held for a short-term period which differs for various items – for example it is 36 months or less in the case of immovable property such as land and building and 12 months or less for equity, bonds, government securities etc. The Securities Transaction Tax (STT) is a type of direct tax which is levied at the time of purchase and sale of securities listed on stock exchanges in India. Securities are tradable investment instruments such as shares bonds, debentures, equity-oriented mutual funds (MFs) and so on and are issued either by companies or by the Indian government. This tax was introduced in the Union Budget of 2004. Question 8 of 35 8. Question Which of the following statements is correct with respect to ‘General Anti-Avoidance Rule (GAAR)’? a) It is designed to prevent tax avoidance in cross border transactions only. b) Its provisions are included in the Income Tax Act, 1961. c) Its provisions are effective from January 2019. d) As of now, there are no appellate tribunals to appeal against the decisions of tax authorities regarding GAAR provisions. Correct Solution (b) GAAR is an anti-tax avoidance law under Chapter X-A of the Income Tax Act, 1961 of India. It is framed by the Department of Revenue under the Ministry of Finance. GAAR applies to any arrangement that is considered an Impermissible Avoidance Arrangement (IAA). Furthermore, under its provisions, certain transactions are deemed to lack commercial substance. Once the Revenue authorities decide to treat an arrangement as an IAA, the onus to prove otherwise is on taxpayers. Consequently, they are required to substantiate the commercial reasons for such arrangements and that availing tax benefit was not the main purpose for these transactions. Under the Income Tax act, a taxpayer has the right to appeal to the Income Tax Appellate Tribunal (ITAT) against an order passed by the Revenue authorities along with the direction of the Approving Panel. Incorrect Solution (b) GAAR is an anti-tax avoidance law under Chapter X-A of the Income Tax Act, 1961 of India. It is framed by the Department of Revenue under the Ministry of Finance. GAAR applies to any arrangement that is considered an Impermissible Avoidance Arrangement (IAA). Furthermore, under its provisions, certain transactions are deemed to lack commercial substance. Once the Revenue authorities decide to treat an arrangement as an IAA, the onus to prove otherwise is on taxpayers. Consequently, they are required to substantiate the commercial reasons for such arrangements and that availing tax benefit was not the main purpose for these transactions. Under the Income Tax act, a taxpayer has the right to appeal to the Income Tax Appellate Tribunal (ITAT) against an order passed by the Revenue authorities along with the direction of the Approving Panel. Question 9 of 35 9. Question In the context of Indian economy, what is ‘Economic Capital Framework’? a) It is a part of the annual budget documents, intended to be placed before the parliament, outlining the various strategies for capital generation. b) It is a framework developed by the RBI to determine the appropriate level of risk provisions and distribution of profits between the RBI and the Union Government. c) It is the annual submission of RBI under the Basel Committee outlining the former's commitments. d) It is the framework developed by the NITI Aayog that guides the sharing of revenues between the Centre and the states. Correct Solution (b) The economic capital framework provides a methodology for determining the appropriate level of risk provisions and profit distribution to be made under Section 47 of the RBI Act, 1934. As per this provision, the central bank is required to pay balance of its profits to the central government after making provision for bad and doubtful debts, depreciation in assets, and contributions to staff. Incorrect Solution (b) The economic capital framework provides a methodology for determining the appropriate level of risk provisions and profit distribution to be made under Section 47 of the RBI Act, 1934. As per this provision, the central bank is required to pay balance of its profits to the central government after making provision for bad and doubtful debts, depreciation in assets, and contributions to staff. Question 10 of 35 10. Question Which of the following fiscal policy statements are required to be laid before the Parliament under the Fiscal Responsibility and Budget Management Act, 2003 (FRBMA)? Medium-term Fiscal Policy Fiscal Policy Strategy Outcome Budget Medium-term Expenditure Framework Select the correct answer using the code given below. a) 1 and 3 only b) 1, 2 and 4 only c) 2 and 4 only d) 1, 2, 3 and 4 Correct Solution (b) Statement 1 Statement 2 Statement 3 Statement 4 Correct Correct Incorrect Correct The FRBM Act includes the Medium Term Fiscal Policy. It sets out the three-year rolling targets for five specific fiscal indicators in relation to GDP at market prices, namely (i) Revenue Deficit, (ii) Fiscal Deficit, (iii) Effective Revenue Deficit (iv) Tax to GDP ratio and (v) Total outstanding Central Government Debt at the end of the year. The Fiscal Policy Strategy Statement, presented to Parliament as per the FRBM Act. The Strategy Statement outlines the strategic priorities of Government in the fiscal area for the ensuing financial year relating to taxation, expenditure, lending and investments, administered pricing, borrowings and guarantees. The Statement explains how the current policies are in conformity with sound fiscal management principles and gives the rationale for any major deviation in key fiscal measures. The introduction of Outcome Budget is an executive action by the government. From the year 2017-18 onwards, it has been decided that the output and outcomes of the schemes of 68 Ministries and Departments will be available along with the financial outlays as a part of the Budget documents, so that clearly defined objectives and goals for each scheme can be seen by all. The medium-term expenditure framework (MTEF) statement sets a three-year rolling target for expenditure indicators, along with specifications of underpinning assumptions and risks. This statement is presented in Parliament under Section 3 of the FRBM Act, 2003.   The statement provides an estimate of expenditure for various sectors, including education, health, rural development, energy, subsidies and pension, and so on. Incorrect Solution (b) Statement 1 Statement 2 Statement 3 Statement 4 Correct Correct Incorrect Correct The FRBM Act includes the Medium Term Fiscal Policy. It sets out the three-year rolling targets for five specific fiscal indicators in relation to GDP at market prices, namely (i) Revenue Deficit, (ii) Fiscal Deficit, (iii) Effective Revenue Deficit (iv) Tax to GDP ratio and (v) Total outstanding Central Government Debt at the end of the year. The Fiscal Policy Strategy Statement, presented to Parliament as per the FRBM Act. The Strategy Statement outlines the strategic priorities of Government in the fiscal area for the ensuing financial year relating to taxation, expenditure, lending and investments, administered pricing, borrowings and guarantees. The Statement explains how the current policies are in conformity with sound fiscal management principles and gives the rationale for any major deviation in key fiscal measures. The introduction of Outcome Budget is an executive action by the government. From the year 2017-18 onwards, it has been decided that the output and outcomes of the schemes of 68 Ministries and Departments will be available along with the financial outlays as a part of the Budget documents, so that clearly defined objectives and goals for each scheme can be seen by all. The medium-term expenditure framework (MTEF) statement sets a three-year rolling target for expenditure indicators, along with specifications of underpinning assumptions and risks. This statement is presented in Parliament under Section 3 of the FRBM Act, 2003.   The statement provides an estimate of expenditure for various sectors, including education, health, rural development, energy, subsidies and pension, and so on. Question 11 of 35 11. Question Which of the following actions implemented by the government may lead to reduction in its deficit? Increase in the rate of personal Income Tax. Sale of shares of Public Sector Undertakings. Introduction of new welfare schemes by the government. How many of the above statements are correct? a) Only one b) Only two c) All three d) None Correct Solution (b) Statement 1 Statement 2 Statement 3 Correct Correct Incorrect Government deficit can be reduced by an increase in taxes or reduction in expenditure. In India, the government has been trying to increase tax revenue with greater reliance on direct taxes. There has also been an attempt to raise receipts through the sale of shares in PSUs. The other way is to change the scope of the government by withdrawing from some of the areas where it operated before. Cutting back government programmes in vital areas like agriculture, education, health, poverty alleviation, etc. would adversely affect the economy. Incorrect Solution (b) Statement 1 Statement 2 Statement 3 Correct Correct Incorrect Government deficit can be reduced by an increase in taxes or reduction in expenditure. In India, the government has been trying to increase tax revenue with greater reliance on direct taxes. There has also been an attempt to raise receipts through the sale of shares in PSUs. The other way is to change the scope of the government by withdrawing from some of the areas where it operated before. Cutting back government programmes in vital areas like agriculture, education, health, poverty alleviation, etc. would adversely affect the economy. Question 12 of 35 12. Question The government of a nation performs its income distribution function through which of the following mechanisms? Levying taxes Transfer Payment Controlling money supply How many of the above statements are correct? a) Only one b) Only two c) All three d) None Correct Solution (b) Statement 1 Statement 2 Statement 3 Correct Correct Incorrect Through its tax and expenditure policy, the government attempts to bring about a distribution of income that is considered, fair by society. The government affects the personal disposable income of households by making transfer payments and collecting taxes and, therefore, can alter the income distribution. This is the distribution function. Transfer Payment is a payment made or income received in which no goods or services are being paid for, such as a benefit payment or subsidy. Unlike the exchange transaction which mutually benefits all the parties involved in it, the transfer payment consists of a donor and a recipient, with the donor giving up something of value without receiving anything in return. In any period, the level of expenditures may not be sufficient for full utilisation of labour and other resources of the economy. Since wages and prices are generally rigid downwards (they do not fall below a level), employment cannot be restored automatically. Hence, policy measures are needed to raise aggregate demand. Incorrect Solution (b) Statement 1 Statement 2 Statement 3 Correct Correct Incorrect Through its tax and expenditure policy, the government attempts to bring about a distribution of income that is considered, fair by society. The government affects the personal disposable income of households by making transfer payments and collecting taxes and, therefore, can alter the income distribution. This is the distribution function. Transfer Payment is a payment made or income received in which no goods or services are being paid for, such as a benefit payment or subsidy. Unlike the exchange transaction which mutually benefits all the parties involved in it, the transfer payment consists of a donor and a recipient, with the donor giving up something of value without receiving anything in return. In any period, the level of expenditures may not be sufficient for full utilisation of labour and other resources of the economy. Since wages and prices are generally rigid downwards (they do not fall below a level), employment cannot be restored automatically. Hence, policy measures are needed to raise aggregate demand. Question 13 of 35 13. Question Which of the following is/are included in Public Debt of India? Treasury bills Outstanding external debts Borrowing from small savings How many of the above statements are correct? a) Only one b) Only two c) All three d) None Correct Solution (b) Statement 1 Statement 2 Statement 3 Correct Correct Incorrect In India, public debt refers to a part of the total borrowings by the Union Government which includes such items as market loans, special bearer bonds, treasury bills and special loans and securities issued by the Reserve Bank. It also includes the outstanding external debt. At end-March 2021, India’s external debt was placed at US$ 570.0 billion Public debt does not include the following items of borrowings: ·       small savings, ·       provident funds, ·       other accounts, reserve funds and deposits. Incorrect Solution (b) Statement 1 Statement 2 Statement 3 Correct Correct Incorrect In India, public debt refers to a part of the total borrowings by the Union Government which includes such items as market loans, special bearer bonds, treasury bills and special loans and securities issued by the Reserve Bank. It also includes the outstanding external debt. At end-March 2021, India’s external debt was placed at US$ 570.0 billion Public debt does not include the following items of borrowings: ·       small savings, ·       provident funds, ·       other accounts, reserve funds and deposits. Question 14 of 35 14. Question With reference to Primary Deficit, consider the following statements: The primary deficit is the sum of Fiscal deficit and interest payments. It includes the burden of the past debt. Which of the statements given above is/are correct? a) 1 only b) 2 only c) Both 1 and 2 d) Neither 1 nor 2 Correct Solution (d) Statement 1 Statement 2 Incorrect Incorrect Gross Primary deficit is defined as gross fiscal deficit minus net interest payments. Net primary deficit, is gross primary deficit minus net domestic lending. It excludes the burden of the past debt and shows the net increase in the government’s indebtedness due to the current year’s fiscal operations. A reduction in primary deficit is reflective of government’s efforts at bridging the fiscal gap during a financial year. Incorrect Solution (d) Statement 1 Statement 2 Incorrect Incorrect Gross Primary deficit is defined as gross fiscal deficit minus net interest payments. Net primary deficit, is gross primary deficit minus net domestic lending. It excludes the burden of the past debt and shows the net increase in the government’s indebtedness due to the current year’s fiscal operations. A reduction in primary deficit is reflective of government’s efforts at bridging the fiscal gap during a financial year. Question 15 of 35 15. Question With reference to Minimum Alternate Tax, consider the following statements: It is a direct tax imposed under Income Tax Act, 1961. It does not apply to any income accruing or arising to a company from life insurance business. It is not applicable to public corporate entities. How many of the statements given above is/are correct? a) Only one b) Only two c) All three d) None Correct Solution (b) Statement 1 Statement 2 Statement 3 Correct Correct Incorrect Minimum Alternate Tax (MAT) is a tax effectively introduced in India by the Finance Act of 1987, vide Section 115J of the Income Tax Act, 1961 (IT Act), to facilitate the taxation of ‘zero tax companies’ i.e., those companies which show zero or negligible income to avoid tax. It is a direct tax. Under MAT, such companies are made liable to pay to the government, by deeming a certain percentage of their book profit as taxable income. However, it does not apply to any income accruing or arising to a company from life insurance business. The applicability of MAT extends to all types of companies, including a private company, one-person company, public limited company and Nidhi company. Incorrect Solution (b) Statement 1 Statement 2 Statement 3 Correct Correct Incorrect Minimum Alternate Tax (MAT) is a tax effectively introduced in India by the Finance Act of 1987, vide Section 115J of the Income Tax Act, 1961 (IT Act), to facilitate the taxation of ‘zero tax companies’ i.e., those companies which show zero or negligible income to avoid tax. It is a direct tax. Under MAT, such companies are made liable to pay to the government, by deeming a certain percentage of their book profit as taxable income. However, it does not apply to any income accruing or arising to a company from life insurance business. The applicability of MAT extends to all types of companies, including a private company, one-person company, public limited company and Nidhi company. Question 16 of 35 16. Question In the context of the Indian Economy, what are Ways and Means advances? a) These are part of the infrastructure projects financing mechanism by the National Investment and Infrastructure Fund (NIIF). b) These are loans advanced by the World Bank to developing economies. c) These are temporary loans advanced by RBI to central and state governments. d) These are loans advanced by the IMF to overcome the medium-term balance of payments crisis. Correct Solution (c) The Reserve Bank of India gives temporary loan facilities to the centre and state governments as a banker to the This temporary loan facility is called Ways and Means Advances (WMA). This facility is provided to help them tide over temporary mismatches in the cash flow of their receipts and This is guided under Section 17(5) of the RBI Act, 1934.   Incorrect Solution (c) The Reserve Bank of India gives temporary loan facilities to the centre and state governments as a banker to the This temporary loan facility is called Ways and Means Advances (WMA). This facility is provided to help them tide over temporary mismatches in the cash flow of their receipts and This is guided under Section 17(5) of the RBI Act, 1934.   Question 17 of 35 17. Question Consider the following statements regarding the differences between Proportional and Progressive Taxation: In progressive taxation, tax rate increases with an increase in the income whereas, in proportional taxation same percentage of tax is levied on all taxpayers. As compared to progressive taxation, proportional taxation makes disposable income less sensitive to fluctuations in GDP. Which of the statements given above is/are correct? a) 1 only b) 2 only c) Both 1 and 2 d) Neither 1 nor 2 Correct Solution (c) Statement 1 Statement 2 Correct Correct A proportional tax is an income tax system where the same percentage of tax is levied on all taxpayers, regardless of their income. A proportional tax applies the same tax rate across low, middle, and high-income taxpayers. Whereas, progressive taxation is based on the taxable amount of an individual’s income. They follow an accelerating schedule, so high-income earners pay more than low-income earners. The proportional income tax acts as an automatic stabiliser – a shock absorber because it makes disposable income, and thus consumer spending, less sensitive to fluctuations in GDP as compared to  progressive taxation. When GDP rises, disposable income also rises but by less than the rise in GDP because a part of it is siphoned off as taxes. This helps limit the upward fluctuation in consumption spending. Incorrect Solution (c) Statement 1 Statement 2 Correct Correct A proportional tax is an income tax system where the same percentage of tax is levied on all taxpayers, regardless of their income. A proportional tax applies the same tax rate across low, middle, and high-income taxpayers. Whereas, progressive taxation is based on the taxable amount of an individual’s income. They follow an accelerating schedule, so high-income earners pay more than low-income earners. The proportional income tax acts as an automatic stabiliser – a shock absorber because it makes disposable income, and thus consumer spending, less sensitive to fluctuations in GDP as compared to  progressive taxation. When GDP rises, disposable income also rises but by less than the rise in GDP because a part of it is siphoned off as taxes. This helps limit the upward fluctuation in consumption spending. Question 18 of 35 18. Question Which of the following are the quantitative methods of credit control in India? Regulation of consumer credit Rationing Credit Discount Rate Policy Open Market Operations Select the correct answer using the code given below: a) 1 and 2 only b) 1, 2 and 3 only c) 3 and 4 only d) 1, 2, 3 and 4 Correct Solution (c) Statement 1 Statement 2 Statement 3 Statement 4 Incorrect Incorrect Correct Correct Regulation of consumer credit is a Qualitative method of money control. Rationing Credit credit is a Qualitative method of money control. Discount Rate Policy points to the rate of interest that is going to be paid by the banks and other financial institutions on the credits that they avail from the central bank through discount window loan procedure. This will infuse more liquidity in the market. Open Market Operations refers to buying and selling of bonds issued by the Government in the open market. When thes central bank want to infuse liquidity in the monetary system, it will buy government securities in the open market. Incorrect Solution (c) Statement 1 Statement 2 Statement 3 Statement 4 Incorrect Incorrect Correct Correct Regulation of consumer credit is a Qualitative method of money control. Rationing Credit credit is a Qualitative method of money control. Discount Rate Policy points to the rate of interest that is going to be paid by the banks and other financial institutions on the credits that they avail from the central bank through discount window loan procedure. This will infuse more liquidity in the market. Open Market Operations refers to buying and selling of bonds issued by the Government in the open market. When thes central bank want to infuse liquidity in the monetary system, it will buy government securities in the open market. Question 19 of 35 19. Question Which of the following was/were the financial sector reforms taken up by the Government of India post the economic reforms of 1991? Allowing Foreign Institutional Investors to invest in Indian financial markets. Necessary approval of RBI needed for all banks to set up new branches. Banks were allowed to generate resources from India and abroad. How many of the statements given above is/are correct? a) Only one b) Only two c) All three d) None Correct Solution (b) Statement 1 Statement 2 Statement 3 Correct Incorrect Correct Foreign Institutional Investors (FII), such as merchant bankers, mutual funds and pension funds, are allowed to invest in Indian financial markets. Those banks which fulfill certain conditions have been given freedom to set up new branches without the approval of the RBI and rationalize their existing branch networks.   Though banks have been given permission to generate resources from India and abroad, certain managerial aspects have been retained with the RBI to safeguard the interests of the account-holders and the nation. Incorrect Solution (b) Statement 1 Statement 2 Statement 3 Correct Incorrect Correct Foreign Institutional Investors (FII), such as merchant bankers, mutual funds and pension funds, are allowed to invest in Indian financial markets. Those banks which fulfill certain conditions have been given freedom to set up new branches without the approval of the RBI and rationalize their existing branch networks.   Though banks have been given permission to generate resources from India and abroad, certain managerial aspects have been retained with the RBI to safeguard the interests of the account-holders and the nation. Question 20 of 35 20. Question Which of the following category of people are exempted from income tax? a) Income of a member of a Scheduled Tribes who resides in any area in the State of Nagaland. b) Members of the Scheduled castes who resides in any area in the State of Rajasthan. c) Income of a member of a Scheduled Tribes who resides in any area in the State of Assam. d) Members of the Scheduled Tribes in any area in State of Himachal Pradesh. Correct Solution (a) Income of a member of a Scheduled Tribe [as per article 366(25) of the Constitution] is exempt from tax, if following conditions are satisfied: Such member resides in any area in the State of Nagaland, Manipur, Tripura, Arunachal Pradesh, Mizoram or district of North Cachar Hills, Mikir Hills, Khasi Hills, Jaintia Hills and Garo Hills or in the Ladakh region. Such exemption is available in respect of income which accrues/arises from any source in such areas or income by way of dividends/interest on securities arises from any area.   Incorrect Solution (a) Income of a member of a Scheduled Tribe [as per article 366(25) of the Constitution] is exempt from tax, if following conditions are satisfied: Such member resides in any area in the State of Nagaland, Manipur, Tripura, Arunachal Pradesh, Mizoram or district of North Cachar Hills, Mikir Hills, Khasi Hills, Jaintia Hills and Garo Hills or in the Ladakh region. Such exemption is available in respect of income which accrues/arises from any source in such areas or income by way of dividends/interest on securities arises from any area.   Question 21 of 35 21. Question Consider the following statements about Anthrax: It is a highly infectious disease that is caused by Bacillus anthracis. It affects animals like cows, sheep, goats, and wild herbivores. It can infect humans if they come in contact with infected animals. It can be treated with antibiotics such as ciprofloxacin, doxycycline, or levofloxacin. Choose the correct code: a) Only one b) Only two c) Only three d) All four Correct Solution (d) Anthrax is a highly infectious disease that is caused by Bacillus anthracis. Hence statement 1 is correct. Anthrax bacteria also occur naturally in soil. It affects animals like cows, sheep, goats, and wild herbivores. Hence statement 2 is correct. The disease manifests in three forms depending on the route of infection – cutaneous, gastrointestinal, and inhalational. It can infect humans if they come in contact with infected animals. Hence statement 3 is correct. It can be diagnosed by identifying Bacillus anthracis in blood, skin lesions, or respiratory secretions through laboratory culture, PCR, or ELISA tests. It can be treated with antibiotics such as ciprofloxacin, doxycycline, or levofloxacin. Hence statement 4 is correct. Vaccines are also available for both livestock and humans. Incorrect Solution (d) Anthrax is a highly infectious disease that is caused by Bacillus anthracis. Hence statement 1 is correct. Anthrax bacteria also occur naturally in soil. It affects animals like cows, sheep, goats, and wild herbivores. Hence statement 2 is correct. The disease manifests in three forms depending on the route of infection – cutaneous, gastrointestinal, and inhalational. It can infect humans if they come in contact with infected animals. Hence statement 3 is correct. It can be diagnosed by identifying Bacillus anthracis in blood, skin lesions, or respiratory secretions through laboratory culture, PCR, or ELISA tests. It can be treated with antibiotics such as ciprofloxacin, doxycycline, or levofloxacin. Hence statement 4 is correct. Vaccines are also available for both livestock and humans. Question 22 of 35 22. Question It is located in the state of Tamil Nadu. It lies adjacent to the Bay of Bengal along the east coast. It is one of the major wetlands on the Coromandel Coast after Pulicat Lake. The southern part of the wetland has been reserved land since 2001. It is listed as one of Tamil Nadu’s 141 prioritised wetlands, Kazhuveli is also a wetland of international significance and a potential Ramsar site. It has a feeding ground for long-distance migrants from the cold subarctic regions of Central Asia and Siberia including Black-tailed Godwits, Eurasian Curlew, White Stork, Ruff, and Dunlin. The above paragraph refers to which of the following? a) Kazhuveli Bird Sanctuary b) Vedanthangal Bird Sanctuary c) Vettangudi Bird Sanctuary d) Kanjirankulam Bird Sanctuary Correct Solution (a) Kazhuveli Bird Sanctuary is located in the state of Tamil Nadu. It lies adjacent to the Bay of Bengal along the east coast. It is one of the major wetlands on the Coromandel Coast after Pulicat Lake. The southern part of the wetland has been reserved land since 2001. It is listed as one of Tamil Nadu’s 141 prioritised wetlands, Kazhuveli is also a wetland of international significance and a potential Ramsar site. It has a feeding ground for long-distance migrants from the cold subarctic regions of Central Asia and Siberia including Black-tailed Godwits, Eurasian Curlew, White Stork, Ruff, and Dunlin. Hence option a is correct.   Incorrect Solution (a) Kazhuveli Bird Sanctuary is located in the state of Tamil Nadu. It lies adjacent to the Bay of Bengal along the east coast. It is one of the major wetlands on the Coromandel Coast after Pulicat Lake. The southern part of the wetland has been reserved land since 2001. It is listed as one of Tamil Nadu’s 141 prioritised wetlands, Kazhuveli is also a wetland of international significance and a potential Ramsar site. It has a feeding ground for long-distance migrants from the cold subarctic regions of Central Asia and Siberia including Black-tailed Godwits, Eurasian Curlew, White Stork, Ruff, and Dunlin. Hence option a is correct.   Question 23 of 35 23. Question Consider the following statements about the Directorate of Revenue Intelligence: It is the premier intelligence and enforcement agency on anti-smuggling matters. It works under the aegis of the Central Bureau of Investigation (CBI). It is headquartered in New Delhi. How many of the above statements are correct? a) Only one b) Only two c) All three d) None Correct Solution (b) The Directorate of Revenue Intelligence is the premier intelligence and enforcement agency on anti-smuggling matters. Hence statement 1 is correct. It has been carrying out its mandate of preventing and detecting cases of smuggling of narcotic drugs & psychotropic substances, wildlife items, notes, foreign currency, hazardous & environmentally sensitive materials, antiques, etc., and taking punitive action against the organised crime groups engaged therein. It works under the aegis of the Central Board of Indirect Taxes & Customs (CBIC), Government of India. Hence statement 2 is incorrect. It is also engaged in unearthing commercial frauds and instances of customs duty evasion. It is headquartered in New Delhi. Hence statement 3 is correct. Incorrect Solution (b) The Directorate of Revenue Intelligence is the premier intelligence and enforcement agency on anti-smuggling matters. Hence statement 1 is correct. It has been carrying out its mandate of preventing and detecting cases of smuggling of narcotic drugs & psychotropic substances, wildlife items, notes, foreign currency, hazardous & environmentally sensitive materials, antiques, etc., and taking punitive action against the organised crime groups engaged therein. It works under the aegis of the Central Board of Indirect Taxes & Customs (CBIC), Government of India. Hence statement 2 is incorrect. It is also engaged in unearthing commercial frauds and instances of customs duty evasion. It is headquartered in New Delhi. Hence statement 3 is correct. Question 24 of 35 24. Question Consider the following statements about Pompe Disease: It is caused due to mutations in the GAA gene. It affects males and females equally. It is treated through enzyme replacement therapy (ERT). How many of the above statements are correct? a) Only one b) Only two c) All three d) None Correct Solution (c) Pompe Disease is caused due to mutations in the GAA gene. Hence statement 1 is correct. The GAA gene provides instructions for producing an enzyme called acid alpha-glucosidase (also known as acid maltase). This enzyme is active in lysosomes, which are structures that serve as recycling centers within cells. The enzyme normally breaks down glycogen into a simpler sugar called glucose, which is the main energy source for most cells. Mutations in the GAA gene prevent acid alpha-glucosidase from breaking down glycogen effectively, which allows this sugar to build up to toxic levels in lysosomes. This buildup damages organs and tissues throughout the body, particularly the muscles, leading to the progressive signs and symptoms of Pompe disease. It affects males and females equally. Hence statement 2 is correct. Some common side effects and symptoms include muscle weakness, respiratory issues, heart problems, and difficulty swallowing. It is treated through enzyme replacement therapy (ERT). Hence statement 3 is correct. Incorrect Solution (c) Pompe Disease is caused due to mutations in the GAA gene. Hence statement 1 is correct. The GAA gene provides instructions for producing an enzyme called acid alpha-glucosidase (also known as acid maltase). This enzyme is active in lysosomes, which are structures that serve as recycling centers within cells. The enzyme normally breaks down glycogen into a simpler sugar called glucose, which is the main energy source for most cells. Mutations in the GAA gene prevent acid alpha-glucosidase from breaking down glycogen effectively, which allows this sugar to build up to toxic levels in lysosomes. This buildup damages organs and tissues throughout the body, particularly the muscles, leading to the progressive signs and symptoms of Pompe disease. It affects males and females equally. Hence statement 2 is correct. Some common side effects and symptoms include muscle weakness, respiratory issues, heart problems, and difficulty swallowing. It is treated through enzyme replacement therapy (ERT). Hence statement 3 is correct. Question 25 of 35 25. Question Consider the following statements: The Doctrine of Laches states that the court will only assist those individuals who are vigilant about their rights and not those who are negligent. The Doctrine of Eclipse asserts that a law that infringes fundamental rights is not null or void from the outset, but is merely non-enforceable. The Doctrine of Territorial Nexus dictates that laws made by a state legislature are not applicable outside that state unless there is a sufficient nexus between the state and the object. The Doctrine of Casus Omissus asserts that a matter which should have been provided for in a statute cannot be supplied by the courts. How many of the above statements are correct? a) Only one b) Only two c) Only three d) All four Correct Solution (d) The Doctrine of Laches states that the court will only assist those individuals who are vigilant about their rights and not those who are negligent. Hence statement 1 is correct. The Doctrine of Eclipse asserts that a law that infringes fundamental rights is not null or void from the outset, but is merely non-enforceable. Hence statement 2 is correct. The Doctrine of Territorial Nexus dictates that laws made by a state legislature are not applicable outside that state unless there is a sufficient nexus between the state and the object. Hence statement 3 is correct. The Doctrine of Casus Omissus asserts that a matter which should have been provided for in a statute cannot be supplied by the courts. Hence statement 4 is correct. The Doctrine of Promissory Estoppel is a legal principle that prevents a person from retracting a promise they made, even if the promise was not supported by a contract. The Doctrine of Colourable Legislation is a principle used to determine the legislative competence of laws enacted by various legislatures. The Doctrine of Harmonious Construction posits that a provision of the statute should not be interpreted in isolation but as a whole, to eliminate any inconsistency or repugnancy. The Doctrine of Incidental or Ancillary Powers suggests that the power to legislate on a particular issue also includes the power to legislate on ancillary matters that are reasonably connected to that issue. The Doctrine of Waiver implies that a person can intentionally relinquish their right or privilege or choose not to exercise their right or privilege conferred on them by the state Incorrect Solution (d) The Doctrine of Laches states that the court will only assist those individuals who are vigilant about their rights and not those who are negligent. Hence statement 1 is correct. The Doctrine of Eclipse asserts that a law that infringes fundamental rights is not null or void from the outset, but is merely non-enforceable. Hence statement 2 is correct. The Doctrine of Territorial Nexus dictates that laws made by a state legislature are not applicable outside that state unless there is a sufficient nexus between the state and the object. Hence statement 3 is correct. The Doctrine of Casus Omissus asserts that a matter which should have been provided for in a statute cannot be supplied by the courts. Hence statement 4 is correct. The Doctrine of Promissory Estoppel is a legal principle that prevents a person from retracting a promise they made, even if the promise was not supported by a contract. The Doctrine of Colourable Legislation is a principle used to determine the legislative competence of laws enacted by various legislatures. The Doctrine of Harmonious Construction posits that a provision of the statute should not be interpreted in isolation but as a whole, to eliminate any inconsistency or repugnancy. The Doctrine of Incidental or Ancillary Powers suggests that the power to legislate on a particular issue also includes the power to legislate on ancillary matters that are reasonably connected to that issue. The Doctrine of Waiver implies that a person can intentionally relinquish their right or privilege or choose not to exercise their right or privilege conferred on them by the state Question 26 of 35 26. Question Consider the following statements about the European Free Trade Association (EFTA): It is an intergovernmental organization established by the Washington Convention. It promotes free trade and economic integration between its members within Europe and globally. It currently has four member countries – Iceland, Liechtenstein, Norway, and Switzerland. How many of the above statements are correct? a) Only one b) Only two c) All three d) None Correct Solution (b) The European Free Trade Association (EFTA) is an intergovernmental organization established by the Stockholm Convention. Hence statement 1 is incorrect. EFTA is not a customs union because the individual EFTA States are free to set their own customs tariffs and arrange other foreign trade measures vis-à-vis the non-EFTA States. It promotes free trade and economic integration between its members within Europe and globally. Hence statement 2 is correct. The EFTA countries have developed one of the largest networks of Free Trade Agreements (FTAs). These FTAs span over 60 countries and territories, including the EU. The EFTA members are all open, competitive economies committed to the progressive liberalization of trade in the multinational arena as well as in free trade agreements. It currently has four member countries – Iceland, Liechtenstein, Norway, and Switzerland. Hence statement 3 is correct.   Note: Customs unions are groups of countries that apply one common system of procedures, rules, and tariffs for all or almost all their imports, exports, and transit goods. Usually, countries participating in customs unions share common trade and competition policies.   Incorrect Solution (b) The European Free Trade Association (EFTA) is an intergovernmental organization established by the Stockholm Convention. Hence statement 1 is incorrect. EFTA is not a customs union because the individual EFTA States are free to set their own customs tariffs and arrange other foreign trade measures vis-à-vis the non-EFTA States. It promotes free trade and economic integration between its members within Europe and globally. Hence statement 2 is correct. The EFTA countries have developed one of the largest networks of Free Trade Agreements (FTAs). These FTAs span over 60 countries and territories, including the EU. The EFTA members are all open, competitive economies committed to the progressive liberalization of trade in the multinational arena as well as in free trade agreements. It currently has four member countries – Iceland, Liechtenstein, Norway, and Switzerland. Hence statement 3 is correct.   Note: Customs unions are groups of countries that apply one common system of procedures, rules, and tariffs for all or almost all their imports, exports, and transit goods. Usually, countries participating in customs unions share common trade and competition policies.   Question 27 of 35 27. Question Consider the following statements about Exchange-Traded Funds (ETFs): It is a collection of marketable securities that tracks an index, a commodity, bonds, or a basket of assets. It can be purchased or sold on a stock exchange in the same way that regular stocks can. Choose the correct code: a) 1 only b) 2 only c) Both 1 and 2 d) Neither 1 nor 2 Correct Solution (c) Exchange-traded funds (ETFs) are a collection of marketable securities that track an index, a commodity, bonds, or a basket of assets. Hence statement 1 is correct. It can be structured to track anything from the price of an individual commodity to a large and diverse collection of securities. ETFs can even be structured to track specific investment strategies. It can be purchased or sold on a stock exchange in the same way that regular stocks can. Hence statement 2 is correct. The traded price of an ETF changes throughout the day like any other stock, as it is bought and sold on the stock exchange. The trading value of an ETF is based on the net asset value of the underlying stocks that it represents. Incorrect Solution (c) Exchange-traded funds (ETFs) are a collection of marketable securities that track an index, a commodity, bonds, or a basket of assets. Hence statement 1 is correct. It can be structured to track anything from the price of an individual commodity to a large and diverse collection of securities. ETFs can even be structured to track specific investment strategies. It can be purchased or sold on a stock exchange in the same way that regular stocks can. Hence statement 2 is correct. The traded price of an ETF changes throughout the day like any other stock, as it is bought and sold on the stock exchange. The trading value of an ETF is based on the net asset value of the underlying stocks that it represents. Question 28 of 35 28. Question Consider the following statements: The Global Cooling Watch Report was released by the UN Environment Programme (UNEP)-led Cool Coalition. The Global Cooling Pledge was signed at COP25 to address the growing demand for cooling while mitigating its climate impacts. Choose the correct code: a) 1 only b) 2 only c) Both 1 and 2 d) Neither 1 nor 2 Correct Solution (a) The Global Cooling Watch Report was released by the UN Environment Programme (UNEP)-led Cool Coalition. Hence statement 1 is correct. It is a comprehensive analysis of the global cooling sector and focuses not only on space cooling but also on cold chains for food and health. Its key highlights are: Currently, cooling equipment currently accounts for 20% of total electricity consumption and is expected to more than double by 2050. Under a business-as-usual scenario, emissions from cooling are predicted to account for more than 10 percent of global emissions in 2050. The Global Cooling Pledge was signed at COP28 to address the growing demand for cooling while mitigating its climate impacts. Hence statement 2 is incorrect. Around 63 countries have signed the pledge. However, India has not signed it yet. Incorrect Solution (a) The Global Cooling Watch Report was released by the UN Environment Programme (UNEP)-led Cool Coalition. Hence statement 1 is correct. It is a comprehensive analysis of the global cooling sector and focuses not only on space cooling but also on cold chains for food and health. Its key highlights are: Currently, cooling equipment currently accounts for 20% of total electricity consumption and is expected to more than double by 2050. Under a business-as-usual scenario, emissions from cooling are predicted to account for more than 10 percent of global emissions in 2050. The Global Cooling Pledge was signed at COP28 to address the growing demand for cooling while mitigating its climate impacts. Hence statement 2 is incorrect. Around 63 countries have signed the pledge. However, India has not signed it yet. Question 29 of 35 29. Question Consider the following statements: Convertible preference shares are those shares that can be easily converted into equity shares. Non-convertible preference shares are those shares that cannot be converted into equity shares. Redeemable preference shares are those shares that can be repurchased or redeemed by the issuing company at a fixed rate and date. Non-redeemable preference shares are those shares that cannot be redeemed or repurchased by the issuing company at a fixed date. Choose the correct code: a) Only one b) Only two c) Only three d) All four Correct Solution (d) Convertible preference shares are those shares that can be easily converted into equity shares. Hence statement 1 is correct. Non-convertible preference shares are those shares that cannot be converted into equity shares. Hence statement 2 is correct. Redeemable preference shares are those shares that can be repurchased or redeemed by the issuing company at a fixed rate and date. Hence statement 3 is correct. These types of shares help the company by providing a cushion during times of inflation. Non-redeemable preference shares are those shares that cannot be redeemed or repurchased by the issuing company at a fixed date. Hence statement 4 is correct. These types of shares help companies by acting as a lifesaver during times of inflation. Incorrect Solution (d) Convertible preference shares are those shares that can be easily converted into equity shares. Hence statement 1 is correct. Non-convertible preference shares are those shares that cannot be converted into equity shares. Hence statement 2 is correct. Redeemable preference shares are those shares that can be repurchased or redeemed by the issuing company at a fixed rate and date. Hence statement 3 is correct. These types of shares help the company by providing a cushion during times of inflation. Non-redeemable preference shares are those shares that cannot be redeemed or repurchased by the issuing company at a fixed date. Hence statement 4 is correct. These types of shares help companies by acting as a lifesaver during times of inflation. Question 30 of 35 30. Question Consider the following statements about Chousingha: It is endemic to the Indian subcontinent. It is the smallest antelope found in Asia. It is listed as endangered on the IUCN Red List. How many of the above statements are correct? a) Only one b) Only two c) All three d) None Correct Solution (b) Chousingha is endemic to the Indian subcontinent. Hence statement 1 is correct. The four-horned antelope, or chousingha, is a small antelope found in India and Nepal. It is the smallest antelope found in Asia. Hence statement 2 is correct. They are usually diurnal and solitary by nature; however, they can be spotted in loose groups of three to four. Animals are sedentary, inhabiting more or less the same region throughout their lives. It is listed as vulnerable on the IUCN Red List. Hence statement 3 is incorrect. Incorrect Solution (b) Chousingha is endemic to the Indian subcontinent. Hence statement 1 is correct. The four-horned antelope, or chousingha, is a small antelope found in India and Nepal. It is the smallest antelope found in Asia. Hence statement 2 is correct. They are usually diurnal and solitary by nature; however, they can be spotted in loose groups of three to four. Animals are sedentary, inhabiting more or less the same region throughout their lives. It is listed as vulnerable on the IUCN Red List. Hence statement 3 is incorrect. Question 31 of 35 31. Question The diagonal of a rectangle is √21 cm and its area is 50 sq. cm. What is the perimeter of the rectangle? (Note – Diagonal equals the square root of the width squared plus the height squared) a) 11 b) 22 c) 14 d) 28 Correct Solution (b) For a rectangle, d^2 = l^2 + b^2 where l= length, b = breadth and d = diagonal of the of the rectangle   d= √21 d^2 = l^2 + b^2 l^2 + b^2= 21 ….. (1) Area = l × b = 50 …..(2)   We know that (a+b)^2 = a^2 + b^2 + 2ab Using the same formula, we have (l+b)^2 = l^2 + b^2 + 2lb (l+b)^2 = 21 + 2*50 (l+b) = 11 Perimeter of the rectangle = 2(l + b) = 2(11) = 22 cm   Incorrect Solution (b) For a rectangle, d^2 = l^2 + b^2 where l= length, b = breadth and d = diagonal of the of the rectangle   d= √21 d^2 = l^2 + b^2 l^2 + b^2= 21 ….. (1) Area = l × b = 50 …..(2)   We know that (a+b)^2 = a^2 + b^2 + 2ab Using the same formula, we have (l+b)^2 = l^2 + b^2 + 2lb (l+b)^2 = 21 + 2*50 (l+b) = 11 Perimeter of the rectangle = 2(l + b) = 2(11) = 22 cm   Question 32 of 35 32. Question A and B playing a game with rolling a fare dice, one after the other replacing it every time till one of them gets a six. If A begins the game, then the probability that A wins the game is a) 1/6 b) 2/6 c) 5/11 d) 6/11 Correct Solution (c) Let  S  denote  the  success  (getting  a  ‘6’)  and  F  denote  the  failure  (not  getting  a  ‘6’)  . Thus, P(S)= 1/6 ​ =p, P(F)= 5/6​ =q P(A wins  in  first  throw)=P(S)=p P(A  wins  in  third  throw)=P(FFS)=qqp P(A  wins  in  fifth  throw)=P(FFFFS)=qqqqp So,  P(A  wins)=p+qqp+qqqqp+… =p(1+q^2 +q^4 +…) = 1−q^2 p ​ = (1/6)/(1− 25/36) ​ ​ =  6/11 ​ P(B  wins)=1–P(A  wins) P(B  wins)  =1− 6/11 ​ = 5/11 ​ So,  P(A  wins)= 6/11 ​ ,  P(B  wins)= 5/11 Incorrect Solution (c) Let  S  denote  the  success  (getting  a  ‘6’)  and  F  denote  the  failure  (not  getting  a  ‘6’)  . Thus, P(S)= 1/6 ​ =p, P(F)= 5/6​ =q P(A wins  in  first  throw)=P(S)=p P(A  wins  in  third  throw)=P(FFS)=qqp P(A  wins  in  fifth  throw)=P(FFFFS)=qqqqp So,  P(A  wins)=p+qqp+qqqqp+… =p(1+q^2 +q^4 +…) = 1−q^2 p ​ = (1/6)/(1− 25/36) ​ ​ =  6/11 ​ P(B  wins)=1–P(A  wins) P(B  wins)  =1− 6/11 ​ = 5/11 ​ So,  P(A  wins)= 6/11 ​ ,  P(B  wins)= 5/11 Question 33 of 35 33. Question When a particular positive number is divided by 3, the remainder is 1. If the same number is divided by 5, the remainder is 4. If the difference between the quotients of the division is 3, then find the number. a) 34 b) 19 c) 49 d) 64 Correct Solution (b) Let the quotients when this number is divided by 3 and 5 be x and y respectively. (Note that x will be greater than y as 3 is smaller than 5) Number = 3x + 1 = 5y + 4 Given that, x – y = 3 On solving both equation we get, x = 6, y=3 Thus the number is 5 × 3 + 4= 19. Incorrect Solution (b) Let the quotients when this number is divided by 3 and 5 be x and y respectively. (Note that x will be greater than y as 3 is smaller than 5) Number = 3x + 1 = 5y + 4 Given that, x – y = 3 On solving both equation we get, x = 6, y=3 Thus the number is 5 × 3 + 4= 19. Question 34 of 35 34. Question In 10 years, A will be thrice as old as B was 5 years ago. If A is now 9 years older than B, the present age of B is a) 19 b) 21 c) 17 d) 15 Correct Solution (c) Let B’s present age = x years. Then, A’s present age = (x + 9) years. (x + 9) + 10 = 3(x – 5) => x + 19 = 3x – 15 => x =17 Incorrect Solution (c) Let B’s present age = x years. Then, A’s present age = (x + 9) years. (x + 9) + 10 = 3(x – 5) => x + 19 = 3x – 15 => x =17 Question 35 of 35 35. Question Out of the four annual examinations, each with a total of 100 marks, a student secured average marks of 35%, 65% and 50% in the first, second and third annual examinations. To have an overall average of 60%, how many marks does the student need to secure in the fourth annual examination? a) 60 b) 70 c) 80 d) 90 Correct Solution (d) Let the average marks in the third Annual examination be x. Total marks = (Marks in first + second + third + forth) Annual examination = > 4(60) ( 100/100) = ( 35/100) (100) + ( 65/100) (100) + ( 50/100) (100) + ( x/100) (100) => 4(60) = 35 + 65 +50 + x =>x = 90 So, the student must score 90% in the fourth annual examination to secure 60% overall average. ∴ Average marks in the third annual examination (90/100) × 100 = 90 marks.   Incorrect Solution (d) Let the average marks in the third Annual examination be x. Total marks = (Marks in first + second + third + forth) Annual examination = > 4(60) ( 100/100) = ( 35/100) (100) + ( 65/100) (100) + ( 50/100) (100) + ( x/100) (100) => 4(60) = 35 + 65 +50 + x =>x = 90 So, the student must score 90% in the fourth annual examination to secure 60% overall average. ∴ Average marks in the third annual examination (90/100) × 100 = 90 marks.   window.wpProQuizInitList = window.wpProQuizInitList || []; window.wpProQuizInitList.push({ id: '#wpProQuiz_3623', init: { quizId: 3623, mode: 1, globalPoints: 70, timelimit: 1800, resultsGrade: [0], bo: 704, qpp: 0, catPoints: [70], formPos: 0, lbn: "Test-summary", json: {"31922":{"type":"single","id":31922,"catId":0,"points":2,"correct":[0,0,0,1]},"31924":{"type":"single","id":31924,"catId":0,"points":2,"correct":[0,0,0,1]},"31925":{"type":"single","id":31925,"catId":0,"points":2,"correct":[0,1,0,0]},"31927":{"type":"single","id":31927,"catId":0,"points":2,"correct":[0,1,0,0]},"31930":{"type":"single","id":31930,"catId":0,"points":2,"correct":[0,1,0,0]},"31932":{"type":"single","id":31932,"catId":0,"points":2,"correct":[0,0,1,0]},"31934":{"type":"single","id":31934,"catId":0,"points":2,"correct":[0,0,0,1]},"31936":{"type":"single","id":31936,"catId":0,"points":2,"correct":[0,1,0,0]},"31937":{"type":"single","id":31937,"catId":0,"points":2,"correct":[0,1,0,0]},"31939":{"type":"single","id":31939,"catId":0,"points":2,"correct":[0,1,0,0]},"31941":{"type":"single","id":31941,"catId":0,"points":2,"correct":[0,1,0,0]},"31943":{"type":"single","id":31943,"catId":0,"points":2,"correct":[0,1,0,0]},"31945":{"type":"single","id":31945,"catId":0,"points":2,"correct":[0,1,0,0]},"31948":{"type":"single","id":31948,"catId":0,"points":2,"correct":[0,0,0,1]},"31950":{"type":"single","id":31950,"catId":0,"points":2,"correct":[0,1,0,0]},"31952":{"type":"single","id":31952,"catId":0,"points":2,"correct":[0,0,1,0]},"31955":{"type":"single","id":31955,"catId":0,"points":2,"correct":[0,0,1,0]},"31957":{"type":"single","id":31957,"catId":0,"points":2,"correct":[0,0,1,0]},"31960":{"type":"single","id":31960,"catId":0,"points":2,"correct":[0,1,0,0]},"31961":{"type":"single","id":31961,"catId":0,"points":2,"correct":[1,0,0,0]},"31963":{"type":"single","id":31963,"catId":0,"points":2,"correct":[0,0,0,1]},"31966":{"type":"single","id":31966,"catId":0,"points":2,"correct":[1,0,0,0]},"31968":{"type":"single","id":31968,"catId":0,"points":2,"correct":[0,1,0,0]},"31969":{"type":"single","id":31969,"catId":0,"points":2,"correct":[0,0,1,0]},"31970":{"type":"single","id":31970,"catId":0,"points":2,"correct":[0,0,0,1]},"31972":{"type":"single","id":31972,"catId":0,"points":2,"correct":[0,1,0,0]},"31975":{"type":"single","id":31975,"catId":0,"points":2,"correct":[0,0,1,0]},"31976":{"type":"single","id":31976,"catId":0,"points":2,"correct":[1,0,0,0]},"31979":{"type":"single","id":31979,"catId":0,"points":2,"correct":[0,0,0,1]},"31981":{"type":"single","id":31981,"catId":0,"points":2,"correct":[0,1,0,0]},"31982":{"type":"single","id":31982,"catId":0,"points":2,"correct":[0,1,0,0]},"31984":{"type":"single","id":31984,"catId":0,"points":2,"correct":[0,0,1,0]},"31986":{"type":"single","id":31986,"catId":0,"points":2,"correct":[0,1,0,0]},"31988":{"type":"single","id":31988,"catId":0,"points":2,"correct":[0,0,1,0]},"31991":{"type":"single","id":31991,"catId":0,"points":2,"correct":[0,0,0,1]}} } }); All the Best IASbaba